You are on page 1of 129

e-Notes by T.N.

Sridhara, NIE, Mysore


Unit 2: Work & Heat
Mechanics definition of work: Work is done when the point of application of a
force moves in the direction of the force. The amount of work is equal to the product of
the force and the distance through which the point of application moves in the direction
of the force. i.e., work is identified only when a force moves its point of application
through an observable distance.
Mathematically, W = dx F.
2
1

However, when treating thermodynamics from a macroscopic point of view, it is


advantageous to tie in the definition work with the concepts of systems, properties and
processes.
Thermodynamic definition of work: It is a kind of interaction that would
occur at the system boundaries. It can be positive or negative.
Definition of Positive work is said to be done by a system when the sole effect external
to the system could be reduced to the raising of a weight.
Comments: The word sole effect indicates that the raising of weight should be the
only interaction between the system and surroundings in order to say that there is work
interaction between the system and the surroundings. The phrase external to the system
indicates that the work is a boundary phenomenon. The magnitude of work interaction
depends upon the system boundary. This is illustrated with an example.
Figure 1: Equivalence of Current Work Interaction between the System and the
Surroundings
1
Figure 2: System Comprising of Battery, Switch & Resistance Coil
For the two systems shown in figure, system (1) comprising battery alone has work
interaction with the surroundings, whereas for system (2) which includes motor, weights
etc along with the battery, the work interaction is zero.
The word could be reduced to indicates that it is not necessary that weights should
actually be raised in order to say that there is work interaction between the system and the
surroundings. It is just sufficient to have an effect which is equivalent to the raising of
weight.
Here an electrical storage battery constitutes system 1 whose terminals are connected to
an electrical resistance coil through a switch. The circuit external to the battery
constitutes the surroundings. When the switch is closed, the current flow through the coil,
and the resistance (surroundings) become warmer and the charge of the battery (system)
decreases. Obviously there has been interaction between the system and the surroundings.
According to mechanics this interaction cannot be classified as work because their has
been no action of force through a distance or of torque through an angle. However, as per
thermodynamics concepts, the battery (system) does work as the electrical energy crosses
the system boundary. Further, the electrical resistance can be replaced by an ideal
frictionless motor pulley arrangement which can wind a string and thereby raise
suspended weight. The sole effect, external to the system, is raising of a weight. As such
interaction of battery with resistance coil is a work.
Sign Conventions for work:
Work is said to be positive, if it is done by the system on the surroundings
Positive work
Work is said to be negative, if it is done on the system by the surroundings
Negative work
The unit of work is N-m or Joule. The rate at which work is done by, or upon, the
system is known as power. The unit of power is J/s or watt.
2
System
System
Therefore, W
system
+ W
surroundings
= Zero
Work is one of the forms in which a system and its surroundings can interact with each
other. There are various types of work transfer which can get involved between them.
Work done at the moving boundary of a system (Expression for
displacement work)
Consider a piston-cylinder arrangement which contains certain working fluid undergoing
quasi-static process.
Let p = Pressure exerted by the fluid on the piston
A = Area of c/s of the cylinder
dx = displacement of the piston when the system has undergone an infinitesimal
change
of state.
Displacement work: dw = Force x displacement
= p.A x dx
i.e., dw = p.dV
Where dv is the infinitesimal change in volume of the system. If the system undergoes a
finite change of state from state (1) to state (2). Then the displacement work is given by
dV p dw .
2
1
2
1

The integration of above equation can be done only if the relationship between P and v
during the process is known i.e., if the path of the process is well defined. Hence, work is
a path function. As work depends on the path of the process which it follows, there will
be different values of work for different process between two given states. Hence the
differentials of the path functions are in exact differentials. The symbol will be used to
designate inexact differentials. The magnitude of the work transfer by the system during
the process from state (1) to state (2) containing unit mass of the fluid will be written as,
2 1
2
1
W w or W
1-2
.
The process can be represented by a full line on an appropriate thermodynamic
coordinate system (in this case p-V diagram) and the area under the curve gives the work
done by the system during the process.
3
p
V
dV
1
2
Quasi-Static
Process
p
V
p
1
p
2
V
2
V
1
1
2
A
B
C
p
dx
Quasi-Static pdV work Work-a path function
Inspection of the pV diagram above shows that just by specifying the end states 1 and 2
does not determine the area (or work); the nature of the curve needs to be known. The
curve may be arched upwards or it may sag downwards, and the area under the curve will
vary accordingly. For the same initial and final states, the work done by the system in
following the paths A, B and C are different. Therefore the work is a path function and
not a point function. Accordingly the work transfer across the system boundaries is not
classified as a thermodynamic property.

The expression w = pdV holds good under the following restrictions
i) The system is closed
ii) There is no friction within the system
iii) The pressure and all other properties are the same on all the boundaries of the
system
iv) The system is not influenced by motion, gravity, capillarity, electricity and
magnetism
Expression for Displacement work for various Quasi-Static
Processes (pdV work):
1. Constant volume process: (Isochoric Process).
For a constant volume process i.e., V = constant (dV = 0 ) as represented in the p-V
diagram below.
W
1-2
=
2
1

p.dV but dV = 0

2. Constant pressure process: (Isobaric process).


4
(W
d
)
1-2
= 0
p
V
1
2
p
1
p
2
For a closed system undergo a constant pressure process from state 1 (volume V
1
and
pressure p
1
) to a final state 2 (volume V
2
). The process is represented in the p-V
diagram as shown below.
W
1-2
=
2
1

p.dV, where p = constant


W
1-2
= p
2
1

dV = p (V
2
V
1
)

3. Hyperbolic process i.e., pV = constant:


The hyperbolic expansion process from state 1 to state 2 is represented on a p-V
diagram as shown below.
Process in which pV = Constant
W
1-2
=
2
1

p.dV
But pV = constant i.e., pV = p
1
V
1
,
V
V p
p
1 1

W
1-2
=
2
1

p.dV
dV
V
V p
.
1 1 2
1

1 1
V p
V
dV
2
1

= p
1
V
1
[ln V
2
ln V
1
] where p
1
= Initial pressure of the system
V
1
= Initial volume of the system
5
(W
d
)
1-2
= p (V
2
V
1
)
V
P
1 2
V
1
V
2
W
1-2
p
1
= p
2
p
2
= Final pressure of the system
V
2
= Final volume of the system
i.e., (W
d
)
1-2
=
1
2
1 1
ln
V
V
V p


4. Polytropic process, i.e., pV
n
= constant
A polytropic process is represented on a p-V diagram as shown below.
Figure: Process in which pV
n
= Constant
W
1-2
=
2
1

p.dV
But pV
n
= constant i.e.,
n n n
V p V p pV
2 2 1 1

n
n
V
V p
p
1 1

W
1-2
=
2
1

dV
V
V p
n
n
.
1 1
= p
1
V
1
n

2
1

V
-n
.dV
= p
1
V
1
n

2
1
1
1
1
]
1

+
+
n
V
n
= [ ]
n n
n
V V
n
V p

1
1
1
2
1 1
1
=
n
V p v V p
n n

1
.
1 1
1
2 1 1
but p
1
V
1
n
= p
2
V
2
n
=
n
V p V p

1
1 1 2 2
(W
d
)
1-2
=
1
2 2 1 1

n
V p V p
Where n is called the index of expansion or compression
Note: 1. Work is a transient phenomenon i.e., it is present only during a process.
Mathematically
6
speaking, work is a path function.

2
1

dw = w
2
w
1
is wrong
= w
1-2
i.e., w is inexact differentials.
2. For irreversible process w
2
1

P.dv
7
Other Types of Work Transfer
1. Shaft Work:
Shaft work
Let F
t
= Tangential force on the shaft
R = Radius of the shaft
d = Angular displacement of the shaft in an interval of time dt
Shaft work in time interval dt, is dW
s
= F
t
. AA
1
= F
t
. R.d
i.e., W
s
=T.d
Work done / unit time =

dt
dW
s
T.
dt
d
= T. where = angular velocity, T =
Torque
But =
60
2 N
where N = rpm of the shaft
Shaft work, W
s
=
60
2 NT
watts
2. Stirring Work: Stirring work is nothing but shaft work is done on the system by
using a stirrer which is driven by a shaft.
Figure3: Paddle-wheel work
As the weight is lowered, and the paddle wheel turns, there is work transfer into the
system which gets stirred. Since the volume of the system remains constant, pdv = 0. If
8
m is the mass of the weight lowered through a distance dz and T is the torque transmitted
by the shaft in rotating through an angle d , the differential work transfer to the fluid is
given by
w = mgdz = Td
and the total work transfer is w =
2
1

mgdz =
2
1

W
1
dz =
2
1

Td where W
1
is
the weight lowered
3. Electrical Work: When a current flows through a resistor, taken as system, there
is work transfer into the system. This is because the current can drive a motor, the motor
can drive a pulley and the pulley can raise a weight.

The current I, flows is given by,
I =
d
dC
where C = charge in coulombs
= time in seconds
Thus dC is the charge crossing a system boundary during time d . If E is the voltage
potential, the work is w = E.dC
= EI d
w =
2
1

EI d
The electrical work is, w = lim
d
dw
= EI
This is the rate at which work is transferred.
4. Work done in stretching a wire: Consider a wire as the system. If the
length of the wire in which there is a tension is changed from L to L + dL, the
infinitesimal amount of work that is done is equal to, w = - dL
The -ve sign is used because a positive value of dL means an expansion of the wire,
for which work must be done on the wire i.e., negative work.
For a finite change of length, w = -
2
1

dL
With in the elastic limits, if E is the modulus of the elasticity, is the stress, is the
strain, and A is the cross sectional area, then
= A = E..A
9
0
I
I
System boundary
Therefore w = - E..AdL
But d = dL/L or dL = L x d
w = - dL = - E..A. L d
i.e., w = -EAL
2
1

d
= -
( )
2
1
2
2
2

L A
5. Work done in changing the area of a surface film: A film on the
surface of a liquid has a surface tension which is a property of the liquid and the
surroundings. The surface tension acts to make the surface area of the liquid a minimum.
It has the unit of force per unit length.
The work done on a homogeneous liquid film in changing its surface area by an
infinitesimal amount dA is
w = - dA when = surface tension (N/m)
w = -
2
1

dA
6. Magnetization of a paramagnetic field: The work done per unit volume
on a magnetic material through which the magnetic and magnetization fields are uniform
is,
w = - H.dI
i.e., w = -
2
1

H.dI
Where H = field strength
I = Component of the magnetization field in the direction of the field.
-ve sign provides that an increase in magnetization (+ve dI) involves -ve work.
Note: It may be noted in the above expressions that the work is equal to the integral of the
product of an intensive property and the change in its related extensive property. These
expressions are valid only for infinitesimally slow quasi-static process.
Network Transfer: The network interaction between the system and the
surroundings for any process will be the algebraic sum of all types of work interaction
that has taken place between the system and the surroundings.
Therefore if W
1-2
represents the net work transfer then,
W
1-2
= (W
d
)
1-2
(W
s
)
1-2
(W
e
)
1-2
(W
mag
)
1-2
......
+ve sign has to be used when the work transfer takes place from the system to the
surroundings and ve sign to be used when work transfer is from the surroundings to the
system.
10
Heat: Heat is a mode of energy transfer that takes place between the system and the
surroundings solely due to the temperature difference. Thus, heat is a transient
phenomenon. It can be recognized only during a process. It is not a thermodynamic
property of the system. Like work, heat is a path function i.e., the magnitude of heat
transfer between the system and surroundings depends upon the type of process the
system is undergoing.
Heat transfer always takes place from a region of higher temperature to a region of low
temperature. The magnitude of the heat transfer into unit mass of the fluid in the system
during a process from state (1) to state (2) will be written as
2 1 2 1
2
1
q or q q and not as
2 1
2
1
q q q
q
2
1
represents the total heat transfer that takes place when the system undergoes a
change of state from state 1 to state 2.
Sign Convention:
Heat transfer is considered as positive if it takes place from the surroundings to the
system and it is considered as negative if it takes place from the system to the
surroundings.
During an adiabatic process, Q = 0
Units: Since heat is a form of energy transfer it will have the same units as that of
energy. In SI units it is expressed in Joules (J) or Kilo Joules (kJ).
Comparison between work and heat:
Similarities:
Both are path functions and inexact differentials.
Both are boundary phenomenon i.e., both are recognized at the boundaries of the
system as they cross them.
Both represent transient phenomenon; these energy interactions occur only when
a system undergoes change of state i.e., both are associated with a process, not a state.
Unlike properties, work or heat has no meaning at a state.
A system possesses energy, but not work or heat.
Concepts of heat and work are associated not with a store but with a process.
Dissimilarities:
Heat is energy interaction due to temperature difference only; work is by reasons
other than temperature difference.
In a stable system, there cannot be work transfer; however there is no restriction
for the transfer of heat.
11
System
System
+ve -ve
The sole effect external to the system could be reduced to rise of a weight but in
the case of a heat transfer other effects are also observed.
Heat is a low grade energy whereas work is a high grade energy.
Problems:
1. Evaluate the work done in the following processes. The systems to be
considered are underlined.
a) An agent slowly raises a body of mass 2 kg a distance of 3 mts in
a gravitational field of standard acceleration.
Solution: By definition, considering agent as the system, it does positive work. The
magnitude of work is measured by the product of the weight its lifts and the distance
through which it is lifted.
W = +
l mg .
= 2 x 9.81 x 3 = + 58.86 J
Work done by the agent = + 58.86 J
Because, work is done by the agent, work is done on the body to the same amount.
Work done on the body = 58.86 J
or Work done by the body = - 58.86 J
b) A mass of 1 tonne is suspended from a pulley block. An agent
slowly raises the mass against the standard gravitational acceleration by 2m.
Solution: By definition, considering agent as the system, it does positive work.
W = +
l mg .
=
2 81 . 9 1000 x x
= 19620 J = 19.62 kJ
Work done by the agent = 19.62 kJ
Because, work is done by the agent, work is done on the mass to the same amount.
Work done on the mass = 19.62 kJ
or Work done by the mass = - 19.62 kJ
c) After raising the mass as in (b), the mass falls freely through the
same vertical distance of 2 m. The drag force of the atmosphere on the body is 50
N.
Actual Surroundings Fictitious Surroundings
Solution: Considering mass as a system (w r t. fictitious surroundings)
W = F x l = 50 x 30 = 1500 J = 1.5 kJ
Work done by the mass = 1.5 kJ
Work done by the atmosphere = - 1.5 kJ
12
1 tonne
Zero
mass
50 N
+
1 tonne
50 N
No drag
Force
+
Or Work done on the atmosphere = 1.5 kJ
d) A body of mass 15 kg falls freely in a vacuum through a vertical
distance of 30m. The gravitational acceleration is 6 m/s
2
.
Solution: Considering body as a system, as it is falling freely there is no interaction with
the system boundary and hence work done by the body is zero. (In other words, the work
done by the body is zero as it can lift no weight. All that is happening as the body is
falling freely is that its PE is decreasing and its KE is increasing accordingly).
e) A rat weighing 5.0 N climbs a stair 0.2 m in height.
Solution: W
rat
= 0 (since there is no interaction between the system and its surroundings).

2. Indicate in the following cases, the heat exchange and work exchange are
positive, negative or zero, and why
a) A copper block of 1 kg heated to 100
0
C is dipped into water at
15
0
C. Consider copper as system.
Ans: W = 0, Q is negative
b) Heat is added to a gas in a rigid container such that pressure and
temperature increases. Consider gas as system,
Ans: W = 0, Q is positive.
c) Gas from a bottle is used to inflate a balloon which is originally
flat. Consider gas as system.
Ans: W is Positive, Q = 0
d) An insulated wire is stretched. Consider wire as a system.
Ans: W is negative, Q = 0
e) A mouse climbs 20 steps of a stair case. Consider mouse as the
system.
Ans: Q = W = 0
f) Gas in an insulated cylinder expands as the piston is slowly
moved outwards.
Ans: Q = 0, W is positive
g) A closed rigid vessel containing steam at a temperature of 200
0
C
is left standing in an atmosphere which is at 20
0
C. Consider steam as the system.
Ans: For a closed rigid vessel, there is no change in volume and accordingly work
done is zero. i.e., W = 0. Since the steam is at a temperature higher than that of the
surrounding atmosphere, the heat is rejected to the atmosphere. i.e., heat interaction is
negative or Q is negative.
h) The air in a tyre and connecting pump the pump plunger is
pushed down, forcing air into the tyre. The tyre, pump walls and connecting tube
can be thought of to be non-conducting. Consider air as a system.
Ans: W is negative and Q = 0
i) An electric current flows steadily through a resistor which is
immersed in running water.
Ans: Considering resistor as system, current flows through the resistance i.e., electrical
work is done on the system W
e
is negative
13
Due to the flow of current the resistor gets heated up resulting in heat transfer to the
surrounding cold water from the resistor.
Q is negative (heat transfer from the system)
j) A container with rigid non-conducting walls holds a complete
electrical circuit consisting of a heating element and charged storage battery. The
temperature and pressure of the air in the container increases.
Ans: No interaction taking place across the boundary. The system boundary does not
move as the walls are rigid. W = 0
The walls are non-conducting though the temperature inside the system increase,
no heat transfer to the surroundings can take place Q = 0

k) 0.1 kg of gas contained in an insulated cylinder expands moving
the piston slowly outwards
Ans: The cylinder is insulated No. heat transfer is possible Q = 0
The gas expands the system boundary expands W
d
is positive
Problems:
3. Gas from a bottle of compressed helium is used to inflate a balloon
originally folded completely flat, to a volume of 0.25 m
3
. If the barometer reads 760
mm of mercury, how much work is done by the system comprising the helium
initially in the bottle, if the balloon is light and requires no stretching. Sketch the
system before and after the process.
Solution:
The firm line S
1
shows the boundary of the system before the process, and the dotted line
S
2
shows the boundary after the process.
Total displacement work is given by
(W
d
)
1-2
= (W
d
)
bottle portion of the system
+ (W
d
)
Balloon portion of the system
Since there is no displacement of the bottle portion of the system boundary, it follows
that (W
d
)
bottle
= 0
14
= 0 + (W
d
)
Balloon
The balloon is expanding against a constant atmosphere pressure of 760 mm of mercury,
i.e., p = wh
= 9810 (13.6) 0.76 = 1.01396 x 10
5
N/m
2
= 101.396 kPa
(W
d
)
balloon
= pdV = p
2
1

dV
= p (V
2
V
1
)
= 101.396 (0.25 0)
= 25.349 kJ
4. Determine the work done by the air which enters an evacuated bottle from
the atmosphere when the cork is opened, atmospheric air rushes into it. If the
atmospheric pressure is 101.396 kPa and 0.6m
3
of air (measured at atmosphere
conditions) enters the bottle.
Solution:
No work is done by the part of the boundary in contact with the bottle. Work is done only
by the moving part external to the bottle. The pressure over this moving part is uniform at
101.396 kPa
Displacement work done by the system,
W
d
= (W
d
)
bottle
+ (W
d
)
atmosphere

= 0 +
2
1

p.dV
= p (V
2
V
1
) = 101.396 (0 0.6)
= - 60.8 kJ
Negative, because the boundary is contracting. Thus the surroundings do positive work at
the boundary and the work done by the air negative
5. A spherical balloon has a diameter of 25 cm and contains air at a pressure
of 1.5 x 10
5
Pa. The diameter of the balloon increases to 30 cm in a certain process
and during this process the pressure is proportional to the diameter. Calculate the
work done by the air inside the balloon during this process.
Solution:
15
D
1
D
2
State 1 (p
1
, D
1
)
State 2 (p
2
, D
2
)
D
1
= 0.25 m, D
2
= 0.3 m,
p D,
p
1
= 1.5 x 10
5
N/m
2
We have W
1-2
=
2
1

p.d V --- (1)


But, volume of sphere = V =
6
3
D
dV =
6

. 3D
2
.dD
Also, p D i.e.,
D
p
= constant = K
2
2
1
1
D
p
D
p
D
p


D
D
p
p .
1
1

Equation (1) becomes, W
1-2
=
2
1


. .
1
1
D
D
p
3
6

D
2
. dD
=
1
1
,
2 D
p
2
1

D
3
=
2
1
4
1
1
4 2
1
]
1

D
D
p
=
1
]
1


4
.
2
4
1
4
2
1
1
D D
D
p
=
25 . 0
10 5 . 1
.
2
5
x
1
]
1


4
25 . 0 3 . 0
4 4
W
1-2
= 988.13 J
Positive sign indicates that work is done by the system.

6. Gas from a bottle of compressed helium is used to inflate an inelastic
flexible balloon, originally folded completely flat to a volume of 0.5 m
3
. If the
barometer reads 760 mm of Hg, what is the amount of work done upon the
atmosphere by the balloon (50.66 kJ)
7. When the valve of the evacuated bottle is opened, atmosphere air rushes
into it. If the atmosphere pressure is 101.325 KPa, and 1.2 m
3
of air (measured at
atmosphere conditions) enters the bottle, calculate the work done by the air (-60.8 kJ).
16
8. A gas system, confined by a piston and cylinder, undergoes a change of
state such that the product of pressure and volume remains constant. If the process
begins at a pressure of 3 bar and a volume 0.015m
3
and proceeds until the pressure
falls to half its initial value, determine the magnitude and direction of the work flow.
Solution:
pV = C i.e., hyperbolic process or p
1
V
1
= p
2
V
2
Given, p
1
= 3 x 10
5
Pa V
1
= 0.015 m
3
p
2
= 1.5 x 10
5
m
3
V
2
= ? W
1-2
= ?
We have, p
1
V
1
= p
2
V
2
V
2
=
2
1 1
p
V p
=
5
5
10 5 . 1
015 . 0 10 3
x
x x
= 0.03 m
3

Displacement work, (W
d
)
1-2
=
2
1

p.dV = p
1
V
1
ln
1
2
V
V

= 3119.16 J = 3.1192 kJ
Positive sign indicates work is done by the system on the surroundings.
9. A certain amount of gas is compressed from 1 bar and 0.1m
3
to 5 bar and
0.03m
3
. The process is according to the law pV
n
= K. Determine the magnitude and
direction of work.
Solution: Given: p
1
= 1 bar; V
1
= 0.1 m
3
; p
2
= 5 bar; V
2
= 0.03
We have for a polytropic process,
Displacement work, = (W
d
)
1-2
=
1
2 2 1 1

n
V p V p
To find the compression index n, we have,
C V p V p
n n

2 2 1 1
i.e.,
n
V
V
p
p

,
_

1
2
2
1
Taking logs on both sides we have
1
2
2
1
ln . ln
V
V
n
p
p

34 . 1
1 . 0
03 . 0
ln
5
1
ln
n

(W
d
)
1-2
=
( ) ( )
1 34 . 1
03 . 0 10 5 1 . 0 10 1
5 5

x x
= - 14706 J = - 14.706 kJ
17
p
i.e., work done on the gas = 14.706 kJ
10. A gas confined in a cylinder by a piston is at pressure of 3 bar and a
volume of 0.015 m
3
. The final pressure is 1.5 bar. Determine the magnitude and
direction of work transfer for the following processes. i) p V, ii) p
V
1
, iii) p V
2
and iv) p
2
1
V
Solution: Given: p
1
= 3 x 10
5
Pa; V
1
= 0.015 m
3
; p
2
= 1.5 x 10
5
Pa V
2
= ? (W
d
) = ?
i) p V i.e.,
2
2
1
1
V
p
C
V
p
V
p


We have, (W
d
)
1-2
=
2
1

p.dV
=
2
1

1
1
1
1
.
V
p
dV
V
V p

2
1

V.dV
=
[ ]
2
1
2
2
1
1
2
V V
V
p

we have
2
2
1
1
V
p
V
p

V
2
= p
2

( ) 015 . 0
3
5 . 1
1
1

p
V
= 0.0075 m
3
(W
d
)
1-2
=
( ) 015 . 0 2
10 3
5
x
[0.0075
2
0.015]
2
= - 1.688 kJ
-ve sign indicates that work is done on the system
ii) p
V
1
i.e., pV = C
Ans: (W
d
)
1-2
= 3.1192 kJ
iii) p V
2
i.e.,
2 2
2
2
2
1
1
V
p
C
V
p
V
p

V
2
=
1
2
1 2
p
V p
= 0.0106 m
3
(W
d
)
1-2
=
2
1

p.dV
=
2
1


dV V
V
p
.
2
2
1
1

=
dV V
V
p
.
2 2
1
2
1
1

[ ]
3
1
3
2 2
1
1
3
V V
V
p

=
( )
[ ]
3 3
2
5
015 . 0 0106 . 0
015 . 0 3
10 3

x
= -0.9707 kJ
-ve sign indicates that work is done on the system
18
iv) p
2
1
V
i.e., pV
2
= C = p
1
V
1
2
= p
2
V
2
2

V
2
=
5 . 1
015 . 0 3
2
2
2
1 1
x
p
V p

= 0.0212 m
3

We have (W
d
)
1-2
=
2
1

p.dV =
2
1

dV
V
V p
.
2
2
1 1
= p
1
V
1
2

2
1

dV
V
.
1
2
= ( ) [ ]
2
1
1 2
1 1
1 2

+ V V p
=
2
1
2
1 1
1
1
]
1

V
V p
=

,
_

+
1 2
2
1 1
1 1
V V
V p
=

,
_

2 1
2
1 1
1 1
V V
V p
Substituting the given values, we get (W
d
)
1-2
= 1.316 kJ
+ve sign indicates that work is done by the system
11. A non-flow reversible process occurs for which p = 3V
2
+ 1/V where p is
in N/cm
2
and V is in m
3
. What is the work done when V changes from 0.5 m
2
to 1 m
3
.
Solution: We have (W
d
)
1-2
=
2
1

p.dV
= 10
4

2
1

dV
V
V
,
_

+
1
3
2
=
2
1

3 V
2
dV +
2
1

dV/V
= 10
4

[ ]

'

+
1
2 3
1
3
2
ln
3
3
V
V
V V
= 10
4

[ ]

'

+
5 . 0
1
ln 5 . 0 1
3 2
= 1.568 x 10
4
J = 15.86 kJ
12. A system consists of a cylinder and piston machine. The external normal
load applied to the piston is given by F = - 7000 + 15000L Newtons, where L is the
distance in mts from the closed and of the cylinder to the piston. How much work is
done when the piston moves from the position L = 1m to L = 1.5 m. Sketch the p-V
diagram for this process and show the work done.
Solution: Given: F = - 7000 + 15000 L
We have Work done = (W
d
)
1-2
= pdV
=
2
1

p.A.dL
19
= A
2
1

p.dL
But p =
A
L
A
F 15000 7000 +

(W
d
)
1-2
= A
2
1


,
_

+
A
L 15000 7000
. DL
=
2
1
L
L

(-7000 + 15000L). dL
= - 7000 (L
2
L
1
) + 15000/2 (L
2
2
L
1
2
)
= - 7000 (0.5) + 15000/2 (1.25)
= - 3500 + 9375
= 5875 J = 5.875 kJ
Sketching of p-V diagram:
F 8000 9500 11000 12500 14000 15500
L 1 1.1 1.2 1.3 1.4 1.5
0
5000
10000
15000
20000
1 1.2 1.4
V= A x L
p
=

F
/
A
13. An insulated system contains a mixture of ice and water. A paddle wheel
is rotated in the system at 100rpm. Torque applied to the shaft is 3 N-m. In order to
effect the transformation of 1 kg of ice to liquid water 300 kJ of heat must be
transferred to the system. Determine the length of time the paddle wheel must be
rotated in order to achieve 2.5 kg reduction in the quantity of ice.
Solution: Given: T = 3 N-m ; N = 1000 rpm
Paddle work (done on the system), W
s
=
60
2 NT
=
60
3 ) 1000 ( 2
= 314 N-m/s = 314 J/s
In order to have a reduction of 2.5 kg of ice into water, the paddle work required is
300 (2.5) = 750 kJ
Length of time for which the paddle wheel should be operated, is
314
10 750
3
x
= 2388.5 sec i.e., 39.8 minutes
14. A system containing 5 kg of a substance is stirred with a torque of 1 N-m
at a speed of 500 rpm for 24 hrs. The system mean while expands from 1.5m
3
to
2.0m
3
against a constant pressure of 5 bar. Determine the magnitude and direction of
net work transfer.
20
Solution: The system is associated with two interactions with the surroundings i.e.,
stirring work (surroundings to the system) and displacement work
i.e., W
1-2
= W
st(1-2)
+ (W
d
)
1-2
Stirring work, W
st
= 2 NT / 60
=
( )
60
1 500 2
= 52.359 n-m/sec
W
st
= 52.359 x 24 x 60 x 60
= 4523893 J = 4523.893 kJ
This is a negative work, as the work is done on the system by the surroundings
Displacement work, (W
d
)
1-2
=
2
1

pdV
= p (V
2
V
1
)
= 5 x 10
5
(2 - 1.5)
= 250000 J = 250 kJ
Net work transfer W
1-2
= - 4523.893 + 250
= - 4273.89 kJ
Negative indicated that net work transfer takes place from the surroundings to the system.
15. A mass of 1.5 kg of a substance is compressed in a quasi-static process
from 0.1 MPa to 0.7 MPa. The initial pressure density of the substance is 1.16 kg/m
3
.
Determine the magnitude of work done on the substance if i) process is pV = C and
pV
1-4
= C
Solution: Given: m = 1.5 kg p
1
= 0.1 x 10
6
Pa p
2
0.7 x 10
6
MPa
1
= 1.16
kg/m
3
Since = 1.16 kg/m
3
and m = 1.5 kg, volume V
1
= m/ = 1.293 m
3
Case i) pV = C
i.e., pV = p
1
V
1
= p
2
V
2
or
3
2
1 1
2
185 . 0 293 . 1
7 . 0
1 . 0
m x
p
V p
V

We have (W
d
)
1-2
=
2
1

p.dV
= dV
V
V p
.
1 1 2
1

i.e., (W
d
)
1-2
= p
1
V
1
ln
1
2
V
V
= 0.1 x 10
6
x 1.293 ln
293 . 1
185 . 0

= - 251595 J = - 251.595 kJ
Negative sign indicates that work is done on the system
Case ii) pV
1-4
= C
i.e., p
1
V
1
1-4
= p
2
V
2
1-4
= V
2
=
4 . 1 / 1
2
4 . 1
1 1
1
]
1

p
V p
= 0.322 m
3
21
We have, adiabatic process (W
d
)
1-2
=
1
2 2 1 1

V p V p
=
( )
4 . 0
) 322 . 0 ( 10 7 . 0 293 . 1 10 1 . 0
6 6
x x
= - 240381 J = - 240.381 kJ
Negative sign indicates that work is done on the system
16. O
2
is compressed in a quasi static process according to the relation pV
1-2
=
C. The initial conditions are 98 KPa and 20
0
C and the final pressure is 1000 KPa.
Assuming an ideal gas behaviour, determine the work required to compress 100 kg of
O
2
. Compare this work with the work of isothermal compression, i.e., pV = C.
Solution: p
1
=98 x 10
3
Pa, T
1
= 293
0
K, p
2
= 1000 x 10
3
Pa, m = 100 kg
We have for polytropic process,
(W
d
)
1-2
=
1
2 2 1 1

n
V p V p
1
) (
2 1

n
T T mR

But gas constant,
32
3143 . 8

M
R
R = 0.2598 kJ
Also for polytropic process we have
n n
p
p
T
T
/ 1
1
2
1
2

,
_

T
2
= 293
2 . 1
2 . 0
98
1000

,
_

= 431.52
0
K
(W
d
)
1-2
=
2 . 0
) 52 . 431 293 )( 259 . 0 ( 100
= -17938.34 kJ
Negative sign indicates that work is done on the system
Case ii) for isothermal process, we have
(W
d
)
1-2
= p
1
V
1
ln
1
2
V
V
Considering oxygen to be a perfect gas, we have pV = mRT
(W
d
)
1-2
=
2
1
1
ln
p
p
mRT
since, pV = C = p
1
V
1
= p
2
V
2
i.e., V
2
/V
1
= p
1
/p
2

= -17626.94 kJ
Negative sign indicates that work is done on the system
17. The following data refer to a12 Cylinder, single-acting, two-stroke marine
Diesel engine:
Cylinder diameter-0.8m
Stroke of piston-1.2m
Area of indicator diagram-5.5E10
-4
m
2
Length of diagram-0.06m
Spring value-147 MPa per m
22
Find the net rate of work transfer from the gas to the piston in kW.
Solution: Mean effective pressure, P
m
, is given by
d
d
m
l
a
P
*spring constant
m
MPa m
147
06 . 0
10 5 . 5
2 4

=1.35 MPa
One engine cycle is completed in two strokes of the piston or one revolution of the crank
shaft.
Work done in one minute= P
m
LAN

150 ) 8 . 0 (
4
2 . 1 35 . 1
2


=122 MJ
Since the engine is single-acting, and it has 12 cylinders, each contributing an equal
power, the rate of work transfer from the gas to the piston is given by
W=12212 MJ/min
=24.4 MJ/s
=24.4 MW = 24,400 kW
18. A gas system has mass m, occupies a volume V at a pressure of p and
temperature T. These properties are related by the equation
[ ] mRT b V
V
a
p
1
]
1

+
2

where a, b and R are constants. Obtain an expression for the displacement work done
by this gas system during a constant temperature process where the gas expands from
1 m
3
to 10 m
3
at a temperature of 293 K. Assume a = 15.7 x 10
4
Nm
4
, b = 1.07 x 10
-2
and R = 0.278kJ/kg-K.
Solution: For a given gas,
[ ] mRT b V
V
a
p
1
]
1

+
2
Solving for p we get,
( )
1
]
1

2
V
a
b V
mRT
p

Displacement work = (W
d
)
1-2
= pdv
( )
1
]
1

2
1
2
V
V
dv
V
a
b V
mRT

( )
( )

,
_

'

1 2 1
2
1 1
ln
V V
a
b V
b V
mRT
On substituting the values we get, (W
d
)
1-2
= 1744.8 x 10
3
J = 1744.8 kJ
23
1. a) An insulated rigid vessel contains some powdered coal and air at a pressure of
10 bar and a temperature of 20
0
C. The coal is ignited, there results a rise in the
pressure and temperature of the contents of the vessel, the final temperature is 538
0
C.
Taking the vessel and the contents to be the system under consideration evaluate the
increase in the energy of the system. b) The insulation is now removed. A heat
transfer. of 50 kJ from the system causes the temperature to fall to the initial value,
20
0
C. Evaluate the increase in the energy of the system during this process. c) Taking
the initial energy of the system 32 kJ, write down the energy values after process (a)
and after process (b)
Solution: a) The vessel is rigid i.e., dv = 0 W = 0
System boundary is insulated, Q = 0
By 1
st
law Q = E + W E = 0
(Though there is a change in temperature and pressure of the system, there is no change
in energy. All that happens is chemical energy of the system is converted into internal
energy).
b) W = 0 ; Insulation removed: Q = - 50 kJ
-50 = 0 + E E = -50 kJ or decrease in energy = 50 kJ
c) During process (a) E = 0 Initial energy = energy after process = 32 kJ
For process (b) initial energy = Final energy in process (a) = 32 kJ
E for process (b) = -50 kJ
i.e., E
final
E
initial
= - 50 kJ
E
final
= -50 + 32
= 18 kJ

2. On a warm summer day, a housewife decides to beat the heat by closing the
windows and doors in the kitchen and opening the refrigerator door. At first she feels
cool and refreshed, but after a while the effect begins to wear off. Evaluate the
situation as it relates to first law, considering the room including the refrigerator as
the system.
Solution:
At first the temperature of air in the room falls since it communicates with the cool
refrigerator. This makes the housewife feel cool.
Considering the room and its contents as a system, and assuming walls, windows and
doors non-conducting, Q = 0. For the operation of refrigerator, electricity is supplied
from outside and hence electrical work W
e
= is done on the system.
From first law of TD Q = E + W
e
0 = E - W
e
E = W
e
24
.
.
+
-
W
e
Room
Ref
System
Positive sign of energy indicates the increase in energy of the system with time. As the
energy is increasing, the temperature of air increases and hence effect of coolness
gradually begins to wear off.

3. The average heat transfer from a person to the surroundings when he is not
actively working is about 950 kJ/hr. suppose that in the auditorium containing 1000
people the ventilation system fails. a) How much does the internal energy of air in the
auditorium increase during the first 15 minutes after the ventilation fails? b)
Considering the auditorium and all the people as system and assuming no heat
transfer. to surroundings, how much does the int. energy of the system change? How
do you account for the fact that the temperature of air increases?
Solution: a) Average heat transfer per person = 960 kJ/hr
= 960 / 60 = 15.83 kJ /min
Average heat transfer / person for 15 min = 237.5 kJ
Average heat transfer for 15 min in the auditorium containing 1000 people
Q = 237.5 x 1000 = 237500 kJ/min
From first law of TD, we have Q = E + W
237500 = E + 0
E = 237.5 MJ
b) Considering the auditorium and all the people as system,
Q = 0; W = 0
Q = E + W
0 = E + 0 E = 0
Increase in internal energy of the air due to increase in its temperature is compensated by
the decrease in internal energy of the people.
4. A household refrigerator is loaded with fresh food and closed. Consider the whole
refrigerator and the contents as a system. The machine uses 1 kWhr of electrical
energy in cooling the food and the internal energy of the food (system) decreases by
5250 kJ, as the temperature drops. Find the magnitude and direction of heat transfer
during the process.
Solution:
Given: Electrical work, = 1 kWhr
W
e
= 860 Kcal
= 860 (4.187) = - 3600.8 kJ
Given, E = - 5250 kJ
From first law of TD Q = E + W
e
= - 5250 3600.8
= - 8850 kJ
25
Ref
+
Conte-
nts
Power
Supply
Negative sign indicates heat flows from the refrigerator to the surroundings
5. A closed system undergoes a constant volume process in which 85 kJ of heat is
supplied to it. The system then undergoes a constant pressure process in which 90 kJ
of heat is rejected by the system and 15 kJ of work is done on it. Finally the system is
brought back to its original state by a reversible adiabatic process. Determine i) The
magnitude and direction of work transfer during the adiabatic process. ii) The energy
of the system at all end states if the energy at the initial state is 100 kJ.
Solution:
Process 1-2: Constant volume process i.e., dv = 0 i.e., W
1-2
= 0
Q
1-2
= (E
2
E
1
) + W
1-2
85 = (E
2
E
1
) + 0
(E
2
E
1
) = 85 kJ
But E
1
= 100 kJ E
2
-100 = 85 E
2
= 185 kJ
Process 2-3: (Constant pressure process)
Q
2-3
= (E
3
E
2
) + W
2-3
-90 = (E
3
E
2
) 15 (E
3
E
2
) = -75 kJ
But E
2
= 185 kJ E
3
= -75 + 185 = 110 kJ
In a cyclic process

W Q
Q
1-2
+ Q
2-3
+ Q
3-1
= W
1-2
+ W
2-3
+ W
3-1
85 90 + 0 = 0 -15 + W
3-1

W
3-1
= 10 kJ
6. A system undergoes a constant pressure process which is followed by a constant
volume process. during the constant pressure process, 125 kJ of heat is transferred to
the system and 50 kJ of work is done by the system. during a constant volume
process, 125 kJ of heat is rejected from the system. find the work interaction if a rev.
adiabatic process restores the system to the initial state.
Solution: Process 1-2: Constant pressure process
We have Q
12
= 125 W
12
= + 50
Q
12
= E
2
E
1
+ W
12

125 = (E
2
E
1
) + 50 (E
2
E
1
) = 75 kJ
Process 2-3: Constant volume process, i.e., dv = 0 W
23
= 0
We have Q
23
= -125
Q
2-3
= (E
3
E
2
) + W
2-3
-125 = (E
3
E
2
) + 0 (E
3
E
2
) = - 125 kJ
26
1
2
3
p
V
Q
3-1
= 0
For a cyclic process

W Q
125 125 + 0 = 50 + 0 + W
3-1
W
3-1
= - 50 kJ
or Process 3-1: (Rev. Adiabatic process) dQ = 0
Q
3-1
= E
1
E
3
+ W
3-1
0 = (E
1
E
2
) + (E
2
E
3
) + W
3-1
= -75 + 125 + W
3-1
W
3-1
= -50 kJ
7. A system executes a cyclic process which includes four processes: 1-2, 2-3, 3-4
and 4-1. The magnitudes of the energy transfer are shown in the following table.
Process heat transfer
Q (kJ)
Work transfer
W(N-m)
Change in internal
energy U (kJ)
1-2 10 0 10
2-3 -25 15 x 10
3
-40 kJ
3-4 60 23.5 kJ 36.5 kJ
4-1 -15 -8.5 x 10
3
-6.5 kJ
Find the magnitude of the unknown quantities in kJ (Hint: For the process 3-4, use

0 dU

8. A system composed of a stone having a mass of 10 kg and a bucket containing
100 kg of water are at the same temperature the stone being at a height of 10.2 m
above the water level. Stone falls into the water. Determine U, kE, PE, Q and W
for the following cases. a) The stone is above to enter the water. b) The stone has just
come to rest in the bucket and c) Heat has been transferred to the surroundings in
such an amount that the stone and water are at the same temperature they were
initially.
Solution: The 1
st
law of THERMODYNAMICS. is
Q
1-2
= U
2
U
1
+
2
m
(v
2
2
v
1
2
) + mg (h
2
h
1
) + W
1-2
--- (1)
a) The stone is about to enter the water
Assuming no heat transfer. to or from the stone a sit falls,
Q
1-2
= 0 W
1-2
= 0 U = 0
Equation: 1) reduces to 0 = kE + pE or - kE = pE
= mg (h
2
h
1
)
= 10 (9.81) (-10.2)
= - 1000 J = - 1kJ
i.e., kE = 1 kJ & pE = - 1kJ
b) Just after the stone comes to rest in the bucket,
Q
1-2
= 0 ; W
1-2
= 0 kE = 0
Equation 1) reduce to pE = -U = mg (h
2
h
1
) = - 1kJ
U = 1kJ & pE = - 1kJ
c) After heat has been transferred so that stone & water are at the same temperature
they were initially, U = 0 In this case
27
U = 0 kE= 0 W
1-2
= 0
Equation 1) reduces to Q
1-2
= PE = mg (h
2
h
1
) = -1kJ
The Pure Substance
The system encountered in thermodynamics is often quite less complex and consists of
fluids that don not change chemically, or exhibit significant electrical, magnetic or
capillary effects. These relatively simple systems are given the generic name the Pure
Substance.
Definition
A system is set to be a pure substance if it is (i) homogeneous in chemical composition,
(ii) homogeneous in chemical aggregation and (iii) invariable in chemical aggregation.
Homogeneous in chemical composition means that the composition of each part of the
system is same as the composition of any other part. Homogeneous in chemical
aggregation implies that the chemical elements must be chemically combined in the same
way in all parts of the system. Invariable in chemical aggregation means that the
chemical aggregation should not vary with respect to time.
(i) (ii) (iii)
Satisfies condition (i) Satisfies condition (i) Does not satisfies condition
(i)
Satisfies condition (ii) Does not satisfies condition (ii)
Satisfies condition (iii)
Figure Illustration of the definition of pure substance
In figure three systems are shown. The system (i) shown in the figure is a mixture of
steam and water. It is homogeneous in chemical composition because in every part of the
system we have, for every atom of oxygen we have two atoms of hydrogen, whether the
sample is taken from steam or water. The same is through for system (ii) consisting of
water and uncombined mixture of hydrogen and oxygen. System (iii) however is not
homogeneous in chemical composition because in the upper part of the system hydrogen
and oxygen are present in the ratio 1:1 where as in the bottom portion they are present in
the ratio 2:1.
System (i) also satisfies condition (ii), because both hydrogen and oxygen have combined
chemically in every part of the system. System (ii) on the other hand does not satisfies
condition (ii) because the bottom part of the system has two elements namely hydrogen
28
Steam
Water
H2 + O2
(Gas)
Water
H2 + O2
(Gas)
Water
and oxygen have chemically combined where as in the upper part of the system the (ii)
elements appear as a mixture of two individual gases.
Invariable in chemical aggregation means that the state of chemical combination of the
system should not change with time. Thus the mixture of hydrogen and oxygen, if it is
changing into steam during the time the system was under consideration, then the systems
chemical aggregation is varying with time and hence this system is not a pure substance.
Thus the system (i) is a pure substance where as the systems (ii) and (iii) are not pure
substances.
The Two Property Rule for a Pure Substance
The thermodynamics state of a pure substance of a given mass can be fixed by specifying
two independent properties provided (i) the substance is in equilibrium and (ii) the
effects of gravity, motion, capillarity, electricity and magnetism are negligible.
The above rule indicates that if the values of two properties of a pure substance are fixed
then the values for all other properties are fixed. This means that there is a definite
relation between the two independent properties and each of the other properties. Each of
these relations is called Equation of state for a pure substance. The equation of state for
a pure substance can be in any one of the following forms: (i) Algebraic equation
(example: perfect gas equation), (ii) Tables (example: steam tables) and (iii) Charts
(example: Mollier chart for steam).

Specific heat, C
When interaction of heat takes place between a closed system and its
surroundings, the internal energy of the system changes. If Q is the amount of heat
transferred to raise the temperature of 1 kg of substance by dT, then, specific heat C =
Q/dT
As we know, the specific heat of gas depends not only on the temperature but also upon
the type of the heating process. i.e., specific heat of a gas depends on whether the gas is
heated under constant volume or under constant pressure process.
We have dQ = m C
V
. dT for a rev. non-flow process at constant volume
and dQ = m C
p
. dT for a rev. non-flow process at constant pressure
For a perfect gas, C
p
& C
V
are constant for any one gas at all pressure and temperatures.
Hence, integrating above equations.
Flow of heat in a rev. constant pressure process = m C
p
(T
2
T
1
)
Flow of heat in a rev. constant volume process = m C
V
(T
2
T
1
)
The internal energy of a perfect gas is a function of temperature only. i.e, u = f (T), to
evaluate this function, let 1 kg of gas be heated at constant volume
From non-flow energy equation, Q = dU + W
W = 0 since volume remains constant
Q = dU = C
V
. dT
Int. U = C
V
T + k where k is a constant
29
For mass m, Int. energy = m C
V
T
Any process between state 1 to state 2,
Change in int. energy = m C
V
(T
2
T
1
)
(U
2
U
1
) = m C
V
(T
2
T
1
)
We can also find the relationship between C
p
& C
V
& shown that
C
p
C
v
= R ;
;
V
p
C
C

1

r
R
C
v
&
( ) 1

r
rR
C
P
Enthalpy: Consider a system undergoing a quasi equilibrium constant pressure process.
We have from 1
st
law of thermodynamics for a non-flow process,
Q
1-2
= U
2
U
1
+ W
1-2
W
1-2
=
2
1

pdv
Since pressure is constant W
1-2
= p (V
2
V
1
)
Q
1-2
= U
2
U
1
+ p (V
2
V
1
)
= (U
2
+ p
2
V
2
) (U
1
+ p
1
V
1
)
i.e., heat transfer during the process is given in terms of the change in the quantity (U +
pV) between initial and final states. Therefore, it find more convenient in
thermodynamics to define this sum as a property called Enthalpy (H)
i.e., H = U + pV
In a constant pressure quasi equilibrium process, the heat transfer is equal to the
change in enthalpy which includes both the change in internal energy and the work for
this particular process.
The enthalpy of a fluid is the property of the fluid, since it consists of the sum of a
property and the product of the two properties. Since enthalpy is a property, like internal
energy, pressure, specific volume and temperature, it can be introduced into any problem
whether the process is a flow or a non-flow process.
For a perfect gas, we have h = u + pV
= C
V
T + RT
= (C
V
+ R) T
= C
p
T
i.e., h = C
p
T & H = mC
p
T
For any process, Q = dH
= mC
p
dT
For a process between states 1 & 2
Change in enthalpy = (H
2
H
1
) = mC
p
(T
2
T
1
)
Specific heat at Constant Volume:
When heat interaction takes place at constant volume, W = 0 and from 1
st
law of
thermodynamics, for unit mass, (q)
V
= dU
The amount of heat supplied or removed per degree change in temperature, when the
system is kept under constant volume, is called as the specific heat at constant volume,
30
Or C
V
=
V
dT
Q
1
]
1


V
dT
dU
1
]
1

Or dU = C
V
dT
Specific heat at Constant pressure
When heat interaction is at constant pressure, (q)
p
= dh
The amount of heat added or removed per degree change in temperature, when the
system is kept under constant pressure, is called as the specific heat at constant pressure.
Or C
p
=
p
dT
Q
1
]
1


p
dT
dh
1
]
1


Or dh = C
p
. dT
Application of 1
st
law of thermodynamics to non-flow or closed system:
a) Constant volume process (V = constant)
Applying 1
st
law of thermodynamics to the process,
Q
1-2
= U
2
U
1
+ W
1-2
= U
2
U
1
+ 0
i.e., Q
1-2
= C
V
(T
2
T
1
)
For mass m of a substance, Q = mC
V
(T
2
T
1
)
b) Constant pressure (p = Constant)
Applying 1
st
law of thermodynamics to the process,
Q
1-2
= u
2
u
1
+ W
1-2
The work done, W
1-2
=
2
1

p dV = p (V
2
V
1
)
i.e., Q
1-2
= u
2
u
1
+ p (V
2
V
1
) = (u
2
+ pV
2
) (u
1
+ pV
1
)
= h
2
h
1
i.e., Q = C
p
(T
2
T
1
)
For mass m of a substance, Q = mC
p
(T
2
T
1
)
c) Constant temperature process (Isothermal process, T = constant)
Applying 1
st
law of thermodynamics to the process,
Q
1-2
= U
2
U
1
+ W
1-2
= C
V
(T
2
T
2
) + W
1-2
i.e., Q
1-2
= W
1-2

2 1
T T
Q
1-2
= p
1
V
1
lnV
2
/V
1

= p
1
V
1
ln p
1
/p
2
d) Reversible adiabatic process (pV

) = constant
Applying 1st law of thermodynamics to the process,
Q
1-2
= U
2
U
1
+ W
1-2
O = u
2
U
1
+ W
1-2
--- (1)
31
Or (U
1
U
2
) =
1
2 2 1 1

V p V p
(U
1
U
2
) =
( )
1
2 1

T T R
The above equation is true for an adiabatic process whether the process is reversible
or not. In an adiabatic experiment, the work done W
1-2
by the fluid is at the expense
of a reduction in the internal energy of the fluid. Similarly in an adiabatic
composition process, all the work done on the fluid goes to increase the internal
energy of the fluid.
To derive pV

= C: For a reversible adiabatic process


We have q = du + u
For a reversible process, w = p dV
q = du+ p dV
= O For an adiabatic process q = 0
Also for a perfect gas, pV

= RT or p =

V
RT
dU + RT
V
dV
Also, u = C
V
T or du = C
V
dT
C
V
dT + RT
V
dV
or C
V

0 +
V
dV
R
T
dT

Int., C
V
ln T + R ln V = constant
Sub. T = pV/R
C
v
ln

v R
R
Pv
ln +
= constant
Or ln
V
C
R
R
pV
V
ln +
= constant
Also, C
V
=
1
1

V
C
R
or
R
ln
V
R
pV
ln ) 1 ( +
= constant
ln
1
ln

+

V
R
pV
= constant
or ln
R
pVxV
1
= constant
i.e., ln
R
pV
r
= constant
or
R
pV
r
= e
constant
= constant
i.e., pV

= constant
32
we have pV = RT
or p =
V
RT
sub. This value of p in pV

= C
V
RT
V

= C or TV

-1
= constant --- (a)
Also, V =
P
RT
sub. This in equation pressure

= C
p

,
_

p
RT
= constant

1
2

p
T
= constant or

,
_

1
p
T
= constant --- (b)
For a reversible adiabatic process for a perfect gas between states 1 & 2, we can
write
p
1
V
1

= p
2
V
2

or
r
V
V
p
p

,
_

2
1
1
2
--- (c)
T
1
V
1

-1
= T
2
V
2

-1
or
1
2
1
1
2

,
_

V
V
T
T
--- (d)
r
r
r
r
p
T
p
T
1
2
2
1
1
1


or
r
r
p
p
T
T
1
1
2
1
2

,
_

--- (e)
The work done in an adiabatic process is W = u
1
u
2
The gain in I.E. of a perfect gas, is u
2
u
1
= C
V
(T
2
T
1
)
W = C
V
(T
1
T
2
)
But C
V
=
1
R
W =
1
) (
2 1

T T R
Using pV = RT, W =
1
2 2 1 1

V p V p
e) Poly tropic process (pV
n
= constant)
Applying 1
st
law of thermodynamics, Q
1-2
= u
2
u
1
+ W
1-2
= (u
2
u
1
) +
( )
1
2 1

n
T T R
i.e., Q =
( )
1
2 1

n
T T R
- C
V
(T
1
T
2
)
33
Also C
V
=
1
R
sub. & simplifying Q =
W
n
n

,
_

In a poly tropic process, the index n depends on the heat and work quantities during
the process.
9. A cylinder contains 0.45 m
3
of a gas at 1 bar & 80
0
C. The gas is compressed to a
volume of 0.13 m
3
, the final pressure being 5 bar. Determine i) the mass of the gas, ii)
the value of index n for composition, iii) the increase in internal energy of the gas
and iv) the heat received or rejected by the gas during compression. (Take = 1.4, R
= 294.2 J/kg-K).
Solution: V
1
= 0.45 m
3
p
1
= 1 x 10
5
Pa V
2
= 0.13 m
3
T
1
= 353 K
p
2
= 5 x 10
5
Pa
i) We have p
1
V
1
= mRT
1
m =
353 2 . 294
45 . 0 10 1
5
x
x x
= 0.433 kg
ii) p
1
V
1
n
= p
2
V
2
n
i.e.,
1
2
2
1
p
p
V
V
n

,
_


Or
,
_


,
_

1
5
13 . 0
45 . 0
n
n = 1.296
iii)
296 . 0
1
2
1
1
2
13 . 0
45 . 0

,
_

,
_

n
V
V
T
T
T
2
= 509.7 K
Increase in int. energy, U = mC
v
(T
2
T
1
)
= 0.433 x
1 r
R
(T
2
T
1
)
= 0.433 x
( )
( ) 353 7 . 509
1 4 . 1
2 . 294

= 49.9 kJ
iv) We have Q = U + W
W =
1
2 2 1 1

n
V p V p
=
( )
1
2 1

n
T T mR
=
( )( )
296 . 0
7 . 509 353 27 . 294 433 . 0
= - 67.44 kJ
Q = 49.9 67.44 = - 17.54 kJ
Heat rejected = - 17.54 kJ
10. The properties of a certain fluid are related as follows
U = 196 + 0.718 t
pv = 0.287 (t + 273) where u is the sp. Internal energy (kJ/kg), t is in
0
C,
p is pressure (kN/m
2
) and v is sp. Volume (m
3
/kg).
For this fluid, find C
v
& C
p
Solution: By definition sp. Heat at constant volume C
v
=
dt
du
dt
du
v

,
_


C
V
=
dt
d
(196 + 0.718 t)
= 0.718 kJ/kg
0
C
34
Also, C
p
=
( ) pv u
dt
d
dt
dh
p
+
,
_

=
( ) pv
dt
d
dt
du
+
=
( ) t
dt
d
718 . 0 196 + ( ) 273 287 . 0 287 . 0 x t
dt
d
+ +
( ) t t
dt
d
287 . 0 718 . 0 +
= 1.005 kJ/kg
0
C
11. A fluid system consisting of 4.17 kg of a pure substance has an energy E of 85 kJ.
The kinetic energy of the system is 17 kJ and its gravitational potential energy is 5 kJ.
The system undergoes an adiabatic process in which the final sp. i.e., is 150 kJ/kg, the
final kinetic energy is 1.9 kJ and the final gravitational potential energy is 1.1 kJ. The
effects due to electricity, capillary and magnetism are assumed to be absent. a)
Evaluate the initial value of the sp. i.e., of the fluid. b) Determine the magnitude and
sign of the work done during the process.
Solution: Total initial energy E
1
=KE
1
+ PE
1
+U
1
85 = 17 + 5 + U
1
U
1
= 63 kJ
Initial sp. i.e., =
kg kJ / 108 . 15
17 . 4
63

Final state: E
2
= k
2
+ P
2
+ U
2
= 1.9 + 1.1 + 4.17 (150) = 628.5 kJ
From 1
st
law, Q = E
2
E
1
+ W
0 = 628.5 63 + W = - 565.5 kJ

12. A mass of 0.2 kg of a pure substance at a pressure of 1 bar and a temperature of
313 k occupies a volume of 0.15 m
3
. Given that the int. energy of the substance is
31.5 kJ, evaluate the sp. Enthalpy of the substance.
Solution: m = 0.2 kg P = 1 x 10
5
N/m
2
T = 313 k v = 0.15 v = 31.5 kJ
We have, enthalpy = U + Pv
= 31.5 x 10
3
+ 1 x 10
5
x 0.15
= 46.5 kJ
sp. Enthalpy = 46.5/0.2 = 232.5 kJ/kg
13. A gas enters a system at an initial pressure of 0.45 MPa and flow rate of 0.25 m
3
/s
and leaves at a pressure of 0.9 MPa and 0.09 m
3
/s. During its passage through the
system the increase in i.e., is 20 kJ/s. Find the change of enthalpy of the medium.
Solution: p
1
= 0.45 x 10
6
Pa V
1
= 0.25 m
3
/s
p
2
= 0.9 x 10
6
Pa V
2
= 0.09 m
3
/s
(u
2
u
1
) = 20 x 10
3
J/s
We have from 1
st
law for a constant pressure quasi static process
Q
1-2
= (u
2
+ p
2
V
2
) (u
1
+ p
1
V
1
)
= (H
2
H
1
)
= Change in enthalpy
= (u
2
u
1
) + p
2
V
2
p
1
V
1
35
= 20 x 10
3
+ 0.9 x 10
6
x 0.09 0.45 x 10
6
x 0.25
(H
2
H
1
) = - 11.5 kJ/s
There is a decrease in enthalpy during the process
14. A closed system of constant volume experiences a temperature rise of 20
0
C when
a certain process occurs. The heat transferred in the process is 18 kJ. The specific heat
at constant volume for the pure substance comprising the system is 1.2 kJ/kg
0
C, and
the system contains 2 kg of this substance. Determine the change in the internal
energy and the work done.
Solution: T = 20
0
C Q = + 18 kJ C
v
= 1.2 kJ/kg
0
C m = 2kg U= ?
W = ?
Change in int. energy, v = mC
v
T
= 2 (1.2) (20) = 48 kJ
From 1
st
law of thermodynamics Q = U + W
+ 18 = 48 + W
W = - 30 kJ
15. The stationary mass of gas is compressed without friction from an initial state of 2
m
3
and 2 x 10
5
N/m
2
to a final state of 1 m
3
and 2 x 10
5
N/m
2
, the pressure remaining
the same. There is a transfer of 360 kJ, of heat from the gas during the process. How
much does the internal energy of the gas change?
Solution: p
1
= p
2
= 2 bar V
1
= 2m
3
V
2
= 1m
3
Q = -360 kJ U = ?
W = pdV = 2 x 10
5
(1-2) = - 2 x 10
5
J
From 1
st
law of thermodynamics, Q = U + W
- 360000 = U 2 x 10
5
U = - 160 kJ
16. The internal energy of a certain substance is given by the following equation
u = 3.56 pv + 84 where u is given in kJ/kg, p is in KPa and v in m
3
/kg. A system
composed of 3 kg of this substance expands from an initial pressure of 500 KPa and a
volume of 0.22 m
3
to a final pressure of 100 KPa in a process pv
1.2
= constant. i) If the
expansion is quasi-static, find Q, U, and W for the process. ii) In another process
the same system expands according to the same pressure volume relationship as in
part (i) and from the same initial state to the same final state as in part (i) but the heat
transfer in this case is 30 kJ. Find the work transfer for this process. iii) Explain the
difference in work transfer in parts (i) and (ii).
Solution: internal energy equation is, u = pv + 84, V
1
= 0.22 m
3
p
1
= 500 kPa p
2
= 100
kPa, Process is pv
1.2
= C
i) u = 3.56 pv + 84
u = u
2
u
1
= 3.56 (p
2
v
2
p
1
v
1
) per kg
U = U
2
U
1
= 3.56 (p
2
V
2
p
1
V
1
) for 3 kg
We have p
1
V
1
1.2
= p
2
V
2
1.2
V
2
= (p
1
/p
2
)
1/1.2
V
1
= 0.8412 m
3
U = 3.56 (100 x 10
3
x 0.8412 500 x 10
3
x 0.22)
= - 92.134 kJ
For a quasi static process, W = pdv =
1
2 2 1 1

n
V p V p
36
2 . 0
22 . 0 10 500 8412 . 0 10 100
3 3
x x x x +

= + 129.4 kJ
From 1
st
law of thermodynamics, Q = U + W
= - 92.134 + 129.4
= 37.27 kJ
ii) Here Q = 30 kJ
Q = U + W
30 = -92.134 + W W = 122.134 kJ
iii) The work in (ii) is not equal to pdv since the process is not quasi-static.
17. A fluid is contained in a cylinder by a spring-loaded, frictionless piston so that the
pressure in the fluid is a linear friction of the volume (p = a + bv). The internal energy
of the fluid is given by the following equation, U = 34 + 3.15 pV where U is in kJ, p
is in kPa and V in m
3
. If the fluid changes from an initial state of 170 kPa, 0.03 m
3
to
a final state of 400 kPa, 0.06 m
3
, with no work other than that on the piston, find the
direction and magnitude of the work and heat transfer.
Solution: Change in internal energy of the fluid during the process, U
U
2
U
1
= 3.15 (p
2
V
2
p
1
V
1
)
= 3.15 (400 x 0.06 170 x 0.03)
= 59.54 kJ
Now p = a + bV
Or 170 = a + b (0.03)
400 = a + b (0.06)
Solve above two equations
230 = b (0.03) b = 7666.67 kN/m
2
a = - 60 kN/m
2
Work transfer involved during the process W
1-2
=
2
1

pdv =
2
1

(a + bV) dV
= a (V
2
V
1
) +
( )
2
.
2
1
2
2
V V b
= - 60 (0.06-0.03) + 7666.67
( )
2
03 . 0 06 . 0
2 2

= 8.55 kJ
From 1
st
law of thermodynamics Q
1-2
= (U
2
U
1
) + W
1-2
= 59.54 + 8.55
= 68.09 kJ
i.e., heat flow into the system during the process.
18. A piston cylinder arrangement has a gas in the cylinder space. During a constant
pressure expansion to a larger volume the work effect for the gas are 1.6 kJ, the heat
added to the gas and cylinder arrangement is 3.2 kJ and the friction between the
piston and cylinder wall amounts to 0.24 kJ. Determine the change in internal energy
of the entire apparatus. (Gas, cylinder, piston).
Solution: W
1-2
= 1.6 kJ Q
1-2
= 3.2 kJ (Q)
f
= - 0.24 U = ?
Q
1-2
= U + W - Q
f
3.2 = U + 1.6 0.24
37
U = 1.84 kJ
19. A system receives 42 kJ of heat white expanding with volume change of 0.123 m
3
against an atmosphere of 12 N/m
2
. A mass of 80 kg in the surroundings is also lifted a
distance of 6 mts. i) Find the change in energy of the system. ii) The system is
returned to its initial volume by an adiabatic process which require 100 kJ of work.
Find the change in energy of the system. iii) Determine the total change in energy of
the system.
Solution: Q = 42 kJ, V = 0.123 m
3
, p = 12 x 10
4
N/m
2
m = 80 kg
d = 6 mt W during adiabatic process = - 100 kJ
i) Q = E + W
Now, W = p V + W
= 1.2 x 10
4
x 0.123 + 80 (9.81) (6)
= 19.469 kJ
E = Q W = 42 19.469 = 22.531 kJ
ii) Q = 0, W = - 100 kJ
Q = E + W
0 = E 100
E = 100 kJ
iii) Total change in energy of the system,
E = Q W ( E)
i
+ ( E)
ii
= 122.531 kJ
= 42 [(-100) + 22.531] = 119.47 kJ

20. A thermally insulated battery is being discharges at atmosphere pressure and
constant volume. During a 1 hr test it is found that a current of 50A and 2v flows
while the temperature increases from 20
0
C to 32.5
0
C. Find the change in internal
energy of the cell during the period of operation.
Solution: Q = 0 W
e
= 50 x 2 x 3600
= 36 x 10
4
J
Q = U + W
e
0 = U + 36 x 10
4
U = - 36 x 10
4
Joules
38
21. When the state of a system changes from state 1 to state 3 along the path 1-2-3 as
shown in figure, 80 kJ of heat flows into the system and the system does 30 kJ of
work. (a) How much heat flows into the system along the path 1-4-3 if work done by
the system is 10 kJ (b) when the state of the system is returned from state 3 to state 1
along the curved path, the work done on the system is 20 kJ. Does the system absorb
or liberate? (c) If U
1
= 0 and U
4
= 40 kJ, find the heat absorbed in the process 1-4 and
4-3 respectively.
Solution:
a) Along the path 1-2-3,
From 1
st
law of T.D., Q
1-3
= U
3
-U
1
+ W
1-3
From the data given, 80 = (U
3
U
1
) + 30
(U
3
U
1
) = 50 kJ
Along the path 1-4-3
Q
1-3
= U
3
U
1
+ W
1-3
From the data given, Q
1-3
= 50 + 10
= 60 kJ U is property of a system
Work done by the system
b) Along the path 3-A-1,
(U
1
U
3
) = Q
3-1
W
3-1
Or Q
3-1
= (U
1
U
3
) + W
3-1
= -50 -20
= -70 kJ
Negative sign indicates that heat is liberated from the system.
c) Along the path 1-4
Q
1-4
= U
4
U
1
+ W
1-4
= 40-0+10
= 50 kJ
Along the path 4-3
Q
4-3
= U
3
U
4
+ W
4-3
= 50 40 + 0
= 10 kJ
22. In a certain steam plant the turbine develops 1000 kW. The heat supplied to the
steam in the boiler is 2800 kJ/kg, the heat rejected by the system from cooling water
in the condenser is 2100 kJ/kg and the feed pump work required to pump the
condensate back into the boiler is 5 kW. Calculate the steam flow round the cycle in
kg/s.
39
p
V
1
2 3
4
A
Solution: W
T
= 1000 kW
Q
b
= 2800 kJ/kg
Q
c
= 2100 kJ/kg
W
P
= - 5 kW


c b
Q Q Q
= 700 kJ/kg

kW W 995 5 1000
Let m be the mass flow,

m Q 700
kW
But

W Q
700 m
= 995

m
= 1.421 kg/s
23. An air compressor takes in air at 10
5
Pa and 27
0
C having volume of 1.5 m
3
/kg and
compressor it to 4.5 x 10
5
Pa. Find the work done, heat transfer and change in internal
energy if the composition is isothermal.
Solution: pv = C, p
1
= 10
5
Pa p
2
= 4.5 x 10
5
Pa, V
1
= 1.5 m
3
/kg
p
1
v
1
=p
2
v
2
v
2
=p
1
/p
2
x v
1
= kg m x
x
/ 3333 . 0 5 . 1
10 5 . 4
10
3
5
5

W = p
1
v
1
ln v
2
/v
1
= - 225.61 kJ
= Q
But, Q = U + W
U = Q W = 0
24. A cylinder fitted with piston contains 0.2 kg of N
2
at 100 kPa and 30
0
C. The
piston is moved compressing N
2
until the pressure becomes 1 MPa and temperature
becomes 150
0
C. The work done during the process is 20 kJ. Determine the heat
transferred from N
2
to the surroundings. Take C
v
= 0.75 kJ/kg-K for N
2
.
Solution: m = 0.2 kg p
1
= 100 x 10
3
Pa T
1
= 303 K p
2
= 1 x10
6
Pa T
2
= 423 K
W = - 20 kJ Q = ?
Change in internal energy during the process = mC
v
(T
2
T
1
)
= 0.2 (0.75 x10
3
) (423 303)
= 18 kJ
From 1
st
law of thermodynamics Q = U + W
= 18 20 = - 2 kJ
25. A closed system consisting of 1 kg of gaseous Co
2
undergoes a rev. process at
constant pressure causing a decrease of 30 kJ in internal energy. Determine the work
during the process. Take C
p
= 840 J/kg -
0
C and C
v
= 600 J/kg -
0
C
Solution: m = 1 kg p = C, U = - 30 kJ W = ?
We have U = mC
v
(T
2
T
1
)
- 30 x 10
3
= 1 x 600 (T
2
T
1
) (T
2
T
1
) = - 50
0
C
i.e., (T
1
T
2
) = 50
0
C
The heat supplied or rejected = Q = mC
p
(T
2
T
1
)
= 1 (840) (-50)
= - 42 kJ
From 1
st
law, Q = U + W
40
-42 = - 30 + W W = - 12 kJ
26. The specific heat at constant pressure of one kg fluid undergoing a non-flow
constant pressure process is given by C
p
=
C kg
kJ
T

1
]
1

+
+
0
20
40
5 . 2
. Where T is in
0
C. The pressure during the process is maintained at 2 bar and volume changes from
1m
3
to 1.8m
3
and temperature changes from 50
0
C to 450
0
C. Determine (i) Heat added
(ii) Work done (iii) Change in i.e., (iv) Change in enthalpy.
Solution: C
p
=
20
40
5 . 2
+
+
T
p
1
= p
2
= 2 x 10
5
Pa V
1
= 1m
3
V
2
= 1.8m
3
T
1
= 50
0
C T
2
= 450
0
C
i) Heat added / kg, Q = dT C
P
T
T
.
2
1

=
dT
T
1
]
1

+
+
20
40
5 . 2
450
50
= [2.5T + 40 /n (T + 20)]
50
450
= 2.5 (450 50) + 40 /n
1
]
1

+
+
20 50
20 450
= 1076.17 kJ
ii) Work done / kg, W = Pdv
= p (V
2
V
1
)
= 2 x 10
5
(1.8 1) = 160 kJ
iii) Change in internal energy, U = Q - W
= 1076.17 160 = 916.17 kJ
iv) Change in enthalpy (for non flow process)
h = Q = 1076.17 kJ
27. Air at 1.02 bar, 22
0
C, initially occupying a cylinder volume of 0.015 m
3
, is
compressed reversibly and adiabatically by a piston to a pressure of 6.8 bar. Calculate
i) the final temperature, ii) the final volume, iii) The work done on the mass of air in
the cylinder.
Solution: p
1
= 1.02 x 10
5
Pa T
1
= 293 k V
1
= 0.015 m
3
Process is pV

= C
p
2
= 6.8 x 10
5
Pa T
2
= ? V
2
= ? W = ?
We have
k T
p
p
T
T
r
r
0
2
1
1
2
1
2
09 . 504

,
_

Also,
1
2
1
1
2

,
_

r
V
V
T
T
or
1
1
1
2
2
1

,
_

r
T
T
V
V
V
2
= 0.003864 m
3
Work done = W =
1
1 2 1 1

V p V p
4 . 0
003864 . 0 10 8 . 6 015 . 0 10 02 . 1
5 5
x x x x

= - 2.74 kJ
i.e., work done on the system = 2.74 kJ
41
28. A storage battery, having a terminal potential of 12 volts draws a current of 8
amps for 2.5 hrs. If the stored energy of the battery decrease by 1250 kJ, evaluate the
heat interaction across the boundary enveloping the storage battery.
Solution: Work done in 2-5 hrs = V x I x T
=
( )
3600
1000
5 . 2 8 12
x
W
e
= 864 kJ ( current flows out of the battery)
Given, dU = - 1250 kJ
From 1
st
law, Q = dU + W
= - 1250 + 866
= - 386 kJ
Negative sign indicates heat rejected from the battery.

29. A 0.75 kW motor drives a paddle wheel which stirs for an hour 50 kg of water
contained in a thermally insulated rigid tank. Assuming the process to be a constant
volume, determine rise in temperature and change in internal energy of water. Take C
for water = 4.186 kJ/kg -
0
K
Solution: Paddle work done in one hour = 0.75 kWh
= 0.75 (3600) = 2700 kJ
This is negative work. The heat interaction, Q = 0
Tank is rigid i.e., Pdv = 0
Q = [ Pdv + Paddle work] + dU
0 = - 2700 + dU
dU = 2700 kJ
= mCdT dT =
) 50 ( 186 . 4
2700
= 12.9
0
C
30. A work piece of 30 cm dia is being turned on a lathe and the tangential force on
the cutting tool is estimated to be 100 N. When the turning operation is executed for
10 minutes, the increase in the int. energy of the work piece works out to be 125 kJ.
Make calculations for the heat transfer from the work piece if the work piece turns at
180 rev./min.
Solution: N = 180 x 10 = 1800, Torque T = F.r= 100 (0.15) = 15 J
Work done, W = 2 NT
=
( )
1000
15 1800 2
= 169.56 kJ
This work is negative as it has been done on the system (workpiece)
From 1
st
law, Q = dU + W
= 125 169.56
= - 44.56 kJ
Negative sign indicates, heat flow from the system
31. A closed rigid vessel containing 10 kg of oxygen at 290 k is supplied heat until its
pressure becomes two-fold that of initial value. Identity the process and calculate the
final temperature, change in internal energy, enthalpy and heat interaction across the
system boundary. Take C
v
= 0.65 kJ/kg -
0
k
42
Solution: R =
32
3142 . 8

M
R
= 259.8 J/kg-
0
K
Process is constant volume, so we can apply Charles law i.e.,
2
2 2
1
1 1
T
V p
T
V p

T
2
= 580
0
k
dU = mC
v
(T
2
T
1
)
= 10 (0.65) (580 290 )
= 1885 kJ
Now, C
P
= C
v
+ R
= 0.65 + 0.2598
= 0.9098 kJ /kg-
0
k
dH = mC
P
(T
2
T
1
)
= 2638.4 kJ
We have Q = dU + W
= 1885 + 0
= 1885 kJ

First Law of Thermodynamics to open system:
In the case of closed system there is only energy transfer across the system boundary. But
in many engineering applications we come across open systems where in both mass and
energy transfer takes place. The energies that cross the system boundary are as follows.
1) Internal energy: Each kg of matter has the internal energy u and as the matter
crosses the system boundary the energy of the system changes by u for every kg
mass of the matter that crosses the system boundary.
2) Kinetic energy: Since the matter that crosses the system boundary will have some
velocity say V each kg of matter carries a k.E. 2 /
2
V thus causing the energy of
the system to change by this amount for every kg of matter entering the system
boundary.
3) Potential energy: P.E. is measured with reference to some base. Thus Z is the
elevation of the matter that is crossing the system boundary, then each kg of matter
will possess a P.E. of gZ.
4) Flow energy or Flow work: This energy is not directly associated with the matter
crossing the system boundary. But it is associated with the fact that there must be
some pumping process which is responsible for the movement of the matter across
the system boundary. Thus external to the system there must be some force which
forces the matter across the system boundary and the energy associated with this is
called flow energy.
43
Flow Work: Consider a flow process in which a fluid of mass dm
1
is pushed into the
system at section 1 and a mass dm
2
is forced out of the system at section 2 as shown in
fig.
In order to force the fluid to flow across the boundary of the system against a pressure p
1
,
work is done on the boundary of the system. The amount of work done is W = - F
1
.dl
1
,
Where F
1
is the force and dl
1
is the infinitesimal displacement, but F
1
= p
1
A
1
W = - p
1
A
1
dl
1
= - p
1
dv
1
i.e., the flow work at section 1 = - p
1
v
1
Similarly, the work done by the system to force the fluid out of the system at section 2 =
+ p
2
v
2
Hence net flow work = p
2
V
2
p
1
V
1
For unit mass, the flow work is (p
2
V
2
p
1
V
1
). Flow work is expressed entirely in terms
properties of the system. The net flow work depends out on the end state of the fluid and
it is a thermodynamics property. Also the fluid contains energies like internal energy,
potential energy and due to the motion of the fluid, kinetic energy, in addition to the flow
work. When a fluid enters an open system, these properties will be carried into the
system. Similarly when the fluid leaves the system, it carries these energies out of the
system. Thus in an open system, there is a change in energy of the system.
5. Control Volume: The first and most important step in the analysis of an open system
is to imagine a certain region enclosing the system. This region having imaginary
boundary is called control volume, which can be defined as follows.
A C.V. is any volume of fixed shape, and of fixed position and orientation relative to the
observer. Across the boundaries of the C.V. apart from mass flow, energy transfer in the
form of heat and work can take place just as similar to the energy transfer across the
boundaries of a system.
Thus the difference between C.V. and system are
i) The system boundary may and usually does change shape, position,
orientation relative to the observer. The C.V. does not by definition.
ii) Matter may and usually does flow across the system boundary of the C.V. No
such flow takes place across the system boundary by definition.
44
p
1
p
2
,
A
1
F
1
dm
1
dm
2
F
2
1
1
2
2
dl
1
dl
2
First law of thermodynamics for an open system (Flow process):
We have 1
st
law of thermodynamics to a closed system as,
Q W = dU + d(KE) + d (PE)
= d [E]
0
The subscript O refers to the states of the system within the boundary. In the case of open
system, energy is transferred into & out of the system not only by heat and work but also
by the fluid that enters into and leaves the boundary of the system in the form of internal
energy, gravitational potential energy, kinetic energy in addition to the energy in the flow
work. Thus, when the first law is applied to an open system, the energy entering into the
system must be equal to the energy leaving the system in addition to any accumulation of
energy within the system.
1
]
1

+ + +
1
2
1
1 1 1 1
2
gZ
V
u v p dm
1
]
1

+ + +
2
2
2
2 2 2 2
2
gZ
V
u v p dm
The flow process is shown in fig. This analysis can be expressed mathematically as,
1
]
1

+ + + +
1
2
1
1 1 1 1
2
gZ
V
u v p dm W Q
[ ]
0 2
2
2
2 2 2 2
2
E d gZ
V
u v p dm +
1
]
1

+ + +
--- (1)
Where state (1) is the entering condition and state (2) is the leaving condition of the fluid.
This is a general equation of the first law of thermodynamics applied to open system.
Note: The equation is valid to both open and closed system. For closed system, dm
1
=0 &
dm
2
=0
Energy Equation for open system: The general form of first law of thermodynamics
applied to an open system is called steady-flow energy equation (SFEE) i.e., the rate at
which the fluid flows through the C.V. is constant or steady flow. SFEE is developed on
the basis of the following assumptions.
i) The mass flow rate through the C.V. is constant, i.e., mass entering the C.V. /
unit time = mass leaving the C.V. /unit time. This implies that mass within the
C.V. does not change.
45
Q
W
d[E]
0
ii) The state and energy of a fluid at the entrance and exit do not vary with time,
i.e., there is no change in energy within the C.V.
iii) The rates of heat and work transfer into or out of the C.V. do not vary with
time.
For a steady flow process,
2 1
m m m
& d(E)
0
= 0 as Q f (T) & W f (T)
SFEE on the basis unit mass:
Energy entering to the system = energy leaving the system
i.e.,
Q gZ
V
v p u +
1
]
1

+ + +
1
2
1
1 1 1
2
1
]
1

+ + + +
2
2
2
2 2 2
2
gZ
V
v p u W
or
1
]
1

+ + +
1
]
1

+ +
2
2
2
2 1
2
1
1
2 2
gZ
V
h W Q gZ
V
h
or
( )
1
]
1

+ +
2
2
2
gZ
V
h W Q
Where Q = heat transfer across the C.V, W = shaft work across the C.V, h = Enthalpy,
V = velocity, Z = elevation and g = gravitational acceleration
SFEE on the basis of unit time:
1
]
1

+ + gZ
V
h m W Q
2
2


Where
Q

= heat transfer/unit time; W

= shaft work / unit time, m


=mass flow rate /
unit time
Hence it can be written as,

,
_

+ + + +
1
2
1
1 1 1
2
gZ
V
v p u m Q

,
_

+ + + +
2
2
2
2 1 2
2
gZ
V
v p u m W
s

Where
2 2 2 2 1 1 1 1
V A m and V A m
Where = density A = cross sectional
area
But m m m
2 1
( )
( )
( )
s
W Z Z g
V V
h h m Q


1
]
1

+ +
2 1
2
2
2
1
2 1
2
This is the Steady Flow Energy Equation
Displacement work for a flow process (open system):
From SFEE, when changes in kinetic & potential energies are neglected, q W = dh
46
Or W = q dh --- (1)
From the 1
st
law of thermodynamics, we have q W = du
For a rev. process, W = Pdv
q = du + Pdv
Also, from the definition of enthalpy, h = u + pv
Or dh = du + d (pv)
Sub q & dh in equation (i)
W = [du + p.dv] [du + d (pv)]
= p.dv p.dv v.dp
W = - vdp
Note: With negligible PE & KE, for a non-flow rev. process, the work interaction is equal
to dv p.
2
1
where as for a steady-flow rev. process, it is equal to vdp
2
1

Application of SFEE:
i) Nozzle and Diffuser: Nozzle is a duct of varying c/s area in which the
velocity increases with a corresponding drop in pressure. Since the flow through
the nozzle occurs at a very high speed, there is hardly any time for a fluid to gain
or loose heat and hence flow of the fluid is assumed to be adiabatic. And also
there is no work interaction during the process, i.e., W
s
= 0, Q = 0, Z
1
= Z
2
We have from SFEE, Q W = h + PE + KE
0 = h
2
h
1
+
2
2
1
2
2
V V
( )
2
1
2
2 2 1
2
1
V V h h
i.e., the gain in KE during the process is equal to the decrease in enthalpy of the fluid.
Diffuser is a device to increase the pressure of a fluid during flow with a
corresponding decrease in KE. Thus its function is reverse to that of a nozzle. As final
velocity V
2
in a diffuser is very small, it is very often negligible.
ii) Turbine and Compressor (rotary): Turbine is a device which produces work
by expanding a high pressure fluid to a low pressure. The fluid is first accelerated
in a set of nozzle and then directed through curved moving blades which are fixed
47
V
1
V
2
Z
1
Z
2
= Z
1
on the rotor shaft. The direction of the fluid changes which it flows through the
moving blades, due to which there is a change in momentum and a force exerted
on the blades producing torque on the rotor shaft. Since the velocity of flow of the
fluid through the turbine is very high, the flow process is generally assumed to be
adiabatic, hence heat transfer q = 0. The change in PE is neglected as it is
negligible.
SFEE is W
1-2
= (h
1
h
2
) (V
2
2
V
1
2
)
If mass flow rate is
, m
then,
( ) ( ) Watts V V m h h m W
2
1
2
2 2 1 2 1
2
1

i.e., power developed by the turbine


Compressor is a device in which work is done on the fluid to raise its pressure. A
rotary compressor can be regarded as a reversed turbine. Since work is done on the
system, the rate of work in the above equation is negative and the enthalpy after
compression h
2
will be greater than the enthalpy before compression h
1
.
iii) Throttling Process: When a fluid steadily through a restricted passages like a
partially closed valve, orifice, porous plug etc., the pressure of the fluid drops
substantially and the process is called throttling. In a throttling process, expansion
of the fluid takes place so rapidly that no heat transfer is possible between the
system and the surroundings. Hence the process is assumed to occur adiabatically.
The work transfer in this process is zero.
SFEE is Q
1-2
W
1-2
= h + KE + PE
We have, Q = 0; W = 0; Z
1
= Z
2
, V
1
V
2
0 0 = h
2
h
1
+ 0 + 0 i.e., h
1
= h
2
In a throttling process, the enthalpy remains constant. The throttling process is
irreversible because when a fluid is throttled, it passes through a series of non-
equilibrium states.
48
V
1
V
2
Z
1
Z
2
= Z
1
W
1-2
1
1
2
2
iv) Heat Exchanger: A heat exchanger is a device in which heat is transferred
from one fluid to another. It is used to add or reduced heat energy of the fluid
flowing through the device. Radiator in an automobile, condenser in a steam
power and refrigeration plants, evaporator in a refrigerator are examples of heat
exchangers. There will be no work interaction during the flow of the fluid through
any heat exchanger.
Eg: i) Steam condenser: Used to condense the steam. It a device in which steam loses
heat as it passes over the tubes through which water is flowing.

Figure: Heat Exchanger
We have KE = 0, PE = 0 (as their values are very small compared to enthalpies)
W = 0 (since neither any work is developed nor absorbed)
SFEE is Q = h
2
h
1
i.e., h
1
Q = h
2
--- (1)
Where Q = heat lost by 1 kg of steam passing through the condenser.
Assuming there are no other heat interactions except the heat transfer between steam
and water, then Q = heat gained by water passing through the condenser.

( ) ( )
1 2 1 2
w w w w w w w
T T C m h h m
Substituting Q in the above equation (1),
) T - (T C m h - h
1 2
w w w w 1 2


Where
w
m
= mass of cooling water passing through the condenser
C
w
= specific heat of water
In a condenser there are 2 steady flow streams namely (i) Vapour that losses heat (ii)
The coolant (water) that receives heat.
Let
w
m
= mass flow rate of coolant
s
m
= mass flow rate of steam
h
1w
= Enthalpy-coolant entry
h
1s
= Enthalpy-steam entry
h
2w
, h
2s
= Enthalpy of coolant, steam at exit
w
m
h
1w
+ s
m
h
1s
= w
m
h
2w
+ s
m
h
2s

49
or
( )
( )
w w
s s
s
w
h h
h h
m
m
2 1
1 2


ii) Evaporator: An evaporator is a component of a refrigeration system and is used to
extract heat from the chamber which is to be kept at low temperature. The
refrigerating liquid enters the evaporator, absorbs latent heat from the chamber at
constant pressure and comes out as a vapour. SFEE is
m
h
1
+
Q

=
m
h
2
Since W

= 0, KE = PE = 0

=
m
(h
2
h
1
)
Q

is taken as positive because heat flows from the chamber to the evaporator coil.
Figure: Evaporator
v) Boiler: It is an equipment used for the generation of the steam. Thermal
energy released by combustion of fuel is transferred to water which vapourizes
and gets converted into steam at the desired pressure and temperature. The steam
thus generated is used for
a. Producing mechanical work by expanding it in steam engine or steam
turbine.
b. Heating the residential and industrial buildings in cold weather and
c. Performing certain processes in the sugar mills, chemical and textile
industries.
1. Velocity change is negligible V
1
= V
2
2. Change in elevation is also negligible Z
2
= Z
1
3. Work done = 0
SFEE is h
1
+ q = h
2
q = h
2
h
1
= (u
2
u
1
) + (p
2
v
2
p
1
v
1
)
Unit 3
First Law of Thermodynamics: Joules Experiments, Equivalence of heat work.
Statement of the 1
st
law of thermodynamics, extension of the 1
st
law to non cyclic
processes, energy, energy as a property, modes of energy, pure substance; Definition, two
50
property rule, specific heat at constant volume, enthalpy, specific heat at constant
pressure. Extension of the 1
st
law to control volume; Steady state-steady flow energy
equation, important applications, analysis of unsteady processes such as filling and
evacuation of vessels with and without heat transfer.
The first law of thermodynamics is often called as the law of the conservation of energy,
with particular reference to heat energy and mechanical energy i.e., work.
First law of thermodynamics for a closed system undergoing a cyclic process
The transfer of heat and the performance of work may both cause the same effect in a
system. Energy which enters a system as heat may leave the system as work, or energy
which enters the system as work may leave as heat. Hence, by the law of conservation of
energy, the net work done by the system is equal to the net heat supplied to the system.
The first law of thermodynamics can therefore be stated as follows:
When a system undergoes a thermodynamic cyclic process, then the net heat supplied to
the system from the surroundings is equal to the net work done by the system on its
surrounding.
i.e.,

Q =

W where

represents the sum for a complete


cycle.
The first law of thermodynamics can not be proved analytically, but experimental
evidence has repeatedly confirms its validity and since no phenomenon has been shown
to contradict it, therefore the first law is accepted as a law of nature.
Joules Experiment:
51
Figure: Joules Experiment Figure: Cycle completed by a
system
with two energy
interactions i.e., work
transfer followed by
heat transfer
Figure shows the experiment for checking the first law of thermodynamics. The work
input to the paddle wheel is measured by the fall of weight, while the corresponding
temperature rise of liquid in the insulated container is measured by the thermometer.
The process 1-2 undergone by the system is shown in figure i.e., W
1-2
. Let the insulation
be removed. The system and the surrounding interact by heat transfer till the system
returns to its original temperature, attaining the condition of thermal equilibrium with the
atmosphere. The amount of heat transfer Q
2-1
from the system during this process 2-1 is
shown in figure. The system thus executes a cycle, which consists of a definite amount of
work input W
1-2
to the system followed by the transfer of an amount of heat Q
2-1
from the
system.
Joule carried out many such experiments with different type of work interactions in a
variety of systems, he found that the net work input the fluid system was always
proportional to the net heat transferred from the system regardless of work interaction.
Based on this experimental evidence Joule stated that,
When a system (closed system) is undergoing a cyclic process, the net heat transfer to
the system is directly proportional to the net work done by the system. This statement is
referred to as the first law for a closed system undergoing a cyclic process.
i.e.,

W
If both heat transfer and work transfer are expressed in same units as in the S.I. units then
the constant of proportionality in the above equation will be unity and hence the
mathematical form of first law for a system undergoing a cyclic process can be written as
i.e.,

Q =

W
If the cycle involves many more heat and work quantities as shown in figure, the same
result will be found.
Figure: Cyclic Process on a Property Diagram
52
For this cyclic process the statement of first law can be written as

1 4 3 2 1 1 4 3 2 1
W Q
The cyclic integral in the above equation can be split into a series of non cyclic integral
as
W W W W Q Q Q Q
1
4
4
3
3
2
2
1
1
4
4
3
3
2
2
1
+ + + + + +
or
1
Q
2
+
2
Q
3
+
3
Q
4
+
4
Q
1
=
1
W
2
+
2
W
3
+
3
W
4
+
4
W
1
i.e.,

Q =

W
or (Q)
cycle
= (W)
cycle
This is the first law for a closed system undergoing a cyclic process. i.e., it is stated as
When a closed system is undergoing a cyclic process the algebraic sum of heat transfers
is equal to the algebraic sum of the work transfers.

First law for a closed system undergoing a non-cyclic process (i.e., for a change of
state):
If a system undergoes a change of state during which both heat transfer and work transfer
are involved, the net energy transfer will be stored or accumulated within the system.
If Q is the amount of heat transferred to the system and W is the amount of work
transferred from the system during the process as shown in figure,
The net energy transfer (Q-W) will be stored in the system. Energy in storage is neither
heat or work and is given the name internal energy or simply, the energy of the system.
Q-W = E where E is the increase in the energy of the system
or Q = E + W
If there are more energy transfer quantities involved in the process as shown in figure.
53
System
W
Q
System
Q
1
W
1
W
2
W
3
W
4
Q
2
Q
3
First law gives
(Q
2
+ Q
3
Q
1
) = E + (W
2
+ W
3
W
1
W
4
)
i.e., energy is thus conserved in the operation. Therefore the first law is a particular
formulation of the principle of the conservation of energy. It can be shown that the
energy has a definite value at every state of a system and is therefore, a property of a
system.
Energy A property of the system:
Figure: First law to a non cyclic process
Consider a system that undergoes a cycle, changing from state 1 to state 2 by process A
and returning from state 2 to state 1 by process B.
We have from 1
st
law of thermodynamics,

W Q

For the process, 1-A-2-B-1,

1 2 1 1 2 1 B A B A
W Q

Considering the two separate processes, we have
W W Q Q
B A B A

1
2
2
1
1
2
2
1
+ + --- (1)
Now consider another cycle, the system changing from state 1 to state 2 by process A, as
before and returning to state 1 by process C. For this cycle we can write
W W Q Q
C A C A

1
2
2
1
1
2
2
1
+ + --- (2)
Subtracting (2) from (1), we get
W W Q Q
C B C B

1
2
1
2
1
2
1
2
+
Or, by rearranging, ( ) ( ) W Q W Q
C B

1
2
1
2

Since B and C represent arbitrary processes between state 1 to state 2, we conclude that
the quantity ( Q - W) is the same for all processes between state 1 and state 2.
( Q - W) depends only on the initial and final states and not on the path followed
54
between the two states. This is a point function and differential is a property of the
system. This property is called the energy of the system, E. Therefore, we can write
Q - W = dE
Or Q = dE + W
If it is integrated between initial and final states, 1 and 2, we get
Q
1-2
= E
2
E
1
+ W
1-2
i.e., Q
1-2
- W
1-2
= E
2
E
1

The above equation is the statement of first law for a closed system undergoing a non
cyclic process, where Q
1-2
represents the net heat transfer between the system and the
surroundings during the process, W
1-2
represents net work transfer between the system
and the surroundings during the process and (E
2
E
1
) represents the change in the energy
of the system during the process.
Classification of Energy of the System:

The energy E is an extensive property and the specific energy e = E/m (J/kg) is an
intensive property. Energy E represents the total energy of the system.
i.e., E = kinetic energy (KE) + Potential Energy (PE) + remaining forms of energy.
Since K.E and P.E are macroscopic quantities and can be measured very easily and so
they are considered separately in thermodynamics. The remaining energies (associated
with the motion and position of the molecules, energy associated with the structure of the
atom, chemical energy etc), which can not be measured directly and is the summation of
all microscopic energies is called internal energy of the system.
Internal energy:
It is the energy associated with internal structure of matter. This energy can not be
determined in its absolute values. But it is possible to determine the change in internal
energy of the system undergoing a process by first law of thermodynamics.
Total energy E = KE + PE + IE
Since the terms comprising E are point functions, we can write
dE = d(KE) + d (PE) + dU
The first law of thermodynamics for a change of state of a system may therefore be
written as
Q = dU + d (KE) + d (PE) + W
In words this equation states that as a system undergoes a change of state, energy may
cross the boundary as either heat or work, and each may be positive or negative. The net
change in the energy of the system will be exactly equal to the net energy that crosses the
boundary of the system. The energy of the system may change in any of three ways,
namely, by a change in IE, KE or P.E
Sub. For KE and PE in the above equation
55
Q = dU +
2
) (
2
mV d
+ d (mgZ) + W
In the integral form this equation is, assuming g is a constant
Q
1-2
= U
2
U
1
+
( )
2
2
1
2
2
V V m
+ mg (Z
2
Z
1
) + W
1-2
In most of the situations the changes in KE and PE are very small, when compared with
the changes in internal energies. Thus KE and PE changes can be neglected.
Q = dU + W
or Q
1-2
= U
2
U
1
+ W
1-2

Law of conservation of energy (2nd corollary of first law of thermodynamics)
From first law of thermodynamics Q
1-2
= E
2
E
1
+ W
1-2
This equation in effect, a statement of the conservation of energy. The net change of the
energy of the system is always equal to the net transfer of energy across the system
boundary as heat and work. For an isolated system, Q = 0, W = 0 E
2
E
1
= 0
For an isolated system, the energy of the system remains constant.
Therefore, the first law of thermodynamics. may also be stated as follows, Heat and
work are mutually convertible but since energy can neither be created nor destroyed, the
total energy associated with an energy conversion remains constant.
Perpetual Machine of first kind (3
rd
Corollary):
Any system which violates the first law of thermodynamics is called the Perpetual
Motion machine of first kind. i.e., It is impossible to construct a perpetual motion
machine of first kind. A perpetual machine is one which can do continuous work
without receiving energy from other systems or surroundings. It will create energy on its
own and thus violates first law. But from our experience we also know that it is
impossible to construct such a machine, as frictional resistance would not allow it to run
for an indefinite period.
Problems:
32. In a cyclic process, heat temperature are + 14.7 kJ, -25.2 kJ, -3.56 kJ and +31.5
kJ. What is the net work for this cyclic process.
Solution: 1
st
law of thermodynamics for a cyclic process is

W Q
i.e., Net work = 14.7 25.2 -3.56 + 31.5
= 17.44 kJ

33. Consider a cyclic process in a closed system which includes three heat
interactions, namely Q
1
= 20 kJ, Q
2
= -6kJ, and Q
3
= -4 kJ and two work interactions
for which W
1
= 4500 N-m. Compute the magnitude of the second work interaction W
2
in Nm.
Solution: We have for a closed system undergoing cyclic process,
56

W Q
20000 6000 4000 = 4500 + W
2
W
2
= 5500 Nm
34. When the state of a system changes from state 1 to state 3 along the path 1-2-3 as
shown in figure, 80 kJ of heat flows into the system and the system does 30 kJ of
work. (a) How much heat flows into the system along the path 1-4-3 if work done by
the system is 10 kJ (b) when the state of the system is returned from state 3 to state 1
along the curved path, the work done on the system is 20 kJ. Does the system absorb
or liberate heat? Find its magnitude. (c) If U
1
= 0 and U
4
= 40kJ, find the heat
absorbed in the process 1-4 and 4-3 respectively.
Solution:
a) Along the path 1-2-3,
From 1
st
law of thermodynamics, Q
1-3
= U
3
-U
1
+ W
1-3
From the data given, 80 = (U
3
U
1
) + 30
(U
3
U
1
) = 50 kJ
Along the path 1-4-3, we have
Q
1-3
= U
3
U
1
+ W
1-3
From the data given, Q
1-3
= 50 + 10
= 60 kJ U is property of a system
i.e., Work is done by the system
b) Along the path 3-A-1,
(U
1
U
3
) = Q
3-1
W
3-1
Or Q
3-1
= (U
1
U
3
) + W
3-1
= -50 -20 = -70 kJ
Negative sign indicates that heat is liberated from the system.
c) Along the path 1-4
Q
1-4
= U
4
U
1
+ W
1-4
= 40-0+10 (since W
1-4-3
= W
1-4
+ W
4-3
= 10 + 0 = 10)
= 50 kJ
Positive sign indicates heat is absorbed by the system
Along the path 4-3
Q
4-3
= U
3
U
4
+ W
4-3
57
1
2 3
4
p
V
A
= 50 40 + 0 = 10 kJ
35. A domestic refrigerator is loaded with food and the door closed. During a certain
period the machine consumes 1 kWhr of energy and the internal energy of the system
drops by 5000 kJ. Find the net heat transfer. for the system.
Solution: W
1-2
= 1kWhr = -1 x3600 kJ U
2
U
1
= -5000 kJ
From 1st law, Q
1-2
= (U
2
-U
1
) + W
1-2
= -5000 -3600 = -8600 kJ = - 8.6 mJ
36. For the following process in a closed system find the missing data (all in kJ)
Process Q W U
1
U
2
U
a) 35 20 -10 5 15
b) 15 -6 -27 -6 21
c) -7 10 20 3 -17
d) -27 -7 28 8 -20
Solution: Process (a): Q = U + W
= U
2
U
1
+ W
1-2
but U
2
-U
1
= 15 U
2
= 5
= 15 + 20 = 35 kJ
Process (b): Q = U
2
U
1
+ W
15 = -6-U
1
-6
27 = -U
1
U
1
= -27 kJ
U = U
2
U
1
= - 6 + 27 = 21 kJ
Process (c) - 7 = U
2
20 + 10
U
2
= 3 kJ U = 3-20 = -17 kJ
Process (d) U = U
2
U
1
= - 20
= 8 U
1
= - 20 U
1
= 28 kJ
A = 8 28 7 = - 27 kJ
37. A fluid system, contained in a piston and cylinder machine, passes through a
complete cycle of four processes. The sum of all heat transferred during a cycle is
-340 kJ. The system completes 200 cycles minutes. Complete the following table
showing the method for each item, and compute the net rate of work output in kW.
Process Q (kJ/min) W (kJ/min) E (kJ/min)
1-2 0 4340 -4340
2-3 42000 0 42000
3-4 -4200 69000 -73200
4-1 -105800 -141340 35540
Solution: Given

cycle
Q
= -340 kJ, No. of cycle = 200 cycles / min
Process 1-2: Q
1-2
= (E
2
E
1
) + W
1-2
0 = E + W
1-2

E = -4340 kJ/min
Process 2-3: 42000 = E + 0
58
Q
1-2
= 42000 kJ/min
Process 3-4: -4200 = -73200 + W
3-4

W
3-4
= 69000 kJ/min
Process 4-1:
cycle
Q
= -340 kJ
The system completes 200 cycle/min

cycle
Q
= -340 x 200 = - 68000 kJ / min
But, Q
1-2
+ Q
2-3
+ Q
3-4
+ Q
4-1
= -68000
Q
4-1
= - 68000 0 42000 + 4200
= - 105800 kJ/min
Also, dE = 0, since cyclic integral of any property is zero
(E)
1-2
+ (E)
2-3
+ (E)
3-4
+ (E)
4-1
= 0
-4340 + 42000 73200 + (E)
4-1
=0
(E)
4-1
= 35540 kJ/min
Therefore Q
4-1
= (E)
4-1
+ W
4-1
- 105800 = 35540 + W
4-1

W
4-1
= -141340 kJ/min
Since
cycle
cycle
W Q

= - 68000 kJ/min
Rate of work output =
60
68000
= 1133.33 kW
59
Problems:
1. 12 kg of a fluid per minute goes through a reversible steady flow process. The
properties of fluid at the inlet are p
1
= 1.4 bar,
1
= 25 kg/m
3
, V
1
= 120 m/s & u
1
=
920 kJ/kg and at the exit are p
2
= 5.6 bar,
2
= 5 kg/m
3
, V
2
= 180 m/s and u
2
= 720
kJ/kg. During the passage, the fluid rejects 60 kJ/s and raises through 60m. Determine
i) the change in enthalpy ii) work done during the process.
Solution: i) Change in enthalpy h = (u + pV)
= (720 x 10
3
920 x 10
3
) + (5.6 x 10
5
x 1/5 1.4 x 10
5
x
1/25)
= - 93.6 kJ
ii) SFEE for unit time basis is
[ ] PE KE h m W Q + +

--- (1)
2
120 180
2
2 2 2
1
2
2


V V
KE = 9000 J = 9 kJ/kg
PE = g (Z
2
Z
1
) = 9.81 (60) = 0.589 kJ/kg
Substituting in equation (1), - 60 W

= 12/60 [- 93.6 + 9 + 0.589]


W

= - 43.2 kW
Negative sign indicates work is done on the fluid in the reversible steady flow process.
2. In the turbine of a gas turbine unit the gases flow through the turbine at 17 kg/s
and the power developed by the turbine is 14000 kW. The enthalpies of the gases at
inlet and outlet are 1200 kJ/kg and 360 kJ/kg respectively, and the velocities of the
gases at inlet and outlet are 60 m/s and 150 m/s respectively. Calculate the rate at
which the heat is rejected from the turbine. Find also the area of the inlet pipe given
that the specific volume of the gases at inlet is 0.5 m
3
/kg.
Solution: m = 17 kg/s v
1
= 0.45 m
3
/kg
P = 1400kW Work done
17
14000
W
= 823.53 kJ/kg
h
1
= 1200kJ/kg h
2
= 360kJ/kg V
1
= 60m/s V
2
= 150 m/s
We have SFEE, Q W = h + PE + KE
kg kJ
V V
KE / 45 . 9
2
2
1
2
2


kg kJ h h h / 840
1 2

PE = 0
Substituting in SFEE equation,
Q 823.53 = 840 + 0 + 9.45
Q = - 7.02 kJ/kg
i.e., heat rejected = 7.02 x 17 = 119.34 kW
Also, we have 1 1
1
1 1 1
1
V A
v
V A m
) 60 (
5 . 0
1
17
1
A
A
1
= 0.142 m
2
60
3. Air flows steadily at the rate of 0.4 kg/s through an air compressor entering at 6
m/s with a pressure of 1 bar and a specific volume of 0.85 m
3
/kg, and leaving at 4.5
m/s with a pressure of 6.9 bar and a specific volume of a 0.16 m
3
/kg. The internal
energy of air leaving is 88 kJ/kg greater than that of the air entering. Cooling
water in a jacket surrounding the cylinder absorbs heat from the air at the rate of 59
kJ/s. Calculate the power required to drive the compressor and the inlet and outlet
pipe cross sectional areas.
Solution: m = 0.4 kg/s V
1
= 6m/s p
1
= 1 x 10
5
Pa
v
1
= 0.85 m
3
/kg V
2
= 4.5 m/s P
2
= 6.9 x 10
5
Pa v
2
=0.16m
3
/kg
u = 88 kJ/kg Q = - 59 kJ/s ? W

pv = 6.9 x 10
5
x 0.16 1 x 10
5
x 0.85 = 25.4 kJ/kg
u = 88 kJ/kg
h = (u + pv) = 113.4 kJ/kg
SFEE for unit time basis is given by,
[ ] PE KE h m W Q + +

kg J
V V
KE / 88 . 7
2
2
1
2
2


PE = 0
Substituting in the SFEE equation,
1
]
1

+ 0
1000
88 . 7
4 . 113 4 . 0 59 W

kW W 4 . 104


Negative sign indicates work is done in the air compressor i.e., power input to the
compressor.
ii) We have
2 2 2 1 1 1
V A V A m

2 2
2
1 1
1
1 1
V A
v
V A
v

2
1
1
1
057 . 0 m
V
v m
A

2
2
2
2
0142 . 0 m
V
v m
A

4. A turbine operating under steady flow conditions receives steam at the following
state. pressure 13.8 bar, specific volume 0.143 m
3
/kg, i.e., 2590 kJ/kg, velocity 30
m/s. The state of the steam leaving the turbine is pressure 0.35 bar, specific volume
4.37 m
3
/kg, i.e., 2360 kJ/kg, velocity 90 m/s. Heat is lost to the surroundings at the
rate of 0.25 kJ/s. If the rate of steam flow is 0.38 kg/s, what is the power developed
by the turbine? (102.8 kW).
5. At the inlet to a certain nozzle the enthalpy of the fluid is 3025 kJ/kg and the
velocity is 60 m/s. At the exit from the nozzle the enthalpy is 2790 kJ/kg. The nozzle
is horizontal and there is negligible heat loss from it. i) Find the velocity at the nozzle
exit. ii) If the inlet area is 0.1 m
2
and specific volume at inlet is 0.19 m
3
/kg, find the
61
rate of flow of fluid. iii) If the specific volume at the nozzle exit is 0.5 m
3
/kg, find the
exit area of the nozzle.
Solution: h
1
= 3025 kJ/kg V
1
= 60 m/s h
2
= 2790 kJ/kg Z
2
= Z
1
Q = 0
SFEE is Q W = h + PE + KE
For a nozzle, W = 0, Q = 0, PE = 0
Substituting in SFEE, we get
2
., .
2
2
2
2
1
1 2
2
2
2
1
V V
h h e i
V V
h


or 2 (h
2
h
1
) = ( )
1 2
2
1 2
2
2
2
1
2 h h V V V V
= 688.2 m/s
ii)
1 1 1 1
V A m
( ) ( ) s kg V A
v
/ 6 . 31 60 1 . 0
19 . 0
1
.
1
1 1
1

iii)
2 2 2 2
V A m

( ) 2 . 688 .
5 . 0
1
6 . 31
2
A
A
2
= 0.0229 m
2
6. In a steam power plant 1.5 kg of water is supplied per second to the boiler. The
enthalpy and velocity of water entering the boiler are 800 kJ/kg and 10 m/s. Heat at
the rate of 2200 kJ/kg of water is supplied to the water. The steam after passing
through the turbine comes out with a velocity of 50 m/s and enthalpy of 2520kJ/kg.
The boiler inlet is 5m above the turbine exit. The heat loss from the boiler is 1800
kJ/m and from the turbine 600 kJ/min. Determine the power capacity of the turbine,
considering boiler and turbine as single unit.
Solution: m
= 1.5 kg/s h
1
= 800 kJ/kg V
1
= 10 m/s Q = 2200 kJ/kg
V
2
= 50 m/s h
2
= 2520 kJ/kg Z
1
= 5 m Z
2
= 0 W = ?
Heat loss from boiler and turbine is 1800 + 600 = 2400 kJ/min
= 40 kJ/sec =
5 . 1
40
= 26.67 kJ/kg
Net heat added to the water in the boiler Q = 2200 26.67 = 2173.33 kJ/kg
SFEE is Q - W = h + KE + PE --- (1)
h = h
2
h
1
= 2500 800 = 1720 kJ/kg
kg kJ kg J
V V
KE / 2 . 1 / 1200
2
10 50
2
2 2 2
1
2
2


PE = g (Z
2
Z
1
) = 9.81 (0-5) = - 49.05 J/kg = -49.05 x 10
-3
kJ/kg
Substituting in (1), 2173.33 W = 1720 + 1.2 49.05 x 10
-3

W = 452.18 kJ/kg
Power capacity of the turbine P = 452.18 x 1.5 = 678.27 kW
7. A centrifugal air compressor used in gas turbine receives air at 100 KPa and 300
K and it discharges air at 400 KPa and 500 K. The velocity of air leaving the
compressor is 100 m/s. Neglecting the velocity at the entry of the compressor,
determine the power required to drive the compressor if the mass flow rate is 15
kg/sec. Take C
p
(air) = 1 kJ/kgK, and assume that there is no heat transfer from the
compressor to the surroundings.
62
Solution: p
1
= 100 x 10
3
N/m
2
T
1
= 300 K p
2
= 400 x 10
3
N/m
2
T
2
= 500K
V
2
= 100 m/s W = ? m = 15 kg/s C
p
= 1kJ/kgK Q = 0
SFEE is Q W = h + KE + PE
h = m
C
p
(T
2
T
1
) = 15 (1) (500 300)
= 3000 kJ/s = 3000 / 15 = 200 kJ/kg
kg kJ kg J
V
KE / 5 / 5000
2
100
2
2 2
2

Substituting in SFEE we have
0 W = 200 + 5 = 205 kJ/kg
i.e., W = - 205 kJ/kg = - 205 x 15 kJ/s = - 3075 kW
Negative sign indicates work is done on the centrifugal air compressor
Power required = 3075 kW
8. In a water cooled compressor 0.5 kg of air is compressed/sec. A shaft input of 60
kW is required to run the compressor. Heat lost to the cooling water is 30% of input
and 10% of the input is lost in bearings and other frictional effects. Air enters the
compressor at 1 bar and 20
0
C. Neglecting the changes in KE & PE, determine the exit
air temperature. Take C
p
= 1kJ/kg
0
C air.
Solution: m = 0.5 kg/s W = 60 kW H
L
= (30% + 10%) input = 40%
input
p
1
= 1 x 10
5
N/m
2
t
1
= 20
0
C
SFEE is Q W = h + KE + PE
KE = 0, PE = 0
W = 60 kW = 60 kJ/sec
5 . 0
60

= 120 kJ/kg
Heat lost to the surroundings = 40% (input) = 0.4 (120)
= 48 kJ/kg
Substituting in SFEE we have
- 48 (-120) = h
But h = C
p
(t
2
t
1
)
- 48 + 120 = C
p
(t
2
t
1
)
i.e., 72 = 1 (t
2
20)
t
2
= 92
0
C
9. A petrol engine develops 50 kW brake power. The fuel and air flow rates are 10
kg and 107 kg/hr. The temperature of fuel air mixture entering the engine is 20
0
C and
temperature of gases leaving the engine is 500
0
C. The heat transfer rate from the
engine to the cooling water circulated is 50kJ/s and that to the surroundings 10 kJ/s.
Evaluate the increase in the specific enthalpy of the mixture as it flows through the
engine.
Solution: W = 50 kW = 50 kJ/s = 1538.46 kJ/kg m
= (10 + 107)kg/hr =
0.0325 kg/s
t
1
= 20
0
C t
2
= 500
0
C Q
L
= 50 + 10 = 60 kJ/s = 1846.15 kJ/kg
SFEE is Q
L
W = h + KE + PE but KE = PE = 0
Substituting in SFEE, we have
- 1846.15 1538.46 = h
63
h = - 3384.61 kJ/kg
= - 109.99 kJ/sec
Negative sign indicates there is decrease in enthalpy of the mixture.
10. Air at a temperature of 15
0
C passes through a heat exchanger at a velocity of 30
m/s where its temperature is raised to 800
0
C. It then enters a turbine with the same
velocity of 30 m/s and expands until the temperature falls to 650
0
C. On leaving the
turbine, the air is taken at a velocity of 60 m/s to a nozzle where it expands until the
temperature has fallen to 500
0
C. If the air flow rate is 2 kg/s, calculate i) the rate of
heat transfer to the air, ii) the power output from the turbine assuming no heat loss,
and iii) the velocity at exit from the nozzle, assuming no heat loss. Take the enthalpy
of air as h = C
p
t, where C
p
= 1.005 kJ/kg
0
C.
Solution: t
1
= 15
0
C, V
1
= 30 m/s, t
2
= 800
0
C V
2
= 30 m/s t
3
= 650
0
C
V
3
= 60 m/s t
4
= 500
0
C m
= 2 kg/s
i) Heat exchanger: Q = ?
Q W = ( h + KE + PE) but KE = PE = 0
Q = C
p
(t
2
t
1
)
= 1.005 (800 15) = 788.93 kJ/kg
or Q = 1577.85 kJ/s
ii) Turbine: W = ?
Q W = h + KE + PE, but Q = 0, PE = 0
h = C
p
(t
3
t
2
) = (650 800) 1.005 = - 150.75 kJ/kg
kg kJ kg J
V V
KE / 35 . 1 / 1350
2
30 60
2
2 2 2
2
2
3


Substituting in SFEE, we have
- W = - 150.75 + 1.35
W = 149.4 kJ/kg
= 149.4 x 2 = 298.8 kW
iii) Nozzle: V
4
=?
Q W = h + KE + PE, but Q = 0, W = 0, PE = 0
Substituting in SFEE, we have
(h
3
h
4
) =
2
2
3
2
4
V V
i.e., 2C
p
(t
3
t
4
) =
2
3
2
4
V V
2 (1.005) (650 500) + 60
2
= V
4
2

V
4
= 552.36 m/s
11. A 260 mm dia cylinder fitted with a frictionless leak pro of piston contains 0.02
kg of steam at a pressure of 6 x 10
5
N/m
2
and a temperature of 200
0
C. As the piston
moves slowly outwards through a distance of 305 mm the steam undergoes a fully
resisted expansion according to the law pV
n
= a constant to a final pressure 1 x
10
5
N/m
2
. Determine i) value of the index n ii) work done by the steam iii) Magnitude
and sign of heat transfer.
Solution: d = 0.26 m, m = 0.02 kg p
1
= 6 x 10
5
Pa t
1
= 200
0
C
l = 0.305 m, pV
n
= C p
2
= 1 x 10
5
Pa
64
Stroke volume =
( ) ( )
3 2 2
016193 . 0 305 . 0 26 . 0
4 4
m l d

Considering steam as a perfect gas, p
1
V
1
= mRT
1
,
( ) ( )
3 3
5
1
10 525 . 4
10 6
473 287 02 . 0
m x
x
V


Final volume = V
2
= V
1
+ stroke volume
= 4.525 x 10
-3
+ 0.016193
= 0.020693m
3
i) n =? p
1
V
1
n
= p
2
V
2
n
179 . 1
10 525 . 4
20693 . 0
10 1
10 6
., .
3 5
5
1
2
2
1

,
_

,
_

n
x x
x
e i
V
V
p
p
or
n
n
ii) kJ
n
V p V p
done Work 607 . 3
1
.
2 2 1 1

iii) For reversible polytropic process,


607 . 3
179 . 0
179 . 1 4 . 1
1

,
_



,
_

W
n
n
Q

= 4.453 kJ
12. Air flows steadily at the rate of 0.5 kg/s through an air compressor, entering at 7
m/s velocity, 100 KPa pressure, 0.95 m
3
/kg volume, and leaving at 5 m/s velocity,
700 KPa pressure and 0.19 m
3
/kg. The internal energy of the air leaving is 93 kJ/kg
greater than that of the air entering. Cooling water in the compressor jackets absorbs
heat from the air at the rate of 58 kW. i) compute the rate of shaft work in kW ii) find
the ratio of the inlet pipe dia to outlet pipe diameter.
Solution: m
= 0.5 kg/s V
1
= 7 m/s p
1
= 100 x 10
3
Pa V
1
= 0.95 m
3
/kg
V
2
= 5 m/s p
2
= 700 x 10
3
Pa v
2
= 0.19 m
3
/kg (u
2
u
1
) = 93 kJ/kg
Q
L
= 58 kJ/s
kg kJ / 116
5 . 0
58

Z
1
= Z
2
Q W = h + KE + PE--- (1)
h = (U + pV) = (U
2
U
1
) + (p
2
V
2
p
1
V
1
)
= 93 x 10
3
+ [700 x 10
3
x 0.19 100 x 10
3
x 0.95]
= 131000 J/kg = 131 kJ/kg
kg kJ x kg J
V V
KE / 10 12 / 12
2
3
2
1
2
2


PE = 0
Equation (1) becomes, - 116 W = 131 12 x 10
-3
W = - 246.99 kJ/kg
= - 246.99 x 0.5 = - 123.49 kW
Negative sign indicates shaft work done on the compressor.
ii)
m
=
1
A
1
V
1
=
2
A
2
V
2
5714 . 3
2 1
1 2
2
1
2
2 2
1
1 1

v V
v V
A
A
v
V A
v
V A
65
5714 . 3
4
4
2
2
2
1

d
d
8898 . 1
2
1

d
d
13. A gas flows into a turbine with an initial pressure of 7 bar, specific volume 0.2 m
3
and velocity 150 m/s. The corresponding values of pressure, specific volume and
velocity at the exit are 3.5 bar, 0.5 m
3
and 300 m/s respectively. During the expansion
of gas in the turbine its internal energy decreases by 92 kJ/kg and loss due to radiation
was 13 kJ/kg. What amount of shaft work is developed per kg of gas flow.
14. The compressor of a large gas turbine receives air from the surroundings at 95
KPa and 20
0
C. The air is compressed to 800 KPa according to the relation pV
1.3
=
constant. The inlet velocity is negligible and the outlet velocity is 100 m/s. The power
input to the compressor is 2500 kW, 20% of which is removed as heat from the
compressor. What is the mass flow rate of the air? Take C
p
= 1.01 kJ/kg
0
K for air.
Solution: p
1
= 95 x 10
3
N/m
2
T
1
= 293 k p
2
= 800 x 10
3
N/m
2
pV
1-3
= C V
2
= 100 m/s W = - 2500 kW
Q

= - 0.2 (2500) =
-500 kW
m = ? C
p
= 1.01 kJ/kg
0
K
we have
K T
p
p
T
T
n
n
0
3 . 1
3 . 0
2
1
1
2
1
2
09 . 479
95
800
293
,
_

,
_

,
_

+ + gZ
V
h m W Q
2
2
2


- 500 + 2500
( )
1
]
1

+
3
2
10
2
100
293 09 . 479 01 . 1 x m

m
= 10.365 kg/sec
15. The steam supply to an engine is comprised of two streams which mix before
entering the engine. One stream is supplied at the rate of 0.01 kg/s with an enthalpy of
2950 kJ/kg and a velocity of 20 m/s. The other stream is supplied at the rate of 0.1
kg/s with an enthalpy of 2569 kJ/kg and a velocity of 120 m/s. At the exit from the
engine the fluid leaves as two streams, one of water at the rate of 0.001 kg/s with an
enthalpy of 420 kJ/kg and the other of steam. The fluid velocity at the exit are
negligible. The engine develops a shaft power of 25 kW. The heat transfer is
negligible. Evaluate the enthalpy of the second exit stream. (Ans. 2462 kJ/kg)
16. A perfect gas flows through a nozzle where it expands in a reversible adiabatic
manner. The inlet conditions are 22 bar, 500
0
C, 38 m/s. At exit the pressure is 2 bar.
Determine the exit velocity and exit area if the flow rate is 4 kg/s. Take R = 190 J/kg-
0
k and = 1.35
[hint:
( )
r
r
p p
p
p
T
T
T T C h R C
1
1
2
1
2
1 2
, ,
1

,
_

66
p
2
v
2
= RT
2
v
2
= 0.3933 m
3
/kg
2
2
1
v

m
=
2
A
2
V
2
]
Ans.: V
2
= 726 m/s , A
2
= 0.002167 m
2
17. A steam turbine operate under steady flow conditions receiving steam at the
following state: Pressure 15 bar, internal energy 2700 kJ/kg, velocity 300 m/s,
specific volume 0.17 m
3
/kg and velocity 100 m/s.
The exhaust of steam from the turbine is at 0.1 bar with internal energy 2175 kJ/kg,
specific volume 15m
3
/kg and velocity 300 m/s. The intake is 3 m above the exhaust.
The turbine develops 35 kW and heat loss over the surface of turbine is 20kJ/kg.
Determine the steam flow rate through the turbine. [Ans.: 0.0614 kg/s]
18. Determine the power required to drive a pump which raises the water pressure
from 1 bar at entry to 25 bar at exit and delivers 2000 kg/hr of water. Neglect changes
in volume, elevation and velocity and assume specific volume of water to be
0.001045m
3
/kg.
Solution: We have
[ ] PE KE h m W Q + +

, 0 Q

KE = 0, PE = 0
Substituting in the SFEE equation,
( ) pv u m W +

u = 0 Water does not experience any change in temperature


( )
1 1 2 2
v p v p m W

( )
1 2
p p v m
but v
1
= v
2
( ) ( )
1000
10 1 10 25 001045 . 0
3600
2000
5 5
x x

= - 1.393 kW
19. In a conference hall comfortable temperature conditions are maintained in winter
by circulating hot water through a piping system. The water enters the piping system
at 3 bar pressure and 50
0
C temperature (enthalpy = 240 kJ/kg) and leaves at 2.5 bar
pressure and 30
0
C temperature (enthalpy = 195 kJ/kg). The exit from the piping
system is 15 m above the entry. If 30 MJ/hr of heat needs to be supplied to the hall,
make calculation for the quantity of water circulated through the pipe per minute.
Assume that there are no pumps in the system and that the change in KE is negligible.
Solution: Q W = ( h + KE + PE)
W = 0, KE = 0
Q = (195 240) +
1000
) 15 ( 81 . 9

= - 44.853 kJ/kg
Mass of water to be circulated = min / 15 . 11
60 10 853 . 44
10 30
3
6
kg
x x
x

Unsteady flow process: In a steady flow process we have assumed that the mass and
energy within the system remain constant and do not vary with time. In an unsteady flow
process, mass and energy within the control volume vary continuously. The fluid flow
into and out of the system.
67
Example: Filling or evacuation of a tank, (internal energy as well as mass of the
tank changes with time), the condition of water in the cylinder jacket of an I.C. engine (is
time dependant)
Analysis: Consider the flow of a fluid through a pipe line into the cylinder. Let m
1
be the
mass of the fluid initially in the cylinder at pressure p
1
, temperature t
1
and m
2
the final
mass in the cylinder at pressure p
2
, temperature t
2
. The mass that flows into the cylinder is
thus (m
2
m
1
).
There are two ways for solving problems involving unsteady flow (i) Closed system
analysis (ii) Control volume analysis
(i) Closed system analysis:
Since no mass crosses the boundary of the system, the boundary of the system is
selected in such a way that it includes not only the cylinder but also that portion of
the fluid in the pipe line which will be introduced eventually into the cylinder as
shown in figure. That means the system has variable boundaries which at the final
state will be the same as that of the cylinder. Initially energy of the system E
1
is
composed of the internal energy of the mass initially in the cylinder, m
1
u
1
plus the
energy of the fluid which will eventually flow the pipe line into the cylinder, (m
2
m
1
)
1
1
]
1

+
2
2
p
p
V
u
where the subscript p refers to the condition of the fluid in the
pipe line. At the final state, energy E
2
of the fluid in the system will be equal to
m
2
u
2
.
Neglecting the change in PE, the change in energy is,
E
2
E
1
= m
2
u
2
-
( )
1
1
]
1

'

+ +
2
2
1 2 1 1
p
p
V
u m m u m
To find out work done on the system, consider a mass in the pipe line (m
2
m
1
)
which is subjected to a controlled pressure P
p
. The flow work due to the flow of mass
(m
2
m
1
) into the cylinder from the (m
2
m
1
) v
p
in the pipe line to a zero volume is
W = P
p
[0 (m
2
m
1
) v
p
] = - (m
2
m
1
) P
p
v
p
Where v
p
is the specific volume of the fluid in the pipeline.
Applying 1
st
law of thermodynamics,
68
Pipe line
System
Boundary
Cylinder
(System)
( ) ( )
1
1
]
1

'

+ + +
2
2
1 2 1 1 2 2 1 2
p
p p p
V
u m m u m u m v P m m Q
But h
p
= u
p
+ P
p
v
p
,
Above equation becomes,
( )
1
1
]
1

+
2
2
1 2 1 1 2 2
p
p
V
h m m u m u m Q
--- (1)
(ii) Control Volume analysis:
The cylinder itself is taken as the control volume as shown in figure. In this case,
there is no work interaction. Using the general equation 1
st
law and considering no
mass flows out of the control volume and neglecting the change in PE, as in the
earlier case we have
( ) ( )

U
d
d
V
h m m Q
p
p

1
1
]
1

+ +
2
2
1 2

or
( )
1 1 2 2
2
1 2
2
u m u m
V
h m m Q
p
p

1
1
]
1

+ +
--- (2)
If the tank would have been thermally insulated and initially empty, Q = 0 and m
1
= 0
substituting into equation (1) and simplifying, we get
2
2
2
u
V
h
p
p
+ --- (3)
Also if KE in the pipe line is not appreciable, h
p
= u
2
i.e., the specific enthalpy of the fluid
in the pipe line is equal to the specific internal energy of the fluid in the cylinder at the
final state.
Note: The tank emptying process is the reverse of filling process i.e., there is flow of
fluid from the tank (cylinder) to the surroundings.
Analogous to filling process, applying 1
st
law, of thermodynamics, we have
( )
1
1
]
1

+
2
2
2 1 2 2 1 1
p
p
V
h m m u m u m Q
--- (4)
69
Pipe line
C.V
Boundary
Cylinder
(Control Volume)
Where h
p
and V
p
are the specific enthalpy and velocity of leaving fluid.
For no heat transfer and negligible exit velocity,
(m
1
m
2
) h
p
= m
1
u
1
m
2
u
2
Further if the tank is to be fully emptied (m
2
= 0)
i.e., m
1
h
p
= m
1
u
1
or h
p
= u
1
i.e., the specific enthalpy of the fluid in the cylinder is equal to the specific
internal energy of the fluid in the pipe line at the final state.
Problems:
1. A household gas cylinder initially evacuated is filled by 15 kg gas supply of
enthalpy 625 kJ/kg. After filling, the gas in the cylinder has the following parameters:
pressure 10 bar, enthalpy 750 kJ/kg and specific volume 0.0487 m
3
/kg. Evaluate the
heat received by the cylinder from the surroundings.
Solution: Given: m
2
= 15 kg h
p
= 625 kJ/kg P
2
= 10 bar h
2
= 750 kJ/kg
v
2
= 0.0487 m
3
/kg
SFEE to the filling process is
Q = m
2
u
2
m
1
u
1
(m
2
m
1
)
1
1
]
1

+
2
2
p
p
V
h
The cylinder is initially evacuated i.e., m
1
= 0, also
0
p
V
i.e., Q = m
2
u
2
m
2
h
p
= m
2
[(h
2
p
2
v
2
) h
p
]
( )
1
]
1

,
_

625
10
0487 . 0 10 10
750 15
3
5
x
= 1144.5 kJ
2. An insulated and rigid tank contains 5 m
3
of air at 10 bar and 425 K. The air is
then let off to atmosphere through a valve. Determine the work obtainable by utilizing
the KE of the discharge air. Take C
p
= 1 kJ/kg K, C
V
= 0.714 kJ/kg
0
-K atmosphere
pressure = 1 bar.
Solution: Given: V
1
= V
2
= 5 m
3
P
1
= 10 bar T
1
= 425 K
The situation corresponds to emptying process, for which the energy balance equation is,
Q + (m
1
m
2
)
2 2 1 1
2
2
u m u m
V
h
p
p
+
Insulated and rigid tank i.e., Q = 0
( )
2 2 1 1
2
2 1
2
u m u m
V
h m m
p
p

1
1
]
1

+
Suffix p refers to discharge condition.
Or ( ) ( )
p
p
h m m u m u m
V
m m
2 1 2 2 1 1
2
2 1
2

= m
1
C
V
T
1
m
2
C
V
T
2
(m
1
m
2
) C
p
T
2
--- (1)
70
We have,
4 . 1
714 . 0
1

V
p
C
C

p
2
= 1 bar, R = C
p
C
V
= 0.286 kJ/kg K
r
P
P
T
T
1
1
2
1
2

,
_

K T e i 220
10
1
425 ., .
286 . 0
2

,
_

Also, p
1
V
1
= m
1
RT
1
( )
kg
x x
m 14 . 41
425 286
5 10 10
5
1

& p
2
V
2
= m
2
RT
2
( )
kg
x x
m 95 . 7
220 286
5 10 1
5
2

Substitute in equation (1) on RHS, we have
( )
2
2
2 1
p
V
m m = 41.14 (0.714) (425) 7.95 (0.714) (220)
- (41.14 7.95) (1.0) (220)
= 3933.35 kJ
3. A vessel of constant volume 0.3 m
3
contains air at 1.5 bar and is connected via a
valve, to a large main carrying air at a temperature of 38
0
C and high pressure. The
valve is opened allowing air to enter the vessel and raising the pressure there in to 7.5
bar. Assuming the vessel and valve to be thermally insulated, find the mass of air
entering the vessel.
Solution: The situation corresponds to filling process, for which the energy balance
equation is,
( )
1 1 2 2
2
1 2
2
u m u m
V
h m m Q
p
p

1
1
]
1

+ +
With thermally insulated and neglecting KE of air,
(m
2
m
1
)h
p
= m
2
u
2
m
1
u
1
or (m
2
m
1
) C
p
T
p
= m
2
C
V
T
2
m
1
C
V
T
1
Air is a perfect gas, i.e., pV = mRT
V V V Given
R
V p
T m and
R
V p
T m e i
2 1
1
1 1
2
2 2
., .
( ) ( )
1 2 1 2
p p
R
V
C T C m m
V p p

( ) ( )
1 2 1 2
.
1
.
1
p p
R
V
r T
m m or
p

( )
5
10 5 . 1 5 . 7
287
3 . 0
.
4 . 1
1
.
311
1

= 1.44 kg
= Mass of the air entering
4. After the completion of exhaust stroke of an I.C. engine the piston cylinder
assembly remains filled up with 1 x 10
-4
kg of combustible products at 800 K. During
the subsequent suction stroke, the piston moves outward and 16 x 10
-4
kg of air at 290
K is sucked inside the cylinder. The suction process occurs at constant pressure and
71
heat interaction is negligible. Evaluate the temperature of gases at the end of suction
stroke. For air and gases, take C
p
= 1kJ/kg
Solution: The energy equation for the filling process is,
( ) W u m u m
V
h m m Q
p
p
+
1
]
1

+ +
1 1 2 2
2
1 2
2
With no heat interaction and negligible KE of air in the supply line, above equation
reduces to,
(m
2
m
1
) h
p
= m
2
u
2
m
1
u
1
+ W
The suction occurs at constant pressure i.e., ( )
1 2
2
1
V V p pdV W
= p (m
2
v
2
m
1
v
1
)
= m
2
(p
2
v
2
p
1
v
1
)
Substituting in the above equation for W, we have
(m
2
m
1
) h
p
= m
2
u
2
m
1
u
1
+ m
2
(p
2
v
2
p
1
v
1
)
= m
2
(u
2
+ p
2
v
2
) m
1
(u
1
+ p
1
v
1
)
= m
2
h
2
m
1
h
1
Or (m
2
m
1
) C
p
T
p
= m
2
C
p
T
2
m
1
C
p
T
1
Or (m
2
m
1
) T
p
= m
2
T
2
m
1
T
1
Substitute the given data, we get
(16 x 10
-4
1 x 10
-4
) 290 = 16 x 10
-4
x T
2
1 x 10
-4
x 800
T
2
= 321.87 K (Temperature of gas at the end of suction stroke)

5. An air receiver of volume 6 m
3
contains air at 15 bar and 40.5
0
C. A valve is
opened and some air is allowed to blow out to atmosphere. The pressure of the air in
the receiver drops rapidly to 12 bar when the valve is then closed. Calculate the mass
of air which has left the receiver.
Solution: The situation correspond to emptying process for which the energy balance
equation is,
( )
2 2 1 1
2
2 1
2
u m u m
V
h m m Q
p
p

1
]
1

+ +
But Q = 0, V
p
= 0
i.e., (m
1
- m
2
) h
p
= m
1
u
1
m
2
u
2
Or (m
1
m
2
) C
p
T
p
= m
1
C
V
T
1
m
2
C
V
T
2
From perfect gas equation, p
1
V
1
= m
1
RT
1
R
V p
T m Similarly
R
V p
T m e i
2
2 2
1
1 1
, ., .
Above equation becomes,
( ) ( )
2 1 2 1
. p p
R
V
T C
C
m m
p p
V


i.e., Mass of air left the receiver,
( ) ( )
2 1 2 1
.
.
1
p p
R
V
T
m m
p

( )
( )
5
10 12 15
287
6
.
5 . 313 4 . 1
1

72
= 14.3 kg
Also,
kg
RT
V p
m 03 . 100
1
1 1
1

m
2
= 85.74 kg
6. The internal energy of air is given, at ordinary temperature by, u = u
0
+ 0.718t.
Where u is in kJ/kg, u
0
is any arbitrary value of u at 0
0
C, kJ/kg and t is temperature in
0
C. Also for air, pv = 0.287 (t + 273) where p is in KPa and v is in m
3
/kg.
i) An evacuated bottle is fitted with a valve through which air from the
atmosphere, at 760 mm oxygen and 25
0
C, is allowed to flow slowly to fill the
bottle. If no heat transfer to or from the air in the bottle, what will its temperature
be when the pressure in the bottle reaches 760 mm Hg?
ii) If the bottle initially contains 0.03 m
3
of air at 400 mm Hg and 25
0
C, what will
the temperature be when the pressure in the bottle reaches 760 mm of Hg?
Solution: u = u
0
+ 0.718t
pv = 0.287 (t + 273)
Using above equation determine C
v
, C
p
i.e., C
v
= du/dt = 0.718 kJ/kg -
0
k
( ) pv u
dt
d
dt
dh
C
p
+ &
= 1.005 kJ/kg-
0
k
i) Situation corresponds to filling process, for which the energy balance equation is,
( )
1 1 2 2
2
1 2
2
u m u m
V
h m m Q
p
p

1
1
]
1

+ +
Heat transfer Q = 0, evacuated bottle, m
1
= 0 and negligible KE, V
p
= 0
m
2
h
p
= m
2
u
2
Or h
p
= u
2

i.e., C
p
T
p
= C
V
T
2
p
V
p
T
C
C
T
2
= T
p
= 1.4 (298)
= 417.12
0
K or 144.12
0
C
ii) Energy balance equation is, (m
2
m
1
) h
p
= m
2
u
2
m
1
u
1
i.e., (m
2
m
1
) C
p
T
p
= m
2
C
V
T
2
m
1
C
V
T
1
Using perfect gas equation, p
1
V
1
= m
1
RT
1
But p
1
= h
= 9810 (13.6) (400 x 10
-3
)
= 53366.4 N/m
2
( )
( )
kg m 0187 . 0
298 287
03 . 0 4 . 53366
1

Similarly p
2
V
2
= m
2
RT
2
P
2
= h = 9810 (13.6) (760 x 10
-3
)
= 101396.2 N/m
2
73
( )
( )
2
2 1
2
2
59 . 10
287
03 . 0 2 . 101396
T
V V
T
m



Substituting in energy balance equation,
( ) ( ) ( ) ( ) 298 718 0187 . 0 718
59 . 10
298 1005 0187 . 0
59 . 10
2
2 2

,
_

T
T T
Solving, T
2
= 344.92
0
K or 71.92
0
C
7. The internal energy of air is given by u = u
0
+ 0.718 t also for air, pv = 0.287 (t +
273). A mass of air is stirred by a paddle wheel in an insulated constant volume tank.
The velocities due to stirring make a negligible contribution to the internal energy of
the air. Air flows out through a small valve in the tank at a rate controlled to keep the
temperature in the tank constant. At a certain instant the conditions are as follows:
tank volume 0.12m
3
, pressure 1 MPa, temperature 150
0
C and power to paddle
wheel 0.1 kW. Find the rate of flow of air out of the tank at this instant.
Solution: The energy balance equation for emptying process is,
Q W
s
+ (m
1
m
2
) 2 2 1 1
2
2
u m u m
V
h
p
p

,
_

+
Insulated tank, Q = 0, there is no change in i.e., of air in the tank. Internal energy, u = 0
and
KE = 0
Above equation becomes,
- W
s
+ (m
1
m
2
) h
p
= 0
i.e., (m
1
m
2
) = W
s
/h
p
Or rate of flow of air out of the tank at this instant
( ) 425 005 . 1
1 . 0

p p p
T C h
= 2.3412 x 10
-4
kg/s
= 0.843 kg/hr
8. A certain water heater operates under steady flow conditions receiving 4.2 kg/s of
water at 75
0
C temperature, enthalpy 314 kJ/kg. The water is heated by mixing with
steam which is supplied to the heater at temperature 100
0
C and enthalpy 2676 kJ/kg.
The mixture leaves the heater as liquid water at temperature 100
0
C and enthalpy 419
kJ/kg. How much steam must be supplied to the heater per hour.
Solution: Energy entering = energy leaving
1
]
1

+ + +
1
]
1

+ + +
2
2
2
2 2 1
2
1
1 1
2 2
gZ
V
h m gZ
V
h m Q

W gZ
V
h m

+
1
]
1

+ +
3
2
3
3 3
2
By the nature of the process, 0 W

KE = 0 PE = 0
0 Q

74
i.e., 3 3 2 2 1 1
h m h m h m +
( ) ( ) ( ) ( ) 419 2 . 4 2676 314 2 . 4
2 2
m m + +
s kg m / 196 . 0
2

= 705 kg/hr
9. In a water cooling tower air enters at a height of 1 m above the ground level with
velocity of 20 m/s and leaves the tower at a height of 7 m above the ground level with
a velocity of 30 m/s. Water enters the tower at a height of 8 m above the ground level
with a velocity of 3 m/s and leaves the tower at a height of 0.8 m with a velocity of 1
m/s. Water temperatures at inlet and exit are 80
0
C and 50
0
C respectively, while the
inlet and exit temperatures of air are 30
0
C and 70
0
C respectively. The cooling tower is
well insulated and a of 2.25 kW drives the air through the cooling tower. Determine
the mass flow rate of air required if the mass flow rate of water is 1.5 kg/s.
Assume that for air C
p
= 1.005 kJ/kg-
0
K and for water C = 4.187 kJ/kg-
0
K.


75
Real Gases
Introduction
An ideal gas is a gas having no forces of intermolecular attraction. The gases which
follow the gas laws at all range of pressures and temperatures are considered as ideal
gases. An ideal gas obeys the perfect gas equation Pv = RT and has constant specific
heat capacities.
A real gas is a gas having forces of inter molecular attraction. At very low pressure
relative to the critical pressure or at very high temperatures relative to the critical
temperature, real gases behave nearly the same way as a perfect gas. But since at high
pressure or at low temperatures the deviation of real gases from the perfect gas relation is
appreciable, these conditions must be observed carefully, otherwise errors are likely to
result from inappropriate application of the perfect gas laws.
Due to these facts, numerous equations of state for real gas have been developed, the
derivation of which is either analytical, based on the kinetic theory of gases, or empirical,
derived from an experimental data.
76
Vander Waals Equation of State:
In deriving the equation of state for perfect gases it is assumed that the volume occupied
by the molecules of the gas in comparison to the volume occupied by the gas and the
force of attraction between the adjacent molecules is very small and hence the molecules
of gas are neglected. At low pressures, where the mean free path is large compared to the
size of the molecules, these assumptions are quite reasonable. But at high pressure, where
the molecules come close to each other, these are far from correct. Vander waals
equation introduces terms to take into account of these two modifying factors into the
equation of state for a perfect gas.
The Vander Waals equation of state is given by,
( ) RT b v
v
a
P
1
]
1

+
2
--- (1)
2
v
a
b v
RT
P or

P = Pressure v = Volume/unit mass R = gas constant


where a and b are constants for any one gas, which can be determined experimentally, the
constants account for the intermolecular attractions and finite size of the molecules which
were assumed to be non-existent in an ideal gas. The term
2
v
a
accounts for the
intermolecular forces i.e, force of cohesion and the term b was introduced to account for
the volume occupied by the molecules i.e., co-volume.
If the volume of one mole is considered, then the above equation can be written as,
( ) T R b v
v
a
P
1
]
1

+
2
Units P (N/m
2
), ( ) mole kg m v /
3
K mol kg m bar R Or K mol kg Nm R
0 3 0
/ 083143 . 0 / 8314
( )
2 4
/ ( mol kg Nm a
mol kg m b /
3
Determination of Van der Waals constants in terms of critical properties
The determination of two constants a and b in the Van der Waals equation is based on the
fact that the critical isotherm on a p-v diagram has a horizontal inflexion point at the
critical point. Therefore the first and second derivative of P with respect v at the critical
point must be zero.
0 0 ., .
2
2

,
_


,
_

c
c T T
T T
v
p
and
v
p
e i

From equation (1) we have,
2
v
a
b v
RT
P


77
( )
3 2
2
v
a
b v
RT
v
p
T
+


,
_



( )
4 3 2
2
6 2
v
a
b v
RT
v
p
and
T

,
_

At critical points the above equation reduces to


( )
0
2
3 2
+

v
a
b v
RT
--- (2a)
( )
0
6 2
4 3

v
a
b v
RT
and
--- (2b)
Also from equation (1) we have,
2
v
a
b v
RT
P
c
c

--- (2c)
Dividing equation (2a) by equation (2b) and simplifying we get
3
c
v
b
Substituting for b and solving for a from equation (2b) we get
a = 9RT
c
v
c
Substituting these expressions for a and b in equation (2c) and solving for v
c
, we get
c
c
c
p
RT
v
8
3


c
c
p
RT
b
8


c
c
p
T R
a and
2 2
64
27

,
_

If the volume of one mole is considered then the above equation can be written as
( ) T R b v
v
a
P
,
_

+
2
Units: P (N/m
2
), ( ) mol kg m v /
3
R =8314 Nm/kg mol
0
K
K mole kg
m
bar R

3
083143 . 0
a = Nm
4
/ (kgmol)
2
Or bar (m
3
/kg-mole)
2
b = m
3
/kg-mol
Note: Usually constants a and b for different gases are given.
Compressibility Factor and Compressibility Chart:
78
The specific volume of a gas becomes very large when the pressure is low or temperature
is high. Thus it is not possible to conveniently represent the behaviour of real gases at
low pressure and high temperature.
For a perfect gas, the equation of state is Pv = RT. But, for a real gas, a correction factor
has to be introduced in the perfect gas to take into account the deviation of the real gas
from the perfect gas equation. This factor is known as the compressibility factor, Z and is
defined as,
RT
Pv
Z
Z = 1 for a perfect gas. For real gases the value of Z is finite and it may be less or more
than unity depending on the temperature and pressure of the gas.
Reduced Properties:
The real gases follow closely the ideal gas equation only at low pressures and high
temperatures. The pressures and temperatures depend on the critical pressure and critical
temperature of the real gases. For example 100
0
C is a low temperature for most of the
gases, but not for air or nitrogen. Air or nitrogen can be treated as ideal gas at this
temperature and atmospheric pressure with an error which is <1%. This is because
nitrogen is well over its critical temperature of -147
0
C and away from the saturation
region. At this temperature and pressure most of the substances would exist in solid
phase. Hence, the pressure and temperature of a substance is high or low relative to its
critical pressure or temperature.
Gases behave differently at a given pressure and temperature, but they behave very much
the same at temperatures and pressures normalized with respect to their critical
temperatures and pressures. The ratios of pressure, temperature and specific volume of a
real gas to the corresponding critical values are called the reduced properties.
c
R
c
R
c
R
v
v
v
T
T
T
P
P
P e i & , ., .
Law of Corresponding states:
This law is used in the approximate determination of the properties of real gases when
their properties at the critical state are known. According to this law, there is a functional
relationship for all substances, which may be expressed mathematically as v
R
= f (P
R
,T
R
).
From this law it is clear that if any two gases have equal values of reduced pressure and
reduced temperature, they will have the same value of reduced volume. This law is most
accurate in the vicinity of the critical point.
Generalized Compressibility Chart:
The compressibility factor of any gas is a function of only two properties, usually
temperature and pressure so that Z
1
= f (T
R
, P
R
) except near the critical point. This is the
basis for the generalized compressibility chart.
The generalized compressibility chart is plotted with Z versus P
R
for various values of T
R
.
This is constructed by plotting the known data of one or more gases and can be used for
any gas.
79
It may be seen from the chart that the value of the compressibility factor at the critical
state is about 0.25. Note that the value of Z obtained from Van der waals equation of
state at the critical point,
8
3

c
c c
c
RT
v P
Z
which is higher than the actual value.
The following observations can be made from the generalized compressibility chart:
At very low pressures (P
R
<<1), the gases behave as an ideal gas regardless of
temperature.
At high temperature (T
R
> 2), ideal gas behaviour can be assumed with good
accuracy regardless of pressure except when (P
R
>> 1).
The deviation of a gas from ideal gas behaviour is greatest in the vicinity of the
critical point.
The compressibility factor can also be obtained from v-T or v-P data. Since the critical
volume may not be consistent with the generalized chart, the pseudo critical specific
volume v
c
1
is used in the definition of reduced volume. It is defined by
c c c
P RT v /
1

. The
pseudo reduced volume
1
R
v is defined as
c
c
c
R
RT
vP
v
v
v
1
1
80
Table: Critical Point Data of Gases
Gas Chemical
Formula
Pressure
P
c
bar
Temperatur
e
T
c
K
Specific Volume
mole kg m v
c
/
3
Air --- 37.7 132.8 0.083
Ammonia NH
3
112.8 405.8 0.073
Argon A 44.7 151.0 0.075
Carbon dioxide CO
2
73.8 304.2 0.094
Carbon monoxide CO 35.0 133.0 0.093
Freon-12 CCl
2
F
2
40.3 385.0 0.215
Helium He 2.3 5.2 0.058
Hydrogen H
2
13.0 33.2 0.065
Nitrogen N
2
33.9 126.1 0.090
Oxygen O
2
50.6 154.5 0.075
Sulphurdioxide SO
2
78.8 430.5 0.0123
Steam H
2
O 220.9 647.3 0.056
Table: Van Der Waals Constant
Gas a
kN m
4
/(kg-mol)
2
b
m
3
/kg-mol
Z = pv/RT
Air 135.8 0.0365 0.284
Oxygen 138.0 0.0318 0.29
Nitrogen 136.7 0.0386 0.291
Water 551.7 0.0304 0.23
Methane 228.6 0.0427 0.29
Carbon monoxide 147.9 0.0393 0.293
Carbon Dioxide 365.6 0.0428 0.276
Ammonia 424.9 0.0373 0.242
Hydrogen 24.8 0.0266 0.304
Helium 3.42 0.0235 0.30
Problems
1. A rigid vessel of volume 0.3 m
3
contains 10 kg of air at 300
0
K. Using (a) the
perfect gas equation, (b) the Vander Walls equation of state and (c) generalized
compressibility chart, determine the pressure which would be exerted by the air on
the vessel.
Solution: (a) The perfect gas equation is Pv = RT
Mv
T R
v
RT
P or
Specific volume of the gas =
kg m v / 03 . 0
10
3 . 0
3

( )
bar
x x
P 67 . 28
10 03 . 0 29
300 10 3143 . 8
5
3

(b) Vander Waals equation


( ) T R b v
v
a
P
,
_

+
2
81
We have
c
c
P
T R
a
2 2
64
27

,
_

From the critical point data of gases, we have T


c
= 132.8 K, P
c
= 37.7 bar
( ) ( )
7 . 37
8 . 132 083143 . 0
64
27
2 2

,
_

a
a = 1.364 bar (m
3
/kg-mole)
2

Also
7 . 37 8
8 . 132 083143 . 0
8 x
x
P
T R
b
c
c

= 0.0366m
3
/kg-mole
Substituting the constants a and b in the Van der Waals equation of state, we get
( )
( ) 300 083143 . 0 0366 . 0 87 . 0
87 . 0
364 . 1
2
x P
1
]
1

+
Noting v = 0.03 m
3
/kg, mole kg m x v / 87 . 0 29 03 . 0
3
( )
bar
x
P 13 . 28
87 . 0
364 . 1
0366 . 0 87 . 0
300 083143 . 0
2


We can also take the values of a and b from Thermodynamic Data Hand Book. [From
table C-8 we have a = 135.8 kN m
4
/(kg-mol)
2
and b = 0.0365 m
3
/kg-mol]
(c) The Pseudo reduced volume,
c
c
R
T R
P v
v
1
( )
( )
97 . 2
8 . 132 083143 . 0
7 . 37 87 . 0

The reduced temperature,
26 . 2
8 . 132
300

c
R
T
T
T
At v
R
1
= 2.97 & T
R
= 2.26 from compressibility chart, P
R
= 0.75
Also since P
c
= 37.7 bar,
The pressure exerted by the air on the cylinder P = P
R
x P
c
= 0.75 (37.7) = 28.27 bar
Or
The pressure can also calculated by reading the value of compressibility factor from the
chart.
At v
1
R
= 2.97 and T
R
= 2.26 or T
R
= 2.26 and P
R
= 0.75, from the compressibility chart Z
= 0.98
( )( )
( )
bar
v
ZRT
P 32 . 28
03 . 0 100
300 289 . 0 98 . 0

2. Determine the value of Z
cp
for the Vander Waals gas
Solution: From the isotherms plotted on P-v diagram it can be seen that the critical
isotherm has an inflection point, whose tangent is horizontal at the critical point.
0 & 0
2
2

cp
cp
c
v
P
v
P

Where c
p
= critical point.
82
We have Vander Waals equation at the critical point,
2
cp cp
cp
cp
v
a
b V
T R
P

--- (1)
As T
cp
is a constant,
( )
0
2
3 2

cp cp
cp
cp
cp
v
a
b v
T R
v
P
--- (2)
( )
0
6
2
4 3 2
2

cp cp
cp
cp
v
a
b v
T R
v
P
--- (3)
( ) ( )
0
2 3
, ) 3 ( ) 2 (
3
2 2

+
b v
T R
b v v
T R
gives equation xequation
v
cp cp cp
cp
( )
b v e i
b v v
or
cp
cp cp
3 ., .
2 3

Substitute for b in equation (2),


( ) [ ] ( )
0
2
3 / 1
3 2
+

cp cp cp
cp
v
a
v v
T R

cp cp
v T R a
,
_


8
9
Substitute for a & b in equation (1)
2
8
9
) 3 / 1 (
cp
cp cp
cp cp
cp
cp
v
v T R
v v
T R
P

,
_

1
8
9
3
2
1

cp
cp cp
T R
v P
As
8
9
2
3
,
cp cp
cp
cp cp
Z Z
T R
v P
8
3

cp
Z
3. If the values for the reduced pressure and compressibility factor for ethylene are 5
bar and 1.04 respectively, compute the temperature.
Solution: From generated computer chart (from chart 7 in thermodynamic Data Hand
Book compiled by B.T. Nijaguna and B.S. Samaga)
We find for P
R
= 5 & Z = 1.04, T
R
= 2.7
But
c
R
T
T
T
T = (282.4) (2.7) Since for ethylene T
c
= 282.4
0
K
= 762.48
0
K
4. Using the compressibility chart calculate (a) density of N
2
at 260 bar & 15
0
C. (b)
What should be the temperature of 1.4 kg of CO
2
gas in a container at a pressure of
200 bar to be have as an ideal gas.
Solution: For N
2
. from table C-6 (in thermodynamic Data Hand Book compiled by B.T.
Nijaguna and B.S. Samaga) T
c
= 125.9
0
K P
c
= 33.94
83
29 . 2
9 . 125
288
, 66 . 7
94 . 33
260

c
R
c
R
T
T
T
P
P
P
From chart 7, for P
R
= 7.66 & T
R
= 2.29, Z = 1.08
ZRT
P
e i
RT
P
RT
Pv
Z

., .
( )( ) 288 9 . 296 08 . 1
10 260
5
x

Since R = 296.9 J/kg-


0
K from table
C6
= 281.54 kg/m
3
(b) For CO
2
, from table C6, T
c
= 304
0
K, P
c
= 73.85 bar.
As the gas behaves like an ideal gas, Z = 1
71 . 2
85 . 73
200

R
P
From compressibility chart (chart 7), for Z = 1 & P
R
= 2.71, T
R
= 2.46
T = T
c
T
R
= 304 (2.46) = 747.84
0
K
5. Find the pressure exerted by methane in a container of capacity 2m
3
, when it
contains 3 kg at 300
0
K. Using (a) Ideal gas equation (b) Vander Waals equation.
Solution: Molecular weight of methane is M
cH4
= 16
mol kg m
M
m
v
n
v
v Volume Molar / 67 . 10
3
) 16 ( 2
,
3
Characteristic gas constant, K kg kJ
M
R
R
0
/ 5196 . 0
16
3143 . 8

(a) Using ideal gas equation
We have, Pv = mRT
( ) ( )
bar
x
P 34 . 2
10 2
300 10 5196 . 0 3
5
3

(b) We have
2
v
a
b v
T R
P

From table C-8, a = 228.6 & b = 0.0427


Substituting in the above equation
( )
bar or KPa P 32 . 2 7 . 232
67 . 10
6 . 228
0427 . 0 67 . 10
300 3143 . 8
2

6. Determine the mass of N


2
contained in a 30 m
3
vessel at 20 MPa and 200 K by
using (a) the ideal gas equation (b) the generalized comp. chart.
(a) Ideal gas equation Pv = mRT
( )
kg
x x
RT
Pv
m 07 . 10103
200 9 . 296
30 10 20
6

(b) From table C 6 for N
2
, P
c
= 33.94 bar T
c
= - 147.1
0
C = 125.9
0
K
589 . 0
94 . 33
20

c
R
P
P
P
84
59 . 1
9 . 125
200

R
R
T
T
T
For P
R
= 0.589 & T
R
= 1.59 from chart Z = 0.9
kg m
P
ZRT
v
RT
Pv
Z but / 00268 . 0
3

kg
v
V
m 03 . 11194
00268 . 0
30

7. The specific volume of CO
2
at 120
0
C is 1.2 m
3
/kg. Determine the pressure exerted
by CO
2
using Van der Waals equation. If CO
2
is treated as an ideal gas, what should
be the pressure exerted.
( )
,
_

mol kg m
n
v
v H / 8 . 52 44 2 . 1 : int
3
Ans: P = 61.8 KPa P
id
= 61.88 KPa
8. One kg mol of NH
3
undergoes a reversible non-flow isothermal compression
process and the volume decreases from 0.2 m
3
/kg to 0.1 m
3
/kg, the initial temperature
being 45
0
C. If the gas obeys Van-der-waals equation during the compression
process, determine the work done during the process and final pressure.
Solution: M
NH3
= 17 mol kg m v mol kg m v / 7 . 1 & , / 4 . 3 ) 17 ( 2 . 0
3
2
3
1

We have v Pd W
2
1 2 1

v d
v
a
b v
T R

'

,
_


2
2
1
1
]
1

+
1
]
1

,
_

1 2
2
1 1
ln
v v
a
b v
b v
T R
From tables, C 8, a = 424.9 KN-m
4
/(kg-mol)
2
b = 0.0373 m
3
/kg-mol
W
1-2
= - 1737.098 KNm/kg-mole (kJ/kg-mol)
Final pressure
KPa
v
a
b v
T R
P 1 . 1443
2
2 2
2

9. Determine the pressure of CO gas having volume of 0.003m


3
/kg at 164 K with the
use of generalized computer chart. Also determine if the volume of gas is reduced to
80% of the initial volume, what is the temperature of the gas at the same pressure?
Solution: From table C 6, for CO, T
c
= 134.3K, P
c
= 34.96 bar
(a) The Pseudo reduced volume,
266 . 0
) 8 . 134 ( 083143 . 0
) 96 . 34 )( 28 ( 003 . 0
1

c
c
R
T R
P v
v
From chart 7, for
6 . 2 & 59 . 0 , 234 . 1
3 . 134
164
266 . 0
1

R R R
P Z T v
Pressure, P = P
R
P
c
= 2.6 (34.96)
= 90.81 bar
(b) Volume is reduced to 80% of initial volume, v = 0.8 (0.003) = 0.0024m
3
/kg
85
213 . 0
) 3 . 134 ( 083143 . 0
) 96 . 34 )( 28 ( 0024 . 0
1

c
c
R
T R
P v
v

From chart 7, for v
R
1
& P
R
, Z 0.47, T
R
1.15
Final temperature is T = T
R
- T
c
= 152.86
0
K
10. Determine the compressibility factor for O
2
at (i) 100 bar, - 70
0
C and (ii) at 5 bar
and 30
0
C.
Ans: (i) 0.71 (ii) 0.98
11. Determine the pressure of air at 205
0
C having a specific volume of 0.00315 m
3
/kg
using (i) Ideal gas equation and (ii) Van der Waals equation.
Ans: (i) 435.7 bar (ii) 557.3 bar
86
Ideal Gas
Definition: A substance is said to be an ideal gas if it satisfies the following equations
i.e., Pv = RT and u = f (T)
Where P is the pressure exerted by the substance, v is the specific volume of the
substance, T is the temperature in degree Kelvin, u is the specific internal energy and R is
the gas constant. Experience has shown that almost all real gases satisfy the above
equations over wide ranges of pressures and temperatures. However there are certain
situations where the real gases cannot be treated as ideal gases.
Mole of a Gas
A mole of a gas is that quantity of gas whose mass is numerically equal to its molecular
weight. For example, 1 kg mol of hydrogen is equal to 2 kg, has molecular weight of
hydrogen is 2. Therefore if n is the total number of moles, m is mass and M is the
molecular weight then, nM=m
Avogadros Hypothesis
Avogadros law states that equal volumes of all gases measured at the same temperature
and pressure contain the same number of moles.
Consider two gases A and B. The law states that if V
A
= V
B
, T
A
= T
B
and P
A
= P
B
then n
A
= n
B
For gas A, the equation of state can be written as P
A
V
A
= m
A
R
A
T
A
= n
A
M
A
R
A
T
A
( )
A A A
A A
A
T R M
V P
n Or

Similarly for gas B we have
( )
B B B
B B
B
T R M
V P
n

According to Avogadros law, V
A
= V
B
, T
A
= T
B
and P
A
= P
B
then n
A
= n
B
. Therefore it
follows that
M
A
R
A
= M
B
R
B
= R
Where R is called the universal gas constant and R is called the characteristic gas
constant. The value of R =8.3143 kJ/kgmol-K.
The ideal gas equation can also be written in terms of R as pV = nMRT = n R T where
R = R/M
87
Specific Heat of Ideal Gases
From the definition of specific heat at constant volume and the specific heat at constant
pressure, we have
( )
( ) T f
dT
dh
c
T f
dT
du
c
P
v


as u and h are functions of temperature.
From the above equations, du = c
V
dT and dh = c
P
dT
For a mass of m kg of gas the equations become
dU = m c
V
dT and dH = m c
P
dT
On integrating we get


2
1
1 2
T
T
v
dT c m U U


2
1
1 2
T
T
P
dT c m H H and
Relation between specific heats for an ideal gas
For an ideal gas h = u + RT
Therefore
R
dT
du
dT
dh
+
,
_

Or c
P
= c
V
+ R
ie, c
P
- c
V
= R
Dividing the above equation by c
V
, we get
V
c
R
1
1

R
c Or
V
Similarly, dividing the above equation by c
P
, we get
1

R
c
P
Changes in internal energy, enthalpy and entropy for an ideal gas
i) Change in internal energy
Let an ideal gas of fixed mass m undergoes a finite change of state from temperature T
1
to temperature T
2
. Then the change in internal energy is given by
( )


2
1
1 2 1 2
du m u u m U U

( )

2
1
T
T
V
dT c m
To integrate the above equation we should know the functional relationship between c
V
and dT. A perfect gas equation is one for which c
v
is a constant.
88
Therefore, U
2
-U
1
= m c
v
(T
2
T
1
)
ii) Change in Enthalpy
( )


2
1
1 2 1 2
dh m h h m H H have We
( )

2
1
T
T
P
dT c m
For a perfect gas c
P
is constant. Therefore the above equation can be integrated and we
get
H
2
-H
1
= m c
P
(T
2
T
1
)
Work done by a perfect gas during a reversible adiabatic process in a closed system:
From the first law of TD,
w du q +
For an adiabatic process, q=0
Hence the work done by a unit mass of a perfect gas on a piston during an adiabatic
expansion process is equal to the decrease in internal energy, i.e., w = -du
Whereas, for an adiabatic compression process, the internal energy of the gas will
increase with a consequent increase in temperature.
For a perfect gas, du=c
v
dT
dT c w
v

dT
R
w
R
c But
v
1 1
,

Hence the work done during an adiabatic non-flow process between states 1 and 2 is
given by
( )
2 1 2 1
1
T T
R
W


1
2 2 1 1

v P v P

1
]
1

1 1
2 2 1 1
1
1 v P
v P v P


1
1
1
]
1

,
_

1
2
1 1 1
1
1 P
P v P
Work done by a perfect gas during a reversible adiabatic Steady Flow process:
89
Neglecting the effect of changes in velocity and elevation, SFEE for a unit mass of fluid
is given by
h w q
2 1 2 1
Since the process is adiabatic, q=0. Hence work done per unit mass of a perfect gas
during an adiabatic steady flow expansion process is equal to the decrease in enthalpy,
i.e.,w
1-2
= h
1
-h
2
For a perfect gas, dh = c
p
dT
Therefore, w
1-2
= c
p
(T
1
-T
2
)
,
1
,

R
c But
p
the work done during an adiabatic steady flow process between states 1 and 2 is given by,
( )
2 1 2 1
1
T T
R
w


( )
2 2 1 1
1
v P v P


1
]
1

1 1
2 2 1 1
1
1 v P
v P v P


1
1
1
]
1

,
_

1
1
2 1 1
1
1 P
P v P
It may be noted that the work done for a steady flow system is times that for a closed
system.
Note: The above equations may be derived for reversible processes by taking integral p
dv for closed system and taking integral of minus v dp for a steady flow system with
negligible changes in the kinetic and potential energies from the initial to the final state.
The polytropic process of a perfect gas: A Polytropic process is one for which the
pressure volume relation is given by pv
n
= constant, where the exponent n for the given
process is a constant and may have any numerical value ranging from plus infinity to
minus infinity.
From the above equation, it is evident that the properties at the end states of the reversible
or irreversible polytropic process of a perfect gas may be written in the form
1
1
2
1
1
2
1
2
2
1
1
2

,
_

,
_

,
_

n
n
n
n
v
v
P
P
T
T
v
v
P
P
There are four values of the exponent n that indicate processes of particular interest.
When
n = 0, constant pressure or isobaric process
n = , constant volume or isovolumic process
n = 1, constant temperature or isothermal process and
n = , constant entropy or isentropic process
90
These processes are shown in the fig. on p-v and T-s diagrams.
Figure: Polytropic Process on p-v and T-s diagrams
Work done and heat transfer by a perfect gas during a polytropic process:
For the closed system, the work done during a polytropic expansion process is given
by,

2
1
2 1 2 1
) (
1
T T
n
R
pdv w

1
2 2 1 1

n
v P v P

1
1
1
]
1

,
_

n
n
P
P
n
v P
1
1
2 1 1
1
1
The work done for a steady flow system during a polytropic expansion process is
given by,
( )

2
1
2 1 2 1
1
T T
n
nR
vdp w

( )
2 2 1 1
1
v P v P
n
n


1
1
1
]
1

,
_

n
n
P
P
n
v P n
1
1
2 1 1
1
1
It may be noted that the work done for a steady flow system is n times that for a closed
system.
i) The heat transfer for a closed system:
From the first law of thermodynamics for a unit mass of substance
q = w + du
Since du = c
v
dT and for a reversible process w = pdv
q = c
v
dT + pdv
91
Therefore heat transfer per unit mass during a polytropic process in a closed system from
the initial state to final state is given by
( )

+

2
1
1 2 2 1
pdv T T c q
v
( )

2
1
2 1 2 1
1
T T
n
R
pdv w But
( ) ( )
2 1 1 2 2 1
1
T T
n
R
T T c q
v

+

( )
,
_


v
c
n
R
T T
1
2 1
( )

,
_


1
2 1
n
nc c
T T
v p
( )
( )
v
c
n
T T
1
1
2 1


( )
1 1
2 1

,
_

n
T T R n

2 1 2 1
1

,
_

w
n
q Or

ii) The heat transfer in a steady flow process:


From the first law for steady flow system for a unit mass of fluid
q = w + dh
But dh = c
p
dT and for a reversible steady flow process w = -vdp
Therefore q = c
p
dT - vdp
( )

2
1
1 2 2 1
vdp T T c q
p
( )
2 1
2
1
2 1
1
T T
n
nR
vdp w But

Therefore heat transfer,


( ) ( )
2 1 1 2 2 1
1
T T
n
nR
T T c q
p

( )
,
_


p
c
n
nR
T T
1
2 1
( )
1
2 1


n
nc c
T T
v p
( )
2 1
1
T T c
n
n
v

,
_

( )
1 1
2 1

,
_

n
T T
R
n
n
n
w
n
n
2 1
1

,
_


Change in Entropy
Let P
1
, v
1
, T
1
, S
1
apply to the initial conditions of certain amount of gas. P
2
, v
2
, T
2
, S
2
,
apply to the final conditions after adding some heat.
92
From first law of TD, q = w + du
q = P.dv + du
Also by definition, du = c
v
dT
Therefore q = P.dv + c
v
dT
Divide by T,
T
dT
c
T
dv
P
T
q
v
+


dS
T
q
But

T
dT
c dv
T
P
dS
v
+
From perfect gas equation for unit mass of gas, Pv = RT, therefore P/T = R/v
T
dT
c dv
v
R
dS
v
+
Therefore integrating,

+
2
1
2
1
2
1
T
dT
c
v
dv
R dS
v
i.e., the change in entropy is given by
1
2
1
2
1 2
ln ln
T
T
c
v
v
R S S
v
+
--- (1)
Equation (1) can also be express in terms of pressure and volume.
We have
2
2 2
1
1 1
T
v P
T
v P

1 1
2 2
1
2
v P
v P
T
T

Substituting this in equation (1) we get,
( )
1
2
1
2
1 1
ln ln ., .
P
P
c
v
v
R c S S e i
v v
+ +

But c
P
c
v
= R, i.e., c
v
+ R = c
P

1
2
1
2
1 1
ln ln
P
P
c
v
v
c S S
v P
+
--- (2)
Equation (1) can also be represented in terms of temperature and volume.
We have
2
2 2
1
1 1
T
v P
T
v P

93
1 1
2 2
1
2
T P
T P
v
v

Substituting this in equation (1) we get
1
2
1
2
2
1
1 2
ln ln .
T
T
c
T
T
P
P
R S S
v
+
1
2
1
2
1 2
ln . ln
P
P
R
T
T
c S S Or
P

--- (3)
Evaluation of change in entropy in various quasi static processes
1. Constant Volume Process
We have Q = W + dU
W = 0 for a constant volume process.
Therefore heat added, Q = dU = m c
v
dT
Divide by T
T
dT
mc
T
Q
v


Integrating,

2
1
2
1
T
dT
mc
T
Q
v


1
2
1 2
ln .
T
T
mc S S
v

2. Constant Pressure Process
Q = W + dU
= P.dv + dU
= m R dT + mc
v
dT
= m (R + c
v
) dT
= m (c
P
c
v
+ c
v
)dT
= mc
P
dT
Divide by T,
T
dT
mc
T
Q
P


Integrating,

2
1
2
1
T
dT
mc
T
Q
P


1
2
1 2
ln .
T
T
mc S S
P


3. Isothermal Process
We have Q = W + dU
But dU = 0
1
2
1 1
ln
v
v
v P Q

94
Divide by T,
1
2
1
1
ln
v
v
T
mRT
T
Q

Integrating we get,
1
2
1 2
ln
v
v
mR S S
4. Reversible Adiabatic Process
Q = 0
Therefore S
2
S
1
= 0
Hence the process is called isentropic process

5. Polytropic Process
We know that for a perfect gas
dv P
n
Q .
1

Divide by T,
T
dv P n
T
Q .
1


2
1
2
1
1 T
dv
P
n
T
Q


We know Pv = RT, i.e., P = RT/v


2
1
1 2
1 T
dv
v
RT n
S S

2
1
1 v
dv
R
n

1
2
1 2
ln .
1 v
v
R
n
S S

Show that the entropy change for an ideal gas undergoing a polytropic process
according to the equation Pv
n
= c is given by
( )( )
1
2
1 2
ln .
1 1 T
T
R
n
n
S S

We have the change in entropy for unit mass of a substance between states (1) and (2) is
given by


2
1
2
1
1 2
T
Q
dS S S

For a perfect gas, we know that
dv P
n
Q .
1

Divide by T and integrating,




2
1
2
1
1 2
.
1 T
dv P n
S S
T
Q

2
1
.
1
1
dv p
T
n

95
( )
1
1
1
2 1

n
T T R
T
n

( )

,
_

1 1
.
2 1 2 2 1 1
2 1
2
1
n
T T R
n
v P v P
W dv P

1
]
1

1 1
1 2
n
T T
T
R n

( )( )

2
1
1 1
dT
T
R
n
n

( )( )

2
1
1 1 T
dT
R
n
n

( )( )
1
2
1 2
ln
1 1 T
T
R
n
n
S S

96
Semi-perfect gas:
It can be observed that from the definition of c
p
and c
v
that the specific heats can be either
constants or functions of temperature.
A semi-perfect gas is one which follows the ideal gas relation with its specific heats
being functions of temperature.
i.e., Pv = RT
And c
p
= (T)
c
v
= f(T)
For example the constant pressure molal specific heat of air at low pressure is related to
the temperature by the empirical relation
2 6 3
10 9 . 0 10 16 . 6 43 . 27 T x T x c
p

+

Where p
c
is in kJ/kg-mole K and T is in Kelvin.
It can be seen that integration of specific heat equation is time consuming. Hence Keenan
and Kaye developed the gas tables (Table C-21) to take into account the variation of
specific heats with temperature. Internal energy and enthalpy of various gases including
air at low pressure for wide range of temperature are tabulated in these tables. The table
illustrates the properties of air taking into account the variation of specific heats with
temperature.
Gas table may also be used for isentropic processes of perfect gases to relate properties
by introducing a relative pressure p
r
and a relative volume v
r
.
The entropy at a reference state where the temperature is T
0
and the pressure is 1 bar is
assumed as zero. Therefore at a temperature T and pressure p, the entropy s is given by
p R
T
dT
c s
T
T
p
ln
0


In gas tables an entropy function s
0
is defined as
T
dT
c s
T
T
p

0
0
Since c
p
is a function of temperature, s
0
is also a function of temperature for a perfect gas.
From the above equations we get s = s
0
R ln p
Therefore the change entropy of a perfect gas between states (1) and (2) can be written as
( )

,
_


1
2 0
1
0
2 1 2
ln
p
p
R s s s s
For an isentropic process, s = 0
Therefore
) ( ln
0
1
0
2
1
2
T f
R
s s
p
p
s

,
_

A relative pressure p
r
is defined a ratio of the pressure p to the reference pressure p
0
.
97
0
., .
p
p
p e i
r

Therefore from the above equation we can write,
) (
ln
/
/
ln ln
1
2
1
2
1
2
0 1
0 2
1
2
T f
p
p
p
p
p
p
p p
p p
p
p
r
r
r
r
s

,
_

,
_

,
_

,
_

Further from the equation state of perfect gas, we have


1 2
2 1
1 2
2 1
1
2
T p
T p
T p
T p
v
v
r
r

The relative volume v
r
is defined as
r
r
p
RT
v
Using this equation in the above equation we get
) (
1
2
1
2
T f
v
v
v
v
r
r
s

,
_

,
_

The values of relative pressure p


r
and relative volume v
r
over a wide range of temperature
are tabulated in gas tables.

Problems
1. 1.25 m
3
of air at 180
0
C at 8 bar is undergoing a constant pressure until the volume
is doubled. Determine the change in the entropy and enthalpy of air.
Solution: Assuming air behaves like a perfect gas we have, C
P
= 1.005 kJ/kg
0
K,
C
v
= 0.718 kJ/kg
0
K and R = 0.287 kJ/kg
0
K
Given: V
1
= 1.25 m
3
, T
1
= 180 + 273 = 453
0
K, P
1
= P
2
= 8 x 10
5
N/m
2
and V
2
= 2V
1
For a constant pressure process, change in entropy is

,
_


1
2
1 2
ln
T
T
mC S S
P
We have P
1
V
1
= mRT
1
kg
x
x x
m 69 . 7
453 287
25 . 1 10 8
5

2
2 2
1
1 1
T
V P
T
V P
Also
K
x x
T
0
2
906
25 . 1
453 25 . 1 2

Therefore change in entropy =
453
906
ln 005 . 1 69 . 7 x
= 5.596 kJ/
0
K
Change in enthalpy = Q
1-2
= mC
P
(T
2
T
1
) = 3500.99 kJ
2. 1kg of air initially at 27
0
C is heated reversibly at constant pressure until the
volume is doubled, and then is heated at constant volume until the pressure is
98
doubled. For the total path find i) Work transfer, ii) Heat transfer, iii) Change in
entropy
Solution: Given: m = 1 kg, T
1
= 300
0
K V
2
= 2V
1
, P
3
= 2P
2
= 2P
1

Process 1-2: Constant pressure process
i) Work done, W
1-2
= P (V
2
V
1
)
= PV
2
PV
1
= mR (T
2
T
1
)
2
2 2
1
1 1
T
V P
T
V P
Also
But P
1
= P
2
1 2
2
2
1
1
2V V But
T
V
T
V

Therefore T
2
= 2T
1
= 600
0
K
Therefore work done W
1-2
= 1 x 0.287 x (600 300)
= 86.1 kJ
ii) From first law of TD, Heat Transfer, Q
1-2
= W
1-2
+ (U
2
U
1
)
= W
1-2
+ mC
v
(T
2
T
1
)
= 86.1 + 1 x 0.718 x (600 300)
= 301.5 kJ
iii) Change in entropy,
1
2
1 2
ln
T
T
mC S S
P

300
600
ln ) 005 . 1 ( 1
= 0.6966 kJ/
0
K
Process 2-3: Constant Volume Process
Given, P
3
= 2P
2
, T
2
= 600 K
3 2
3
3 3
2
2 2
V V But
T
V P
T
V P
have We
2
2 2
2
3 2
3
2
P
P x T
P
P T
T
= 2 x T
2
= 1200
0
K
i) Work done, W
2-3
= 0
ii) Heat transfer, Q
2-3
= W
2-3
+ (U
3
U
2
)
= mC
v
(T
3
T
2
)
99
V
P
3
2
1
= 430.8 kJ
iii) Change in entropy
2
3
2 3
ln
T
T
mC S S
P

= 0.4977 kJ/
0
K
Therefore work transfer in total path, W
1-3
= W
1-2
+ W
2-3

= 86.1 + 0
= 86.1 kJ
Heat transfer in total path, Q
1-3
= Q
1-2
+ Q
2-3
= 301.5 + 430.8
= 732.3 kJ
Change in entropy for the total path = (S
3
S
1
) = (S
3
S
2
) + (S
2
S
1
)
= 0.4977 + 0.6966
= 1.1943 kJ/
0
K
3. A mass of air is initially at 260
0
C and 700 kPa, and occupies 0.028m
3
. The air is
expanded at constant pressure to 0.084m
3
. A polytropic process with n = 1.5 is then
carried out, followed by a constant temperature process which completes a cycle. All
the processes are reversible. (i) sketch the cycle in the P-v and T-s diagrams. (ii) find
the heat received and heat rejected in the cycle. (iii) find the efficiency of the cycle.
Solution: P
1
= 700 kPa, T
1
= 533K = T
3
, V
1
= 0.028m
3
, V
2
= 0.084m
3
We have P
1
V
1
= mRT
1

kg
x
x
m 128 . 0
533 287 . 0
028 . 0 700


3
027 . 0
084 . 0
1 1
2 2
1
2

V P
V P
T
T
Now
Therefore T
2
= 3 x 533 = 1559 K
27 ) 3 (
533
1599
3
5 . 0
5 . 1
1
3
2
3
2

,
_

,
_

n
n
T
T
P
P
Again
Heat transfer in process 1-2, Q
1-2
= mc
p
(T
2
T
1
)
= 0.128 x 1.005 (1599 533)
= 137.13kJ
Heat transfer in process 2 -3,
pdv U Q +
3 2
100
( )
( )
1
3 2
2 3

+
n
T T mR
T T mc
v
( )
2 3
1
T T
n
n
mc
v


On substituting Q
2-3
= - 19.59 kJ
For process 3-1
Q
3-1
= dU + W
3-1

But dU = 0, i.e.,



1
3 1
3
1
3
1
1 1 3 1 3
ln ln
P
P
mRT
v
v
mRT pdv W Q
On substituting, Q
3-1
= - 64.53 kJ
Heat received in the cycle 137.13 kJ
Heat rejected in the cycle Q
2
= 19.59 + 64.53 = 84.12 kJ
The efficiency of the cycle
% 39 39 . 0
13 . 137
12 . 84
1 1
1
2
or
Q
Q
cycle


4. 1 kg of air at a pressure of 7 bar and a temperature of 90
0
C undergoes a reversible
polytropic process which may be represented by PV
1.1
= C, final pressure is 1.4 bar.
Evaluate i) The final specific volume, temperature and increase in entropy, ii) Work
done and heat transfer.
Solution: Given, m = 1 kg, P
1
= 7 bar, T
1
= 363
0
K, PV
1.1
= C, P
2
= 1.4 bar
Air is perfect gas i.e., P
1
V
1
= mRT
1
kg m
x
v / 14883 . 0
10 7
) 363 )( 287 ( 1
3
5
1

Also we have, P
1
V
1
1.1
= P
2
V
2
1.1

Therefore V
2
= 0.6429 m
3
Also P
2
V
2
= mRT
2
Therefore T
2
= 313.61
0
K
Change in entropy for a polytropic process is,

,
_

,
_


1
2
1 2
ln .
1 V
V
R
n
S S

Substituting the values, noting = 1.4 and R = 0.287 kJ/kg


0
K, we get
S
2
S
1
= 0.31495kJ/kg
0
K
Work done,
1
2 2 1 1
2 1

n
V P V P
W
Substituting we get, W
1-2
= 141.75 kJ/kg
101
Heat transfer Q
1-2
= W
1-2
+ (U
2
U
1
)
= W
1-2
+ mC
v
(T
2
T
1
)
= 141.75 35.462
= 106.29 kJ/kg
5. Show that for a reversible adiabatic process the equation is Pv

= Constant.
Solution: The general property relations for an ideal gas may be written as
Tds = du + pdv
= c
v
dT + pdv
And, also Tds = dh vdp
= c
p
dT vdp
For a reversible adiabatic change, ds = 0
Therefore c
v
dT = - pdv
And c
p
dT = vdp
By division,
pdv
vdp
c
c
v
p


0 +
v
dv
p
dp
Or
Or d (ln p) + d (ln v) = d (ln c)where c is the constant
Therefore ln p + ln v = ln c
i.e., pv

= constant

6. The cooking gas cylinder (mostly methane) is about 250mm in diameter and
800mm in height. It is charged to 12 MPa at room temperature of 27
0
C. i) Assuming
ideal gas law, find the mass of the gas filled in the cylinder, ii) If the cylinder is to be
protected against excessive pressure by means of a fusible plug, at what temperature
the plug has to melt to limit the maximum pressure to 15 MPa? Assume the molecular
weight of methane to be 16kg/kg-mole.
Solution: Volume of the cylinder=V=
3
2
0393 . 0 8 . 0
4
25 . 0
m x
x

, M=16, T=300
0
K,
P=12x10
6
N/m
2
, K kg kJ
M
R
R / 5183 . 0
16
3143 . 8
kg
x x
x x
RT
PV
m 033 . 3
10 300 5183 . 0
0393 . 0 10 12
3
6

When the pressure is limited to 15 MPa the corresponding temperature is
C K
x x
x x
mR
PV
T
0
3
6
102 375
10 5183 . 0 033 . 3
0393 . 0 10 15

7. Show that for an ideal gas, the slope of the constant volume line on the T-s
diagram is more than that of the constant pressure line.
Solution: We have for 1 kg of ideal gas,
Tds = du + pdv
= c
v
dT + pdv
102
v v
c
T
s
T

,
_

Also Tds = dh vdp


= c
p
dT vdp
p p
c
T
s
T

,
_

p v
p v
v p
s
T
s
T
c
T
c
T
c c But

,
_

>
,
_

> > ,
i.e., the slope of the constant volume line passing through a point is steeper than that of
the constant pressure line passing through same point.
8. A mass of 0.25 kg of an ideal gas has a pressure of 300 kPa, a temperature of
80
0
C, and a volume of 0.07 m
3
. The gas undergoes an irreversible adiabatic process to
a final pressure of 300 kPa and final volume of 0.1 m
3
during which the work done on
the gas is 25 kJ. Evaluate the c
p
and c
v
of the gas and increase in entropy of the gas.
Solution: We have, P
1
V
1
= mRT
1
kgK kJ
x
x
R / 238 . 0
353 25 . 0
07 . 0 300

Final temperature K
x
x
mR
V P
T 505
238 . 0 25 . 0
1 . 0 300
2 2
2

We have from first law of TD, Q W = U
2
U
1

But Q = 0 Therefore W = U
1
U
2
W = mc
v
(T
1
T
2
)
On substituting, -25 = 0.25 x c
v
x (353 505)
Therefore c
v
= 0.658 kJ/kg-K
Now R = c
p
c
v

Therefore c
p
= 0.896 kJ/kg-K
Entropy change
1
2
1
2
1 2
ln ln
V
V
mc
P
P
mc S S
p v
+
103
07 . 0
10 . 0
ln 896 . 0 25 . 0 ln
1
2
x
V
V
mc
p

= 0.08 kJ/kg-K
9. An ideal gas cycle consisting of three processes uses Argon (M = 40) as a
working substance. Process 1-2 is a reversible adiabatic expansion from 0.014m
3
, 700
kPa, 280
0
C to 0.056m
3
. Process 2-3 is a reversible isothermal process and process 3-1
is a constant pressure process. Sketch the cycle on P-v and T-s diagrams and
determine (i) work transfer in each of the three processes (ii) heat transfer in each of
the three processes and (iii) net work output from the cycle. Assume for Argon =
1.67.
Solution: M = 40, V
1
= 0.014m
3
, P
1
= 700 kPa, T
1
= 553 K
V
2
= 0.056m
3
, = 1.67
Figure P-v and T-s diagrams
Process 1-2 Reversible adiabatic process
K kg kJ
M
R
R / 208 . 0
40
3143 . 8

( )
K kg kJ
R
c
v

/ 311 . 0
1 67 . 1
208 . 0
1
c
p
= 0.208 + 0.311 = 0.519 kJ/kg-K
Since the process is reversible adiabatic, the working substance is treated as an ideal gas
kPa P or
x
V
V
P P or V P V P e i
13 . 69
056 . 0
014 . 0
700 ., .
2
67 . 1
2
1
1 2 2 2 1 1

,
_

,
_



K x
V
V
T T Also 45 . 218
056 . 0
014 . 0
553
67 . 0
1
2
1
1 2

,
_

,
_


( )
( )
( )
J
x x x x V P V P
W
1 67 . 1
056 . 0 10 13 . 69 014 . 0 10 700
1
3 3
1 2 1 1
2 1

= 8.85 x 10
3
J = 8.85 kJ
Q
1-2
= 0 as the process is reversible adiabatic.
Process 2-3 Reversible isothermal process
T
2
= T
3
= 218.45 K

,
_

,
_

,
_

1
2
2
3
2
2
2
3
2 3 2
ln ln ln
P
P
mRT
P
P
mRT
V
V
mRT W
104
kg
x x
x x
RT
V P
m But 2157 . 0
45 . 218 10 208 . 0
014 . 0 10 700
3
3
1
1 1

,
_



700
13 . 69
ln 45 . 218 10 208 . 0 2157 . 0
3
3 2
x x x x W
= - 22.69 x 10
3
J = - 22.69 kJ
By first law of TD Q
2-3
= (U
3
U
2
) + W
2-3

But U
3
U
2
= mc
v
(T
3
T
2
) = 0
Therefore Q
2-3
= W
2-3
= - 22.69 kJ
Process 3-1 Constant Pressure Process
W
3-1
= (P
1
V
1
P
3
V
3
) = mR (T
1
T
3
)
= 0.2157 x 0.208 x 10
3
(553 218.45) = 15 kJ
Therefore Net work output from the cycle = W
c
= W
1-2
+ W
2-3
+ W
3-1

= 8.85 22.69 + 5.93
= - 7.91 kJ
For the cyclic process we have
W Q Q Q or W Q + +
1 3 3 2 2 1

0 - 22.69 + Q
3-1
= - 7.91
Therefore Q
3-1
= 14.78 kJ
10. A mass of an ideal gas exists initially at 200 kPa, 300K and 0.5 m
3
/ kg. The value
of is 1.4. (i)Determine the specific heats of the gas. (ii)What is the change in
entropy when the gas is expanded to a pressure of 100 kPa according to the law pv
1.3
= constant (iii) What will be the change in entropy if the process is according to the
law pv
1.5
= constant (iv) What inference you can draw from this example.
Solution: P
1
= 200 kPa, T
1
= 300 K, v
1
= 0.5 m
3
/kg, =1.4, P
2
= 100 kPa
(i) For an ideal gas P
1
v
1
= RT
1
or
( )
K kg J
T
v P
R / 33 . 333
1
1 1

K kg J c c
K kg J
R
c
v p
v

/ 66 . 1166
/ 33 . 833
1

(ii) For the given process


n n
v P v P
2 2 1 1

kg m v
P
P
v or
n
/ 852 . 0
3
1
1
2
1
2

,
_

( )
K
R
v P
T Also 6 . 255
2 2
2

Change in entropy per unit mass is given by

,
_

,
_


1
2
1
2
1 2
ln ln
v
v
R
T
T
c S S
v
105
= 44.18J/K
(iii) For the process for which the index n = 1.5 we have
kg m x v / 764 . 0 5 . 0
100
200
3
5 . 1
1
2

,
_

K
x x
T 2 . 238
33 . 333
794 . 0 10 100
3
2

,
_

+
,
_


5 . 0
794 . 0
ln 33 . 333
300
2 . 238
ln 33 . 833
1 2
x x S S
= - 38.07 J/K
(iv) It can be seen from the results of (ii) and (iii) above that entropy increases
when n < and it decreases when n > .

11. A closed rigid cylinder is divided by a diaphragm into two equal compartments,
each of volume 0.1 m
3
. Each compartment contains air at a temperature of 20
0
C. The
pressure in one compartment is 2.5 MPa and in the other compartment it is 1 MPa.
The diaphragm is ruptured so that the air in the compartments mix to bring the
pressure to a uniform value through the insulated cylinder. Find the net change in
entropy due to mixing process.
Solution: Let suffix 1 denote the condition of air in the left half of the cylinder, suffix 2
the conditions in the right half of the cylinder and suffix 3 the condition after mixing
It if given that,
V
1
= V
2
= 0.1 m
3
, T
1
= T
2
= 20 + 273 = 293K, P
1
= 2.5 MPA, P
2
= 1.0 MPa
After mixing the temperature is given by
2 2 1 1
2 2 2 1 1 1
3
p p
p p
c m c m
T c m T c m
T
+
+

Since T
1
= T
2
and c
p1
= c
p2
, it follows that T
3
= T
1
= T
2
= 293 K
( S)
universe
= ( S)
1
+ ( S)
2
+ ( S)
surroundings
( S)
surroundings
= 0 as the cylinder is insulated and no heat transfer can take place during
the mixing process.
Therefore ( S)
universe
= ( S)
1
+ ( S)
2
1
]
1

+ +
1
]
1

+
2
3
2
3
2
1
3
1
3
1
ln ln ln ln
v
v
R
T
T
c m
v
v
R
T
T
c m
v v
kg
x
x x
RT
v P
m 973 . 2
293 287
1 . 0 10 5 . 2
6
1
1 1
1

kg
x
x x
m Similarly 189 . 1
293 287
1 . 0 10 0 . 1
6
2

Therefore on substituting, ( S)
universe
= 0.828kJ/K

12. 1 kg of air expands isentropically in a steady flow process from an initial state of
6.5 bar and 500 K to a final state of 1 bar. Neglecting kinetic and potential energies,
assuming the air to be a semi perfect gas, compute the temperature of the air at the
final state and work developed during the process.
106
Solution: From table C-21 (Thermodynamic Data Hand Book, by B.T. Nijaguna and B.S.
Samaga) at T
1
= 500 K P
r1
= 8.411 and h
1
= 503.02 kJ/kg
1
2
1
2
r
r
s
P
P
P
P
have We

,
_


294 . 1
5 . 6
1
411 . 8 ., .
1
2
1 2

,
_

,
_

s
r r
P
P
p P e i
Again from table C-21 at P
r2
= 1.294, by interpolation, h
2
= 294.2 kJ/kg and T
2
= 294 K
From SFEE, work developed w = h
1
h
2
= 503.02 294.2 = 208.82 kJ/kg
13. Air is compressed from 1 bar, 27
0
C to 4.5 bar, 177
0
C. Determine the change in
entropy per kg of air (i) taking variation of specific heats into account and (ii)
assuming specific heat at constant pressure to be constant and is equal to 1.01 kJ/kg-
K over this range of temperature.
Solution: T
1
= 300 K, T
2
= 450 K, P
1
= 1 bar, P
2
= 4.5 bar
(i) We have change in entropy per kg of air,
( )

,
_


1
2 0
1
0
2 1 2
ln
p
p
R s s s s
From table C-21 at T
1
= 300 K, K kg kJ s / 5153 . 2
0
1
and
at T
2
= 450 K, K kg kJ s / 9245 . 2
0
2

Therefore on substituting, s
2
s
1
= - 0.02508 kJ/kg-K
(ii) The change in entropy per kg of air
1
2
1
2
1 2
ln ln
P
P
R
T
T
c s s
p


K kg kJ / 0252 . 0
1
5 . 4
ln 289 . 0
300
450
ln 01 . 1

Thermodynamics of Non-reactive Mixtures
Assumptions:
1. Each individual constituent of the mixture behaves like a perfect gas.
2. The mixture behaves like a perfect gas.
3. Individual constituents do not react chemically when the mixture is undergoing a
process.
Mixture characteristics:
Figure: Homogeneous gas mixture
107
a,
b,
c,
.
.
.
P,
T,
V
Consider a mixture of gases a, b, c, . existing in equilibrium at a pressure P,
temperature T and having a volume V as shown in figure.
The total mass of the mixture is equal to the sum of the masses of the individual gases,
i.e., m
m
= m
a
+ m
b
+ m
c
+ .. where subscript m = mixture, a, b, c = individual gases.
Mass fraction: The mass fraction of any component is defined as the ratio of the mass of
that component to the total mass of the mixture. It is denoted by m
f
.
m
m
m
m
m
m
m
m
m Thus
c
fc
b
fb
a
fa
, , ,
1 . . . . . + + +
i
f
i
f c f b f a
m m m m
Where the subscript i stands for the i
th
component. It shows that the sum of the mass
fraction of all components in a mixture is unity.
Mole fraction: If the analysis of a gas mixture is made on the basis of the number of
moles of each component present, it is termed a molar analysis. The total number of
moles for the mixture is equal to the sum of the number of moles of the individual gases
i.e., n
m
= n
a
+ n
b
+ n
c
+ .. where subscript m = mixture, a, b, c = individual
gases.
(A mole of a substance has a mass numerically equal to the molecular weight of the
substance, i.e., 1 kg mol of O
2
has a mass of 32 kg, 1 kg mol of N
2
has a mass of 28 kg,
etc.,)
The mole fraction of any component is defined as the ratio of the number of moles of that
component to the total number of moles. It is denoted by y
m
c
c
m
b
b
m
a
a
n
n
y
n
n
y
n
n
y e i , , ., .
1 . . . . . . + + +
i
i
c b a
y y y y a n d
i.e., the sum of the mole fraction of all components in a mixture is unity.
The mass of a substance m is equal to the product of the number of moles n and the
molecular weight (molar mass) M, or m = nM
For each of the components we can write,
n
m
M
m
= n
a
M
a
+ n
b
M
b
+ n
c
M
c
+ ......
Where M
m
is the average molar mass or molecular weight of the mixture.
Or M
m
= y
a
M
a
+ y
b
M
b
+ y
c
M
c
Thus, the average molecular weight of a gas mixture is the sum of the products of all the
components of the mole fraction and corresponding molecular weight of each component.
Note: Universal gas constant MR R where M = molecular weight, R: specific gas
constant, and R =8.3143 kJ/kg-mole K
108
Partial Pressure:
Partial pressure of a constituent in a mixture is the pressure exerted when it alone
occupies the mixture volume at mixture temperature. If P
a
is partial pressure of gas a,
then P
a
V = m
a
R
a
T
Where m
a
= mass of gas a, R
a
= gas constant for gas a, similarly P
b
V = m
b
R
b
T
Partial Volume: Partial volume of a gas in a mixture is the volume occupied by the gas
component at mixture pressure and temperature. Let V
a
= partial volume of gas a and V
b
= partial volume of gas b
i.e., PV
a
= m
a
R
a
T & PV
b
= m
b
R
b
T
The Gibbs-Dalton Law
Consider a mixture of gases, each component at the temperature of the mixture occupying
the entire volume occupied by the mixture, and exerting only a fraction of the total
pressure as shown in figure.
+ + + ................. =
Applying the equation of state for this mixture we may write,
P
m
V = m
m
R
m
T = n
m
M
m
R
m
T = n
m
R
T
T R n V p Similarly
a a


T R n V p
b b


T R n V p
c c

We know that n
m
= n
a
+ n
b
+ n
c
+ ......
....... + + +
T R
V p
T R
V p
T R
V p
T R
V p
Hence
c b a m
Or p
m
= p
a
+ p
b
+ p
c
+ ....... |
V,T
= p
i
The above equation is known as the Gibbs Dalton Law of partial pressure, which states
that the total pressure exerted by a mixture of gases is equal to the sum of the partial
pressures of the individual components, if each component is considered to exist alone at
the temperature and volume of the mixture.
109
a+b
Mixture
P,
T,
V
a & b = gases of the mixture
V = Total volume of the mixture
T = Temperature of the mixture
P = Pressure of the mixture
m
a
, p
a
,
T, V
m
b
, p
b
,
T, V
m
c
, p
c
,
T, V
m
m
, p
m
,
T, V
110
Gas constant for the mixture:
We have P
a
V = m
a
R
a
T
P
b
V = m
b
R
b
T
Or (P
a
+ P
b
) V = (m
a
R
a
+ m
b
R
b
) T
Also, since the mixture behaves like a perfect gas,
We have PV = mRT --- (1)
By Daltons law of partial pressure, which states that, the pressure of mixture of gas is
equal to the sum of the partial pressures of the individual components, if each component
is considered to exist alone at the temperature and volume of the mixture.
i.e., P = P
a
+ P
b
PV = (m
a
R
a
+ m
b
R
b
) T --- (2)
From equation (1) and (2), mR = m
a
R
a
+ m
b
R
b
m
R m R m
R
b b a a
+

Also for gas mixture, P
a
V = m
a
R
a
T
= n
a
M
a
R
a
T

( ) R MR T R n V P
a a

Similarly T R n PV
a
a a
y
n
n
P
P

Similarly it can be shown that mole fraction = volume fraction
V
V
n
n
P
P
y Hence
a a a
a
,
Molecular weight of the mixture:
We have, P
a
V = m
a
R
a
T
P
a
V = n
a
M
a
R
a
T Similarly P
b
V = n
b
M
b
R
b
T
(P
a
+ P
b
) V = (n
a
M
a
R
a
+ n
b
M
b
R
b
) T
Also PV = nMRT
By Daltons law of partial pressure, P = P
a
+ P
b
nMRT = (n
a
M
a
R
a
+ n
b
M
b
R
b
) T
MR = y
a
M
a
R
a
+ y
b
M
b
R
b
R
R M y R M y
M
b b b a a a
+

Also, mR = m
a
R
a
+ m
b
R
b
R = m
fa
R
a
+ m
fb
R
b
b
b
a
a
M
R
R
M
R
R
M
R
R But , ;
b
f
a
f
M
R
m
M
R
m
M
R
b a
. . +
b a
a fb b fa
b
fb
a
fa
M M
M m M m
M
m
M
m
M
e i
. .
1
., .
+
+
111
a fb b fa
b a
M m M m
M M
M
. . +

The Amagat-Leduc Law: Expresses the law of additive volume which states that the
volume of a mixture of gases is equal to the sum of the volumes of the individual
components at the pressure and temperature of the mixture.
i.e., V
m
= V
a
+ V
b
+ V
c
.|
P, T
= i
i
V
For Dalton law, P
m
= P
a
+ P
b
+P
c
+ .|
V, T
= i
i
P

Gibbs Law: It states that the internal energy, the enthalpy and the entropy of a mixture
of gas is equal to sum of the internal energies, the enthalpies and entropies respectively of
the individual gases evaluated at mixture temperature and pressure.
U = U
a
+ U
b
mU = m
a
U
a
+ m
b
U
b
U = m
fa
U
a
+ m
fb
U
b
b V fb a V fa V
b
fb
a
fa
C m C m C
dT
dU
m
dT
dU
m
dT
dU
) ( ) ( + +
Similarly C
P
= m
fa
(C
p
)
a
+ m
fb
(C
p
)
b
If

V
C
Specific heat at constant volume on mole basis

P
C Specific heat at constant pressure on mole basis
( ) ( ) &
b
V b
a
V a V
C y C y C +
( ) ( )
b P b a P a P
C y C y C +
Isentropic process of gaseous mixture: When a mixture of say two gases, a & b, is
compressed or expanded isentropically, the entropy of the mixture remains constant i.e.,
there is no change in the entropy of the entire system. i.e., S
m
= S
a
+ S
b
= 0
But this does not mean that there is no change in the entropy of an individual gas. During
the reversible adiabatic compression or expansion process, the entropy of one of the two
gases will increase, while the entropy of the other one will decrease by the same amount,
and thus, as a whole, the entropy of the system will remain constant.
The compression or expansion of each constituent will be reversible, but not adiabatic
and hence the energy transferred as heat from one of the two gases must be exactly equal
to the energy received by the other one. This is also true when more than two gases are
involved in the process.
112
Volumetric and Gravimetric Analysis: When the analysis of a gaseous mixture is based
on the measurement of volume, it is called a volumetric analysis, whereas when it is
based on the measurement of mass, it is called the gravimetric analysis. Flue gases
generally contain CO
2
, CO, N
2
O
2
and H
2
O in the form of vapour.
The volumetric analysis of a dry flue gas is generally done with Orsat apparatus, which is
designed to absorb CO
2
, O
2
and CO. The N
2
content of the gas is obtained by difference.
Note 1: The volume fraction & mole fraction of each individual gas are equal. This
enables the conversion of the volumetric analysis to gravimetric analysis and vice versa.
Note 2: Molecular weight of common gases is given in Table C-6.
Note 3: Specific heat of gases at constant pressure C
P
are given in Table C-11
Problems:
1. A perfect gas mixture consists of 2.5 kg of N
2
and 1.5 kg of CO at a pressure of 2
bar and at a temperature of 15
0
C. Determine (a) The mass and mole fraction of each
constituent, (b) The equivalent molecular weight of the mixture, (c) The partial
pressure of each gas, and (d) The specific gas constant of the mixture.
Solution: (a) The total mass of the mixture m
m
= 2.5 + 1.5 = 4 kg
The mass fraction
625 . 0
4
5 . 2
2
2

m
N
f
m
m
m
N
375 . 0
4
5 . 1

m
CO
f
m
m
m Similarly
CO
The mass of the substance m = nM
0893 . 0
28
5 . 2
2
2
2

N
N
N
M
m
n
0536 . 0
28
5 . 1

CO
CO
CO
M
m
n and
Total no. of moles in the mixture, n
m
= 0.0893 + 0.0536 = 0.1429
The mole fraction of each constituent,
625 . 0
1429 . 0
0893 . 0
2

N
y
375 . 0
1429 . 0
0536 . 0
&
CO
y
(b) The equal molecular weight of the mixture,
CO CO N N m
M y M y M +
2 2
= 0.625 (28) + 0.375 (28)
= 28 kg/kg-mole
(c) The partial pressure of
bar P y P N
m N N
25 . 1 ) 2 ( 625 . 0 ,
2 2
2

The partial pressure of
bar P y P CO
m CO CO
75 . 0 ) 2 ( 375 . 0 ,

(d) The specific gas constant of the mixture,
K kg kJ
M
R
R
m
m
0
/ 297 . 0
28
3143 . 8

113
CO f N f m
R m R m R or
CO N
+
2
2
= 0.625 (0.297) + 0.375 (0.297)
= 0.297 kJ/kg-
0
K
2. A mixture of gas has the following volumetric analysis. O
2
= 30%, CO
2
= 40%, N
2
= 30%. Determine (a) the analysis on a mass basis (b) the partial pressure of each
component if the total pressure is 100 kPa and a temperature is 32
0
C (c) the molecular
weight of the mixture.
Solution: V
O2
/ V
m
= V
fO2
= 0.3, V
CO2
/ V
m
= V
fCO2
= 0.4, V
N2
/ V
m
= V
fN2
= 0.3, P = 100 x
10
3
Pa T = 305 K
We know that, V
fi
= y
i
= P
i
/P
m
Therefore, y
O2
= 0.3, y
CO2
= 0.4, y
N2
= 0.3,
P
O2
= 0.3 (1) = 0.3 bar;
P
CO2
= 0.4 bar & P
N2
= 0.3 bar
Also P
O2
V = m
O2
R
O2
T
& P
m
V = m
m
R
m
T
m O
f O
m
O
R M
R
m y
P
P 1
. .
2
2
2
02

R
R M
y m
m O
O f
O
.
.
2
2
2
--- (1)
M
m
= y
O2
M
O2
+ y
CO2
M
CO2
+ y
N2
M
N2
= 0.3 (32) + 0.4 (44) + 0.3 (28)
= 35.6 kg/kg-mole
K kg kJ R
m
0
/ 2335 . 0
6 . 35
3143 . 8

Therefore from equation (1),
2697 . 0
3143 . 8
) 2335 . 0 )( 32 ( 3 . 0
02

f
m
4943 . 0
3143 . 8
) 2335 . 0 )( 44 ( 4 . 0
2

CO
f
m Similarly
2359 . 0
3143 . 8
) 2335 . 0 )( 28 ( 3 . 0
&
2

N
f
m
3. A mixture of perfect gas at 20
0
C, has the following composition by volume, N
2
55%, O
2
20%, methane 25%. If the partial pressure of methane is 0.5 bar, determine
(i) partial pressure of N
2
& O
2
, (ii) mass fraction of individual gases, (iii) gas constant
for the mixture (iv) molecular weight of the mixture.
Solution: Let V
m
= Total volume of the mixture
bar P K T
V
V
V
V
V
V
Given
CH
m
CH
m
O
m
N
5 . 0 , 293 , 25 . 0 , 2 . 0 , 55 . 0 ,
4
4 2 2

(i) We have 4
4 4
CH
m
CH
m
CH
y
V
V
P
P

bar P
P
m
m
2 25 . 0
5 . 0

114
bar P
V
V
P
P
Also
N
m
N
m
N
1 . 1 55 . 0 ,
2
2 2

bar P
V
V
P
P
O
m
O
m
O
4 . 0 2 . 0 &
2
2 2

Molecular weight of the mixture, M
m
= y
N2
M
N2
+ y
O2
M
O2
+ y
CH4
M
CH4
= 0.55 (28) + 0.2 (32) + 0.25 (16)
= 25.8 kg/kg-mole
Gas, constant of the mixture,
K kg kJ
M
R
R
m
m
0
/ 322 . 0
8 . 25
3143 . 8

Mass Fraction
We have P
N2
V = m
N2
R
N2
T
& P
m
V = m
m
R
m
T
m
N
m
N
m
N
R
R
m
M
P
P
2 2 2
.
m N
f
R M
R
m
N
1
. . 55 . 0
2
2

596 . 0
322 . 0
1
.
28
3142 . 8
.
2 2

N N
f f
m m
248 . 0
3143 . 8
322 . 0 ) 32 ( 2 . 0
. .
2 2
2

m
O
m
O
f
R
R
M
P
P
m Similarly
O
155 . 0
3143 . 8
) 322 . 0 )( 16 ( 25 . 0
. . &
4 4
4

m
CH
m
CH
CH
R
R
M
P
P
m
4. A mixture of 3.5 kg of O
2
& 2.5 kg N
2
is stored in a vessel of 0.3 m
3
at a
temperature of 27
0
C. Find the partial pressures and mole fraction of each constituent.
Also determine the molecular weight and characteristic gas constant for the mixture.
Solution: m
O2
= 3.5 kg m
N2
= 2.5 kg V
m
= 0.3 m
3
T = 300
0
K. P
O2
= ?
P
N2
= ? y
O2
= ? y
N2
= ? M
m
= ? R
m
= ?
We have m = nM
0893 . 0
28
5 . 2
& 1094 . 0
32
5 . 3
2 2

N O
n n
n
m
= 0.1094 + 0.0893 = 0.1987 moles
4494 . 0
1987 . 0
0893 . 0
& 5506 . 0
1987 . 0
1094 . 0
2 2

N O
y y
The average molecular weight, M
m
= y
O2
M
O2
+ y
N2
M
N2
= 0.5506 (32) + 0.4494 (28)
= 30.204
The characteristic gas constant,
K kg kJ
M
R
R
m
m
0
/ 2753 . 0
204 . 30
3143 . 8

Partial Pressure:
We have P
m
V
m
= m
m
R
m
T
115
bar
x
P
m
52 . 16
3 . 0
300 ) 10 2753 . 0 ( 6
3

bar P y
P
P
But
O O
m
O
095 . 9
2 2
2

bar P y
P
P
N N
m
N
424 . 7 &
2 2
2

Or P
N2
= P
m
P
O2
= 16.52 9.095 = 7.424 bar
5. A mixture of ideal gases consists of 3 kg of N
2
and 5 kg of CO
2
at a pressure of
300 kPa and a temperature of 20
0
C, determine (i) the mole fraction of each
constituent (ii) molecular weight of the mixture (iii) gas constant of the mixture (iv)
the partial pressure and partial volumes of the constituent.
Solution: m
N2
= 3 kg m
CO2
= 5 kg P
m
= 300 x 10
3
N/m
2
T
m
= 293
0
K
(i) We have m = nM
1136 . 0
44
5
1071 . 0
28
3
2 2

CO N
n n
Total no. of moles in the mixture = n
m
= 0.1071 + 0.1136 = 0.2207 moles
5146 . 0
2207 . 0
1136 . 0
&
4852 . 0
2207 . 0
1071 . 0
2
2


CO
N
y
y
(ii) M
m
= y
N2
M
N2
+ y
CO2
M
CO2
= 0.4852 (28) + 0.5146 (44)
= 36.23 kg/kg-mole
(iii)
K kg kJ
M
R
R
m
m
0
/ 2295 . 0
23 . 36
3143 . 8

(iv) P
N2
= y
N2
P
m
= 0.4852 (3) = 1.456 bar
P
CO2
= y
CO2
P
m
= 0.5146 (3) = 1.544 bar
Also P
m
V
N2
= m
N2
R
N2
T
K kg kJ
M
R
R
N
N
0
2
2
/ 2969 . 0
28
3143 . 8

( )
3
3
3
2
87 . 0
10 300
10 293 2969 . 0 3
m
x
x
V
N

Similarly P
m
V
CO2
= m
CO2
R
CO2
T

K kg kJ
M
R
R
CO
CO
0
2
2
/ 1889 . 0
44
3143 . 8

( )
( )
3
3
3
2
923 . 0
10 300
293 10 1889 . 0 5
m
x
x x
V
CO

6. A gaseous mixture contains 21% by volume N
2
, 50% by volume of H
2
and 29%
by volume of CO
2
. Calculate (i) the molecular weight of the mixture (ii) gas constant
of the mixture (iii) the ratio of specific heats of the mixture. Assume that C
P
for N
2
,
H
2
and CO
2
as 1.038, 14.235 and 0.821 kJ/kg-
0
K respectively.
116
Solution: We have y
a
= v
fa
= P
a
P
m
Given: y
N2
= 0.21 y
H2
= 0.5 y
CO2
= 0.29
M
m
= y
N2
M
N2
+ y
H2
M
H2
+ y
CO2
M
CO2
= 0.21 (28) + 0.5 (2) + 0.29 (44) = 19.64 kg/kg-mole
Gas constant
K kg kJ R
m
0
/ 4233 . 0
64 . 19
3143 . 8

(iii) We have P
N2
V = m
N2
R
N2
T
& P
m
V = m
m
R
m
T
m
N
m
N
m
N
R
R
m
m
P
P
2 2 2

m N
f N
m
N
R M
R
m y
P
P
But
N
1
21 . 0
2
2
2
2

4233 . 0
1
28
3143 . 8
2 N
f
m
2994 . 0
2

N
f
m
( )( )
1018 . 0
3143 . 8
4233 . 0 4 5 . 0
2
2
2

m
H
H f
R
R
M
y m Similarly
H
( )( )
6496 . 0
3143 . 8
4233 . 0 44 29 . 0
&
2
2
2

m
CO
CO f
R
R
M
y m
CO
Specific heat at constant pressure for the mixture,
( )
2 2 2 2 2 2
. . .
CO CO H H N N
P f P f P f
m
p
C m C m C m C + +
= 0.2994 (1.038) + 0.1018 (14.235) + 0.6496 (0.821)
= 2.2932 kJ/kg-
0
K
( ) ( )
m m P m V
R C C
= 2.2932 0.4233 = 1.8699 kJ/kg-
0
K
The ratio of specific heats of the mixture
2264 . 1
8699 . 1
2932 . 2

m
m
V
p
m
C
C

117
Entropy of a Gas Mixture
Entropy of a gas mixture S
m
= S
a
+ S
b
+ S
c
+ .
The specific entropy on the mass basis
S
m
= m
fa
s
a
+ m
fb
s
b
+ m
fc
s
c
+ .. =
i
i
fi
s m

On differentiating we get
(dS)
m
= m
fa
(ds)
a
+ m
fb
(ds)
b
+ m
fc
(ds)
c
+
Expressing the entropy of a perfect gas as a function of temperature and pressure, the
change in entropy is given by
p
dp
R
T
dT
c dS
p

On substituting this expression in the above equation, the change in entropy of a gas
mixture in the differential form becomes
( ) ( ) ( ) ( ) ....... +
1
]
1

+
1
]
1

+
1
]
1


c
c
c
c
p fc
b
b
b
b
p fb
a
a
a
a
p fa m
p
dp
R
T
dT
c m
p
dp
R
T
dT
c m
p
dp
R
T
dT
c m dS

Assuming the specific heats as constant and integrating the above equation between states
(1) and (2), the change in entropy of the perfect gas mixture on the mass basis can be
written as
( ) ( ) ( )
( ) ..... ln ln
ln ln ln ln
1 ,
2 ,
1
2
1 ,
2 ,
1
2
1 ,
2 ,
1
2
1 2
+
1
1
]
1

+
1
1
]
1

+
1
1
]
1


c
c
c
c
p fc
b
b
b
b
p fb
a
a
a
a
p fa m
p
p
R
T
T
c m
p
p
R
T
T
c m
p
p
R
T
T
c m s s
Similarly the change in entropy in the gas mixture on molal basis
( ) ( ) ( )
( ) .... ln ln
ln ln ln ln
1 ,
2 ,
1
2
1 ,
2 ,
1
2
1 ,
2 ,
1
2
1 2
+
1
1
]
1

+
1
1
]
1

+
1
1
]
1


c
c
c
c
p c
b
b
b
b
p b
a
a
a
a
p a m
p
p
R
T
T
c y
p
p
R
T
T
c y
p
p
R
T
T
c y s s
where p
a, 1
, p
b, 1
p
c, 1
etc are partial pressures at state one and p
a, 2
, p
b, 2
p
c, 2
etc are partial
pressures at state 2 of the constituent gases, a, b, c, etc respectively.
Isentropic process of gaseous mixture:
When the mixture of gases is compressed or expanded from state (1) to state (2)
isentropically, we have
m
m
p
p
T
T

1
1
2
1
2

,
_


Where
m
is the specific heat ratio of the gaseous mixture, which is given by
( )
( )
( ) ( )
( ) ( )
b v fb a v fa
b
p fb
a
p fa
m v
m
p
m
c m c m
c m c m
c
c
+
+

118
( ) ( )
( ) ( )
b v b a v a
b
p b
a
p a
c m c m
c m c m
+
+

On molal basis
( )
( )
( ) ( )
( ) ( )
( ) ( )
( ) ( )
b v b a v a
b
p b
a
p a
b v b a v a
b
p b
a
p a
m v
m
p
m
c n c n
c n c n
c y c y
c y c y
c
c
+
+

+
+

Adiabatic Mixing of Perfect Gases Initially at Same Temperature and Pressure
Consider several gases of mass m
a
, m
b
, m
c
etc at the same temperature T and
pressure p confined in separate compartment of an adiabatic vessel. When the partitions
between compartments are pulled out from the vessel, the gases mix with each other until
a new equilibrium state is attained.
Since in an adiabatic mixing processes, there is no heat transfer and also no work crosses
the boundary of the system, for a closed system U = 0 i.e., U
initial
= U
final
. Hence the
resulting temperature of the mixture remains the same as that of each gas before mixing.
In other words, the mixing process is isothermal. However each gas after free expansion
will attain the partial pressure of the mixture and each gas will occupy the entire volume
of the vessel. By using the perfect gas equation, it can be shown that the total pressure of
the mixture is equal to the initial pressure p of each individual gas.
As the mixing process is irreversible, according to the second law of thermodynamics,
the entropy of the mixture increases. Assuming constant specific heats, the change in
entropy of the gas a is given by
( ) ( )
1
1
]
1


1 ,
2 ,
1
2
ln ln
a
a
a
a
p a a
p
p
R
T
T
c m S

Since the temperature remains constant, T
1
= T
2
The change in entropy of the gas a may then be written as
( )
1 ,
2 ,
ln
a
a
a a a
p
p
R m S
Where the subscripts (1) and (2) refer to the initial and final states of the component a in
the mixture.
Since the partial pressure p
a,2
is less than the partial pressure p
a,1
of the mixture which is
equal to p, the pressure of the individual gas, the change in entropy is always positive.
This conclusion is consistent with the fact that for an irreversible process, entropy always
increases.
Similarly the change in entropy of the gas b
( )
1 ,
2 ,
ln
b
b
b b b
p
p
R m S
Hence the total change in entropy of the mixture is
119
( ) ....... ln ln
1 ,
2 ,
1 ,
2 ,
b
b
b b
a
a
a a m
p
p
R m
p
p
R m S
[ ]
p T b b b a a a
y R m y R m
,
....... ln ln + +


i
i i i
y R m ln
Similarly on molal basis, the total change in entropy of the mixture
( )


i
i i m
y n R S ln
It is evident from the above equation that the increase in entropy of the mixture of perfect
gases depends only on the number of moles of the component gases and is independent of
the nature of the individual gases.
Adiabatic Mixing of Perfect Gases Initial at Different Pressure and Temperature:
Consider several gases of mass m
a
, m
b
, m
c
, etc, initially at different pressure and
temperature confined at separate compartments in a thermally insulated vessel (adiabatic
vessel). When the partitions are removed, the gases mix with one another. The mass and
volume of the mixture are given by
m
m
= m
a
+ m
b
+ m
c
+ ..
V
m
= V
a
+ V
b
+ V
c
+ ..
Where, m
a
, m
b
, m
c
, . are the initial masses and V
a
, V
b
, V
c
. are the initial volumes of
each constituent respectively.
Since during the mixing process there is no heat and work interaction, the change in
internal energy of the system is zero, i.e., the sum of the internal energies of the
individual gases before mixing is equal to the internal energy of the mixture.
Or U
m
= U
a
+ U
b
+ .
Taking 0
0
C as the reference point, we get
m
m
(c
v
)
m
T
m
= m
a
(c
v
)
a
T
a
+ m
b
(c
v
)
b
T
b
+
( ) ( )
( )
m v m
b b v b a a v a
m
c m
T c m T c m
T Or
+

Where T
m
is the temperature of the mixture. Similarly on molal basis,
( ) ( )
( )
m v m
b b v b a a v a
m
c n
T c n T c n
T
+

Using equation of state, the final pressure of the mixture on mass basis is
m
m m m
m
V
T R m
p
Where R
m
is the specific gas constant of the mixture. On molal basis the pressure of the
mixture is,
m
m m m
m
V
T R n
p
Since it is a irreversible adiabatic mixing process, the entropy must increase. Since the
change in entropy of the mixture is equal to the sum of the changes in entropies of the
individual gases, we can write
( ) ( ) ( ) ..... + +
b a m
S S S
120
The change in entropy of individual gases can be calculated by considering that each
individual gas exists alone and is expanded from the initial state to the temperature and
volume of the mixture, the pressure at the final state being the partial pressure of the
constituent in the mixture. Thus the changes in entropy of the constituent a is
( ) ( )
1
1
]
1


1 ,
2 ,
ln ln
a
a
a
a
m
a
p a a
p
p
R
T
T
c m S
Where p
a,1
is the initial pressure of the gas a and p
a,2
is the partial pressure of gas a in the
mixture. Similarly for gas b the change in entropy is
( ) ( )
1
1
]
1


1 ,
2 ,
ln ln
b
b
b
b
m
b
p b b
p
p
R
T
T
c m S
On molal basis,
( ) ( )
1
1
]
1


1 ,
2 ,
ln ln
a
a
a
m
a
p a a
p
p
R
T
T
c n S
( ) ( )
1
1
]
1


1 ,
2 ,
ln ln
b
b
b
m
b
p b b
p
p
R
T
T
c n S
Problems

7. The mass analysis of a gas mixture shows that it consists of 60% of N
2
& 30% of
CO
2
& 10% of CO. If the temperature and pressure of the mixture is 20
0
C & 3 bar,
compute (i) the partial pressure of the components, (ii) Molecular weight of the
mixture, (iii) gas constant for the mixture.
Solution: We have
CO f CO fCO N f m
R m R m R m R
CO N
+ +
2 2 2
2
R
M
m
R
M
m
R
M
m
M
R
e i
CO
f
CO
f
N
f
m
CO CO N
+ +
2 2
2 2
., .
CO
f
CO
f
N
f
m
M
m
M
m
M
m
M
CO CO N
+ +

2 2
2 2
1
mole kg kg
+ +
/ 429 . 31
28
1 . 0
44
3 . 0
28
6 . 0
1
K kg kJ R
m
0
/ 2645 . 0
429 . 31
3143 . 8

Considering 1 kg of mixture,
0318 . 0
429 . 31
1

m
m
m
M
m
n
We have
m
m i
m
i
i
n
M m
n
n
y
/

6 . 0
0318 . 0
429 . 31 / 6 . 0
2

N
y
3 . 0
0318 . 0
429 . 31 / 3 . 0
2

CO
y
121
1 . 0
0318 . 0
429 . 31 / 1 . 0
&
CO
y
bar P y
P
P
N N
m
N
8 . 1 ) 3 ( 6 . 0
2 2
2

bar P y
P
P
CO CO
m
CO
9 . 0 ) 3 ( 3 . 0
2 2
2

bar P y
P
P
CO CO
m
CO
3 . 0 ) 3 ( 1 . 0 &
1. b) For a reference state of 0
0
C and 1 bar also determine (i) constant pressure specific
heat of the mixture (ii) enthalpy and internal energy of mixture (iii) entropy of mixture.
Solution: Constant pressure molal specific heat of a perfect gas is given by
R
f
C
P

,
_

+
2
1
Where f = degree of freedom of the molecule.
At low temperatures, for monatomic gas f = 3, for diatomic gas, f = 5 and for
polyatomic gas f = 6
R C gas manatomic a For
P
2
5
,

R C gas diatomic a For
P
2
7
,
R C gas polyatomic a For
P
4 ,
( )
K k g k J M R C
N P
N
0
2
/ 0 3 9 3 . 1
2 8
3 1 4 3 . 8
2
7
/
2
7
2

K kg kJ C
CO
P
0
/ 6614 . 0
44
) 3143 . 8 ( 5 . 3
2

K kg kJ C
CO
P
0
/ 0393 . 1
28
) 3143 . 8 ( 5 . 3

CO CO CO CO N N
P f P f P f m P
C m C m C m C + +
2 2 2 2
) (
= 0.6 (1.0393) + 0.3 (0.6614) + 0.1 (1.0393)
= 0.9259 kJ/kg-
0
K
( ) ( )
m m P m v
R C C
K kg kJ
0
/ 6614 . 0
429 . 31
3143 . 8
9259 . 0
(ii) Assuming that at 0
0
C u = 0
( ) ( ) 273 293
0 0
0 20

V
C C
C u u
= 13.229 kJ/kg
Enthalpy, h
20
=
( )
C
PV u 0
20
+
= 13.229 + RT
= 13.229 + 0.2645 (293)
= 90.73 kJ/kg
(iii) Entropy at 0
0
C and 1 bar is assumed to be zero
122
( )
1
]
1

,
_


,
_


P
dP
R
T
dT
C S s S S
p i
bar C bar C 1 , 0 3 , 20
0 0


3
1
293
273
P
dP
R
T
dT
C
P
K kg kJ
0
/ 225 . 0
1
3
ln 2645 . 0
273
293
ln 9259 . 0


8. A tank of 0.7 m
3
capacity contains O
2
at a pressure of 5 bar and 320 K
temperature. N
2
is introduced without change in temperature until the pressure in the
tank becomes 8 bar. Determine the mass of each gas in the tank and partial volume of
each gas.
Solution: V
m
0.7 m
3
P
O2
= 5 bar T = 320 K P
m
= 8 bar m
O2
= ? m
N2
= ?
V
O2
= ? V
N2
= ?
We have P
O2
V = m
O2
R
O2
T
P
N2
V = m
N2
R
N2
T
Also, P
m
= P
O2
+ P
N2
P
N2
= 3 bar
& P
m
V = m
m
R
m
T
3
2
2
2
2
4375 . 0 ) 7 . 0 ( 625 . 0 625 . 0 m V
V
V
y
P
P
that know we
O
m
O
O
m
O

3
2
2
2
2
2625 . 0 ) 7 . 0 ( 375 . 0 375 . 0 m V
V
V
y
P
P
N
m
N
N
m
N

Also P
O2
V
O2
= m
O2
R
O2
T &
2
2
O
O
M
R
R
kg
x x
x x x
m
O
631 . 2
320 10 3143 . 8
32 4375 . 0 10 5
3
5
2

kg
x x
x x x
m Similarly
N
8288 . 0
320 10 3143 . 8
28 2625 . 0 10 3
3
5
2

9. a) A gaseous mixture contains 21% by volume of N
2
, 50% by volume of H
2
and
29% by volume of CO
2
. Calculate the gas constant of the mixture and the ratio of
specific heats. If the mixture pressure is at 1 bar and the mixture temperature is 10
0
C,
calculate the partial pressures and mass fractions of the constituents.
b) A cylinder contains 0.085m
3
of this mixture at 1 bar and 10
0
C. The gas undergoes a
polytropic process according to the law Pv
1.2
= constant to a final volume which is
one fifth of the initial volume. Determine i) the magnitude and direction of work
transfer, ii) magnitude and direction of heat transfer and iii) the change in entropy.
Solution: From tables C-6, M
N2
= 28.02, M
H2
= 2.016 and M
CO2
= 44
From table C-11, C
PN2
= 1.039 kJ/kg-K, C
PH2
= 14.15 kJ/kg-K and C
PCO2
= 0.818 kJ/kg-K
a) Given V
N2
/ V = V
fN2
= 0.21, V
H2
/ V = V
fH2
= 0.5, V
CO2
/ V = V
fCO2
= 0.29
P = 1 bar, T = 283 K
123
K kg kJ
M
R
R
K kg kJ
M
R
R
K kg kJ
M
R
R have We
CO
CO
H
H
N
N



/ 189 . 0
44
3143 . 8
/ 124 . 4
016 . 2
3143 . 8
/ 297 . 0
013 . 28
3143 . 8
2
2
2
2
2
2
Therefore C
vN2
= 1.039 0.297 = 0.742 kJ/kg-K
C
vH2
= 14.15 4.124 = 10.026 kJ/kg-K
C
vCO2
= 0.818 0.189 = 0.629 kJ/kg-K
We know that volume fraction = mole fraction
i.e., y
N2
= V
fN2
= 0.21
y
H2
= V
fH2
= 0.5
y
CO2
= V
fCO2
= 0.29
Molecular weight of the mixture M
m
= y
N2
M
N2
+ y
H2
M
H2
+ y
CO2
M
CO2
= 0.21 x 28.013 + 0.5 x 2.016 + 0.29 x 44
= 19.654 kg/kg-mole
Gas constant for the mixture = R
m
= 8.3143 / 19.654
= 0.423 kJ/kg-K
Mass fraction of N
2
m
N
N
m m
N N
m
N
fN
M
M
y
M n
M n
m
m
m
2
2
2 2 2
2



2993 . 0
654 . 19
013 . 28
21 . 0 x
Similarly
0513 . 0
654 . 19
016 . 2
5 . 0
2
x m
fH
6494 . 0
654 . 19
44
29 . 0
2
x m
fCO
For the mixture C
vm
= M
fN2
C
vN2
+ M
fH2
C
vH2
+ M
fCO2
C
vCO2
= 0.2993 x 0.742 + 0.0513 x 10.026 + 0.6494 x 0.629
= 1.145 kJ/kg-K
C
Pm
= C
v
+ R
m

= 1.145 + 0.423
= 1.568 kJ/kg-K
Therefore the ratio of specific heat for the mixture is
m
= C
Pm
/ C
vm

= 1.568 / 1.145
= 1.369
b) Mass of the mixture
kg
x
x x
RT
V p
m 071 . 0
283 423
085 . 0 10 1
5
1
1 1

,
_



For the given process,
2 . 1
2 2
2 . 1
1 1
V p V p
( ) bar x p
V
V
p 9 . 6 1 5
2 . 1
1
2 . 1
2
1
2

,
_


( )
1
2 2 1 1
2 1 2 1



n
V p V p
W W
d
124

( )
J
x x x x x
16150
1 2 . 1
085 . 0
5
1
10 9 . 6 085 . 0 10 1
5 5

1
]
1

,
_

Negative sign indicates that work is done by the surroundings on the gas mixture.
Temperature of the mixture at the end of the process is,
K
x
x x x
mR
V p
T
6 . 390
423 071 . 0
085 . 0
5
1
10 9 . 6
5
2 2
2

,
_



Or
K T e i
p
p
T
T
n
n
5 . 390
1
9 . 6
283 ., .
2 . 1
2 . 0
2
1
1
2
1
2

,
_

,
_

Change in internal energy, U


2
U
1
= mC
v
(T
2
T
1
)
= 0.071 x 1.145 x (390.6 283)
= 8.747 kJ
By first law of TD, Q
1-2
= W
1-2
+ (U
2
U
1
)
= - 16.15 + 8.747
= - 7.403 kJ
Negative sign indicates that heat transfer takes place from the mixture to the
surroundings
Change in entropy of the mixture 1
]
1

,
_

,
_


1
2
1
2
1 2
ln ln
V
V
R
T
T
C m S S
v

K kJ
x x
/ 02214 . 0
5
1
ln 423 . 0
283
6 . 390
ln 145 . 1 071 . 0

1
]
1

,
_

+
,
_

10. Calculate the constant volume and constant pressure specific heats of a gas
mixture consisting of 1 kg of oxygen and 2 kg of nitrogen at a pressure of 1.5 bar and
temperature 20
0
C. Also determine the change in internal energy, enthalpy and entropy
of the mixture when it is heated under constant volume to a temperature of 100
0
C.
Solution: From tables C-6, M
O2
= 32, M
N2
= 28
From table C-11, C
PN2
= 1.038 kJ/kg-K, C
PO2
= 0.917 kJ/kg-K
C
vN2
= C
pN2
R
N2
= 1.038 8.3143 / 28 = 0.741 kJ/kg-K
C
vO2
= C
pO2
R
O2
= 0.917 8.3143 / 32 = 0.653 kJ/kg-K
( ) ( )

i
i v fi m v
c m c
K kg kJ x x + / 712 . 0 741 . 0
3
2
653 . 0
3
1
( ) ( )

i
i
p fi
m
p
c m c
K kg kJ x x + / 998 . 0 038 . 1
3
2
917 . 0
3
1
Change in internal energy,
125
( ) ( )
1 2
T T c m U
m v m

= 3 x 0.712 (100 20)
= 170.88 kJ
Change in enthalpy,
( ) ( )
1 2
T T c m H
m
p m

= 3 x 0.998 (100 20)
= 239.52 kJ
Change in entropy of the mixture,
( ) ( )
1
]
1

,
_

,
_


1
2
1
2
ln ln
V
V
R
T
T
c m S
m m v m m
Since the volume is constant, the change in entropy,
( ) ( )
1
]
1

,
_


1
2
ln
T
T
c m S
m v m m
293
373
ln 712 . 0 3x
= 0.5156 kJ/K
11. A gaseous mixture consisting of 1 kg of helium and 2.5 kg of nitrogen is
compressed isentropically in a closed system from a pressure of 1 bar, 27
0
C to a
pressure of 7 bar. Assuming specific heats of helium and nitrogen to be constant,
determine the specific heats of the mixture, the change in entropy of individual gases,
and the change in internal energy of the mixture. Also find the work required for the
compression process. Assume C
vHe
= 3.14 kJ/kg-K, C
vN2
= 0.741 kJ/kg-K, C
pHe
=
5.233 kJ/kg-K and C
pN2
= 1.038 kJ/kg-K
Solution:
( ) ( )

i
i v fi m v
c m c
K kg kJ x x + / 426 . 1 741 . 0
5 . 3
5 . 2
140 . 3
5 . 3
1
( ) ( )

i
i
p fi
m
p
c m c
K kg kJ x x + / 236 . 2 038 . 1
5 . 3
5 . 2
233 . 5
5 . 3
1
( )
( )
568 . 1
426 . 1
236 . 2

m v
m
p
m
c
c

m
m
p
p
T T

1
1
2
1 2

,
_


K 607
1
7
300
568 . 1
568 . 0

,
_

Hence the change in entropy of helium


( ) ( )
1
1
]
1


1 ,
2 ,
1
2
ln ln
He
He
He
He
p He He
p
p
R
T
T
c m S

126
1
7
,
1
2
1 ,
2 ,
1 ,
2 ,
2
2

p
p
p
p
p
p
that know We
N
N
He
He
( )
1
]
1


1
7
ln 093 . 2
300
607
ln 233 . 5 1
He
S
= - 0.38 kJ/K
Similarly
( )
1
]
1


1
7
ln 297 . 0
300
607
ln 038 . 1 5 . 2
2
N
S
= + 0.38 kJ/K
Internal energy of the mixture
( ) ( )
1 2
T T c m U
m v m m

= 3.5 x 1.426 (607 300)
= 1532.24 kJ
For a closed system we have, q w = du.
Since the process adiabatic, q = 0
Therefore w = -dU.
Therefore the work required for the compression process,
W
1-2
= - (dU)
m
= - 1532.24 kJ

12. Two thermally insulated vessels, each of 0.85 m
3
in volume are isolated from each
other by a partition wall. One of the vessels contains nitrogen and the other oxygen,
each at 5 bar and 100
0
C. As soon as the partition between the vessels is removed, the
two gases mix adiabatically. Determine the increase in entropy of the gas mixture.
Solution: Since the pressure and temperature of each gas are equal, the pressure and
temperature of the mixture will not change, i.e., the mixture will be at 5 bar and 373 K.
But the volume of the mixture will be 2x0.85= 1.7m
3
and the pressure of N
2
and O
2
is
2
1
7 . 1
85 . 0
2 2

m
N
m
N
v
v
p
p
Therefore p
N2
=0.5x5=2.5 bar and p
O2
=2.5 bar
Using perfect gas equation, mass of the N
2
,
kg
x
x
x x
m
N
84 . 3
373
28
10 3143 . 8
7 . 1 10 5 . 2
3
5
2

and
kg
x
x
x x
m
O
39 . 4
373
32
10 3143 . 8
7 . 1 10 5 . 2
3
5
2

Therefore the increase in entropy of the mixture is,
( ) [ ]
2 2 2 2 2 2
ln ln
O O O N N N m
y R m y R m S +
1
]
1

+
5
5 . 2
ln
32
3143 . 8
39 . 4
5
5 . 2
ln
28
3143 . 8
84 . 3 x x
=1.579kJ/K
13. Two kg-mole of CO
2
at a pressure of 1.8 bar, 80
0
C is mixed in a thermally
insulated vessel with 3 kg-mole of N
2
at 2.2 bar, 60
0
C. When the mixture is at
127
equilibrium, determine the final temperature and pressure and the change in entropy
of the mixture. Assume C
vCO2
= 0.653 kJ/kg-K, C
vN2
= 0.741 kJ/kg-K
Solution: we have,
( ) ( )
( )
m v m
N N v N CO CO v CO
m
c n
T c n T c n
T
2 2 2 2 2 2
+

Now the molal specific heats at constant volume of the constituent gases are
( ) ( )
2 2 2
CO v CO CO v
c x M c
= 44x0.653
=28.73 kJ/kg-mole K
( ) ( )
2 2 2
N v N N v
c x M c And
=28x0.741
=20.75 kJ/kg-mole K
75 . 20 3 73 . 28 2
333 75 . 20 3 353 73 . 28 2
x x
x x x x
T
m
+
+

=342.6 K
The final pressure of the mixture is
m
m m
m
V
T R n
p
The volume of the individual gases is,
3
61 . 32
8 . 1 100
353 3143 . 8 2
2
2 2
2
m
x
x x
p
T R n
V
CO
CO CO
CO

3
75 . 37
2 . 2 100
333 3143 . 8 3
2
2 2
2
m
x
x x
p
T R n
V
N
N N
N

3
36 . 70 75 . 37 61 . 32
2 2
m V V V
N CO m
+ +
Therefore the final pressure of the gas mixture is,
bar
x
x x
p
m
02 . 2
36 . 70 100
6 . 342 3143 . 8 5

The partial pressure of the individual gases,
bar x xp
n
n
p
m
m
CO
CO
808 . 0 02 . 2
5
2
2
2

bar x xp
n
n
p
m
m
N
N
212 . 1 02 . 2
5
3
2
2

Therefore the entropy change of the mixture,
( ) ( ) ( )
2 2
N CO m
S S S +
( )
1
1
]
1


1 ,
2 ,
2
2
2
2 2
ln ln
CO
CO
CO
m
CO v CO
p
p
R
T
T
c n
( )
1
1
]
1

+
1 ,
2 ,
2
2
2
2 2
ln ln
N
N
N
m
N v N
p
p
R
T
T
c n
128
( ) ( ) 73 . 28 3143 . 8
2
2
+ +
CO v
CO
p
c R c But
=37.04 kJ/kg mole K
( ) ( ) 75 . 20 3143 . 8
2
2
+ +
N v
N
p
c R c and
=29.064 kJ/kg mole K
( ) +
1
]
1


8 . 1
212 . 1
ln 3143 . 8
353
6 . 342
ln 04 . 37 2
m
S
1
]
1

2 . 2
212 . 1
ln 3143 . 8
333
6 . 342
ln 064 . 29 5
=2[-1.108+6.659]+3[0.826+4.957]
=28.45 kJ/ K
129

You might also like